Задачі Medicine
Задачі Medicine
Задачі MEDICINE
Опис:
Факультет іноземних студентів (Medicine)
Перелік питань:
1. A strain of bacteria produced colorless colonies when cultured on standard MacConkey
agar. It was then used to inoculate a Sugar-Iron Agar slant. After overnight incubation,
the tube was entirely yellow and there were numerous bubbles inside the agar. There is
no black precipitate. Which of the following descriptions most accurately describes the
metabolic reasons for these observations?
2. Epidemiologists (using the Kauffman-White classification) report that an outbreak has been
caused by Salmonella enterica, serotype Newport. Structure of which pair of antigens determined
the serotype as Newport?
3. Bacteriologists (using the Kauffman-White classification) made conclusion that outbreak has
been caused by Salmonella enterica, serotype Anatum. Structure of which pair of antigens
determined the serotype as Anatum?
4. A five-day-old child develops high fever and stiff neck. A spinal tap is performed. A Gram-
stained smear of cerebrospinal fluid reveals many neutrophils and short, thick, Gram-negative
rods of uniform length. This microbes can ferment lactose.Which organism below is most likely
to have caused this infection?
A. * Escherichia coli
B. Haemophilus influenzae
C. Streptococcus pneumoniae
D. Neisseria meningitidis
E. Streptococcus agalactiae
6. E. coli and Neisseria meningitidis isolated from blood cultures often have transport systems (for
acquiring metal ions from the environment) lacking in their less-virulent relatives. Which metal is
acquired by the majority of these systems?
A. Copper
B. Sodium
C. Potassium
D. Magnesium
E. * Iron
7. A young woman has a urinary tract infection caused by E. coli. By what route are such infections
most commonly acquired?
C. * Fecal-oral route, followed by colonization of the intestine and then the perineum.
D. Intestinal infection, followed by spread of bacteria to the urethra via the bloodstream.
8. 23-year-old woman comes to your office because, for 3 days, she has experienced burning with
urination, increased frequency of urination, and a continual feeling that she needs to urinate. She
does not have vaginal discharge, fever, or flank pain. Rapid 'dipstick' urine tests are consistent
with uncomplicated cystitis. Culture of urine on standard media produces a lactose-fermenting
Gram-negative rod. In situations such as this, what is the most likely pathogen?
A. Enterobacter faecium
B. * Escherichia coli
C. Klebsiella pneumoniae
D. Proteus vulgaris
E. Pseudomonas aeruginosa
A. * Flagella.
B. Pili.
C. Capsule.
E. Porins.
10. A vacationing couple both develop severe watery diarrhea which lasts three days. Which
organism is the most common etiologic agent of such infections?
A. Shigella sonnei.
B. Pseudomonas aeruginosa
C. Salmonella typhi.
D. Campylobacter jejuni.
E. * Escherichia coli.
11. Several medical students go to Cancun over spring break. One evening, at a local bar, they drink
several pitchers of Margaritas made with crushed ice. They spend the next two days in their hotel
room with severe watery diarrhea, but no chills or fever. By the third day they recover
completely. Which organism is most likely to have been the cause of their illness?
A. Non-typhoidal Salmonella
B. Helicobacter pylori.
C. Campylobacter jejuni.
D. * Enterotoxigenic E. coli.
E. Non-cholera Vibrio.
12. The fimbriae of enterotoxigenic E. coli are essential to the bacterium when it causes disease
because
A. * they are needed for initial attachment of the bacteria to the gastrointestinal epithelium
B. they activate host adenylate cyclase enzymes leading to disruption of cellular permeability
C. they make the bacteria resistant to phagocytic killing and digestion by white blood cells
E. they allow the bacteria to swim "upstream" against the peristaltic action of the GI tract thereby
remaining established in the GI tract
13. The main effect of E. coli and Vibrio cholerae enterotoxin on host cells of the gastrointestinal
tract that leads to the pathology of the diseases, is to
14. A reason not to administer beta lactam antibiotics for a Gram-negative septicemia (blood-borne
infection), unless there is no other alternative, is
E. the antibiotic suppresses the immune response necessary to eliminate the pathogen
15. Water to be used for parenteral injection must sterilized to kill living microbes and then further
purified to remove non-living bacterial components. Of those listed below, which is the most
toxic?
A. * Lipopolysaccharide.
B. Polysaccharides of capsules.
C. Plasma-membrane lipids.
E. Nucleic acids
16. A stool sample is plated on standard Endo-agar plates. After overnight incubation some of the
colonies are dark pink in color, others are pale, nearly colorless. What can you conclude about the
bacteria that produced dark-pink colonies?
A. Gram-positive.
B. * Ferment lactose.
D. Non-motile.
E. Produce capsules
17. A patient with septicemia goes into septic shock. Blood cultures later showed the pathogen to be
Escherichia coli. Which component of the bacteria was most likely to have produced shock?
A. Capsular polysaccharide.
B. Teichoic acid.
C. A superantigen toxin.
D. * Lipopolysaccharide [LPS].
18. A sexually-active woman develops a urinary tract infection which ascends to the kidneys.
Blood culture yields a Gram-negative rod. In this situation, which pathogen is most
common?
A. * Escherichia coli.
B. Klebsiella pneumoniae.
C. Pseudomonas aeruginosa.
D. Salmonella enteritidis.
E. Serratia marcescens
19. Infections with Salmonella enterica, serotype typhi, spread throughout the body. A key to the
ability of serotype typhi to spread systemically is its ability to multiply intracellularly.
Multiplication in which cell type(s) is principally responsible for systemic spread?
A. * Monocytes/macrophages.
B. Basophils.
C. Enterocytes.
D. Erythrocytes.
E. Neutrophils
20. A vacationer has “Traveler’s diarrhea” produced by enterotoxigenic E. coli. His profuse watery
diarrhea is produced by a protein exotoxin secreted by this bacterium. Which statement below
best describes the effect of this exotoxin?
E. Causes rearrangement of the enterocyte cytoskeleton and disappearance of the brush border.
21. A patient becomes infected with Salmonella enterica serotype typhi. Which condition would most
strongly favor development of a “chronic carrier state” with this organism?
C. * Gallstones.
D. Kidney stones.
22. Your patient is a 70-year-old man who under went bowel surgery for colon cancer 3 days ago. He
now has a fever and abdominal pain. You are concerned that he may have peritonitis. Which one
of the following pairs of organisms is MOST likely to be the cause?
23. Which one of the following organisms causing gastrointestinal tract infections is the MOST
frequent agent of bacteremia ?
A. Shigella flexneri
B. Campylobacter jejuni
C. Vibrio cholerae
D. * Salmonella typhi
E. Salmonella enteritica
24. A young woman develops urgency, frequency, and dysuria. Two days later she develops fever,
chills, and costovertebral angle tenderness. Tests of urine for leukocyte esterase and nitrate
reductase are positive. Urine culture yields a beta-hemolytic Gram-negative rod which forms
dark-pink colonies on MacConkey agar. Which of the following organisms is most likely?
A. Pseudomonas aeruginosa
B. Klebsiellla pneumoniae
C. * Escherichia coli
D. Staphylococcus saprophyticus
E. Enterobacter, sp
25. A lactose-fermenting Gram-negative rod is isolated from the bloody stool of a young child.
Which pathogen is most likely?
A. Salmonella enterica.
B. * Escherichia coli.
C. Vibrio cholerae.
D. Shigella dysenteriae.
E. Clostridium difficile.
26. A vacationer develops watery diarrhea but not chills or fever. In this setting which of the
following pathogens is the most common?
A. Salmonella enterica
B. * Escherichia coli
C. Staphylococcus aureus
D. Campylobacter jejuni
E. Helicobacter pylori
27. Which of the following is most associated with Traveler diarrhea; infant diarrhea in developing
countries; watery diarrhea, vomiting, cramps, nausea, and low-grade fever?
28. Epidemiologists (using the Kauffman-White classification) report that an outbreak has been
caused by Salmonella enterica, serotype Newport. Structure of which pair of antigens determined
the serotype as Newport?
29. A five-day-old child develops high fever and stiff neck. A spinal tap is performed. A Gram-
stained smear of cerebrospinal fluid reveals many neutrophils and short, thick, Gram-negative
rods of uniform length. This microbes can ferment lactose.Which organism below is most likely
to have caused this infection?
A. * Escherichia coli
B. Haemophilus influenzae
C. Streptococcus pneumoniae
D. Neisseria meningitidis
E. Streptococcus agalactiae
30. A young woman has a urinary tract infection caused by E. coli. By what route are such infections
most commonly acquired?
C. * Fecal-oral route, followed by colonization of the intestine and then the perineum.
D. Intestinal infection, followed by spread of bacteria to the urethra via the bloodstream.
31. A 23-year-old woman has a history of recurrent urinary tract infections, including at least one
episode of pyelonephritis. Blood typing shows the P blood group antigen. Which of the following
is likely to be the primary cause of her infections7
C. DNA sequencing
33. A 60-year old man was admitted to the hospital 2 weeks previously because of head trauma and
other injuries that resulted from an automobile accident. A urinary tract catheter was inserted at
admission and remains in place. The man develops a urinary tract infection with a gram negative
bacillus. The probable cause of this patients infection is
A. Pseudomonas aeruginosa
B. Providenaa rettgeri
C. Escherichia coli
D. Morganella morgana
34. A 20-year-old college student goes to the student health center because of dysuna, frequency, and
urgency on urination for 24 hours. She has recently become sexually active. On urmalysis, many
polymorphonuclear cells are seen. The most likely organism responsible for these symptoms and
signs is
A. Staphylococcus aureus
B. Streptococcus agalactiae
C. Gardnerella vaginalis
D. Lactobacillus species
E. * Escherichia coli
35. A 27 year-old woman is admitted to the hospital because of fever, with increasing anorexia,
headache, weakness, and altered mental status of 2 days'duration. She works for an airline as a
cabin attendant, flying between the Indian subconti nent and other places in Southeast Asia and
the West Coast of the USA. Ten days prior to admission she had a diarrheal illness that lasted for
about 36 hours. She has been constipated forthe last 3 days. Her temperature is 39 °C, heart rate
68/min, blood pressure 120/80 mm Hg, and respirations 18/mm. She knows who she is and where
she is but does not know the date. She is picking at the bed clothes. Rose spots are seen on the
trunk. The remainder of the physical examination is normal. Blood cultures are done and an
intravenous line is placed. The most likely cause of her illness is
B. Shigella sonnei
36. ?A patient has a positive Rapid Plasma Reagin (RPR) test for ‘non-treponemal antibodies’. A test
for specific anti-treponemal antibodies is positive but the patient has no rash, genital lesions,
inflammation, or discharge. Which conclusion best follows from these findings?
A. The patient does not require antibiotic treatment; absence of symptoms indicates that the positive
RPR is a ‘biological false positive”.
B. The patient has Syphilis, but does not require antibiotic treatment, because infections with
Treponema pallidum normally resolve spontaneously.
C. The patient has Syphilis, but the diagnosis should be confirmed by culture before
antibiotictreatment is started.
E. No correct answer
37. A 16-year-old, sexually active patient comes to your office because of a circular, 1-cm lesion in
the groin area which is ulcerated but not painful. A Rapid Plasma Reagin test is performed and is
reactive with a titer of 1:16. Culture and Gram stain results from an exudate of the lesion are
negative. The most likely cause of this lesion is:
A. Chlamydia trachomatis
B. Neisseria gonorrhoeae
C. Staphylococcus aureus
D. * Treponema pallidum
E. Haemophilus ducreyi
38. Children born with congenital syphilis often exhibit Hutchinson Triad, which includes deafness,
blindness, and centrally notched teeth. If the mother has been exposed to Chlamydia, syphilis, or
gonorrhea, the newborn may receive a prophylactic antibiotic for their:
A. Ears
B. * Eyes
C. Nose
D. Mouth
39. A mother is worried that the child may have contracted Lyme disease. Which finding would most
strongly suggest that she is correct?
C. A lesion at the site of the tick bite that develops a thick, black, crusted, scab.
40. A patient in a rural area of New York had a 5 cm red rash with an expanding margin, on his back,
that lasted for about a month and then went away. Later, the patient had episodes of partial facial
paralysis and painful joints. Which of the following is the most likely infectious agent in this
case?
A. Borrelia hermsii
B. * Borrelia burgdorferi
C. Leptospira interrogans
E. Spirillum minor
41. A mother brings her 12-year-old son to your office because he has a skin lesion on his back. It is
20 cm in diameter, with a red inflamed border and a red center, with a paler ring between the red
areas. The mother says it has expanded rapidly over the last few days. Which of the following
would be most likely to cause such a lesion?
A. Rickettsia rickettsii
B. Treponema pallidum
C. * Borrelia burgdorferi
D. Chlamydia psittaci
E. Streptococcus pyogenes
42. A 16-year-old boy has a skin lesion on his back. It is 20 cm in diameter, with a red inflamed
border and a red center, with a paler ring between the red areas. He says it has expanded rapidly
over the last few days after tick bite. Which of the following would be most likely to cause such a
lesion?
A. Rickettsia rickettsii
B. * Borrelia burgdorferi
C. Treponema pallidum
D. Chlamydia psittaci
E. Streptococcus pyogenes
43. A farmer comes to the ER with a 1-week history of flu-like symptoms with photophobia. His
severe headache, cough, and myalgias suggest to the physician some kind of respiratory infection.
However, more careful physical exam reveals conjunctival suffusion and macular rash. Lab
findings include elevated serum bilirubin, alkaline phosphatase, aminotransferase, and creatine
phosphokinase. With this clinical picture and lab results, the physician prescribes penicillin G
immediately. His suspicions are confirmed later when a spirochete is isolated from the patient’s
blood. Which of the following is the most likely cause?
A. Treponema pallidum
B. * Leptospira interrogans
C. Borrelia burgdorferi
D. Borrelia recurrentis
E. Treponema vincentii
B. Rickettsia akari
C. Ureaplasma urealyticum
D. Mycoplasma pneumoniae
E. Staphylococcus aureus
45. A child comes to your office with a headache, malaise, mild fever, and a rash developing on the
trunk of the body. The mother tells you that the family recently returned from South America,
where each member had picked up a body louse infestation. A possible cause of this disease is:
A. Rickettsia akari
B. Coxiella burnetti
C. Rickettsia typhi
D. Rickettsia rickettsii
E. * Rickettsia prowazekii
46. In August, a young man presents with a history of fever for 1 week and a red rash which extends
to the palms and soles. IgM titer to Rickettsia rickettsii is 1:640. From which vector was this
infection most likely to have been acquired?
A. * Dog tick.
C. Blood-sucking sandfly.
D. Flea.
47. ?A bacterium was isolated from the urine of a 58-year-old male who also was suspected of
having endocarditis. Because of technical problems' the Gram stain was inconclusive. However,
the organism produced beta hemolysis on blood agar, grew well on mannitol salt agar and was
coagulase positive. The most probable identification is
A. Cnlamydia . trachomatis
B. Eschericbia coli
C. * Staphylococcus aureus
D. Pseudomonas aeruginosa
E. Staphylococcus saprophyticus
48. A group of six children under 8 years of age live in a semitropical country. Each of the children
has several crusted weeping skin lesions of impetigo (pyoderma). The lesions are predominantly
on the arms and faces Which of the following microorganisms is a likely cause of the lesions?
A. Escherichia coli
B. Chlamydia trachomatis
C. * Staphylococcus aureus
D. Streptococcus pneumoniae
E. Bacillus anthracis
49. An 11 year-old boy develops a mild fever and pain in his upper arm. An x ray film of his arm
shows a lytic lesion (dissolution) in the upper part of the humerus with penosteal elevation over
the lesion. The patient is taken to surgery, where the lesion is debrided (dead bone and pus
removed). Culture from the lesion yields gram-positive cocci. A test shows that the organism is a
staphylococcus and not a streptococcus. Based on this information, you know the organism is
A. Susceptible to nafcillin
B. Beta lactamase-positive
C. * Catalase positive
D. A producer of protein A
E. Encapsulated
50. A 36-year old male patient has an abscess with a strain of Staphylococcus aureus that is beta-
lactamase positive. This indicates that the organism is resistant to which of the following
antibiotics?
A. Trimethoprim sulfamethoxazole
D. Vancomycin
E. Cefazolmand ceftnaxone
51. A 40-year old man presents to the emergency room with a 3 day history of cough productive of
blood-tinged sputum and a fever. The Gram stain of the sputum has many white blood cells and
Gram-positive cocci. The most likely causative organism is
A. * Staphylococcus aureus
B. Streptococcus pneumoniae
C. Mycoplasma pneumoniae
D. Klebsiella pneumoniae
E. Legionella pneumophila
52. A 54-year-old woman develops a right shoulder abscess with a strain of Staphylococcus
aureus that is resistant to nafcillin. She was treated with a 2-week course of intravenous
vancomycin and improved. Three weeks later (week 5), the infection recurred and she
was given 2 more weeks of intravenous vancomycin and again improved four weeks
later (week 11), the infection recurred and the patient was again started on intravenous
vancomycin. The minimum inhibitory concentrations (MICs) for vancomycin for the S
aureus isolates were as follows initial isolate (day 1), 1 mg/mL, week 5,2 mg/mL, and
week 11,8 mg/mL. The patient failed to improve with the third course of vancomycin, and
alternative therapy was used. The mechanism that best explains the relative resistance
of the patient's strain of S. aureus to vancomycin is
C. Action of beta-lactamase
D. * Increased cell wall synthesis and alterations in the cell wall structure
A. * S. pneumoniae
B. E. coli
C. Clostridium difficile
E. Viridans streptococci
54. Over a period of 3 weeks.a total of five newborns in the hospital nursery developed
Staphylococcus aureus infections with S aureus bacteremia. The isolates all had the same colony
morphology and hemolytic properties and identical antimicrobial susceptibility patterns,
suggesting that they were the same (Later molecular methods showed the isolates were identical).
Which of the following should be done now?
C. * Culture using mannitol salt agar of the anterior nares of the physicians, nurses,and others who
cared for the infected babies
55. Seven days ago, a 27 year old medical student returned from Central America, where she had
spent the summer working in a clinic for indigenous people. Four days ago, she developed an
erythematous sunburn like rash. She also has had headache, muscle aches, and abdominal cramps
with diarrhea. Her blood pressure is 70/40 mm Hg. Pelvic examination shows she is having her
menstrual period with a tampon in place, otherwise, the pelvic examination is normal. Her kidney
function tests (serum urea nitro gen and creatmme) are abnormal, indicating mild renal failure. A
blood smear for malaria is negative. Her illness is likely to be caused by which of the following?
A. A toxin that results in greatly increased levels of intracellular cyclic adenosine monophosphate
(cAMP)
C. * A toxin that binds to the class II major histo compatibility complex (MHC) of an antigen-
presenting cell and the V? region of a T cell
D. A two component toxin that forms pores in white blood cells and increases cation permeability
56. The laboratory calls to alert you that a patient, who has been in the intensive care unit of the
hospital for a long period of time, has several blood cultures with Gram positive cocci in clusters.
You suspect the diagnosis to be line related sepsis due to which organism? ('Line related sepsis' is
sepsis related to the presence of, for example, an IV tube or line.)
A. Group A streptococcus
B. Clostridium difficile
C. * Staphylococcus S. aureus
D. Viridans streptococci
E. E. coli
57. After initial infecting in the childhood which it was finished by a petrification of the locus,
at patient E., immunodefence to a tuberculosis is observed. What does nonsterility
immunity cause at a tuberculosis?
A. Production of antibodies
C. An activation of an englobement
D. * An activation of T lymphocytes
58. Patient I. is preliminary diagnosised the Tuberculosis of lungs. What of the following media is
used to get pure culture of mycobacteria:
A. A casein-coal agar
B. * Levenstaine-Jensen medium
C. A yolk-saline agar
D. A blood agar
E. Endo agar
59. A 26 year old woman visits her physician because of an unusual vaginal discharge. On
examination the physician observes a thin, homogeneous, white-gray discharge that adheres to
the vaginal wall. The pH of the discharge is 5 5 (normal < 4,3). On Gram stain, many epithelial
cells covered with gram-variable rods are seen. Bacterial vaginosis is diagnosed. Which one of
the following normal genital flora microorganisms is present in greatly decreased numbers in
bacterial vaginosis?
A. Corynebactenum species
B. Staphylococcus epidermidis
C. Prevotella species
D. Candida albicans
E. *
Lactobacillus species
60. Certain microorganisms are never considered to be members of the normal flora. They are always
considered to be pathogens. Which one of the following organisms fits into that category?
A. Streptococcus pneumomae
B. Escherichia coli
C. * Mycobacterium tuberculosis
D. Staphylococcus aureus
E. Neisseria meningitidis
61. A 9-year-old girl develops fever and severe pain on the right side of her throat. On examination,
redness and swelling in the right peritonsillar area are seen A peritonsillar abscess is diagnosed.
The most likely organisms to be cultured from this abscess are
A. Staphylococcus aureus
B. Streptococcus pneumomae
62. A 70-year-old man with a history of diverticulosis of the sigmoid colon experiences a sudden
onset of severe left lower quadrant abdominal pain. Fever develops. The severe pain gradually
subsides and is replaced by a constant aching pain and marked abdominal tenderness. A diagnosis
of probable ruptured diverticulum is made and the patient is taken to the operating room. The
diagnosis of ruptured diverticulum is confirmed and an abscess next to the sigmoid colon is found
The most likely bacteria to be found in the abscess are
63. After initial infecting in the childhood which it was finished by a petrification of the locus,
at patient E., immunodefence to a tuberculosis is observed. What does nonsterility
immunity cause at a tuberculosis?
A. Production of antibodies
B.
Activization of system of a complement.
C. An activation of an englobement
D. * An activation of T lymphocytes
E. reduction of IgM
64. Patient I. is preliminary diagnosised the Tuberculosis of lungs. What of the following media is
used to get pure culture of mycobacteria:
A. A casein-coal agar
B. * Levenstaine-Jensen medium
C. A yolk-saline agar
D. A blood agar
E. Endo agar
65. A 26 year old woman visits her physician because of an unusual vaginal discharge. On
examination the physician observes a thin, homogeneous, white-gray discharge that adheres to
the vaginal wall. The pH of the discharge is 5 5 (normal < 4,3). On Gram stain, many epithelial
cells covered with gram-variable rods are seen. Bacterial vaginosis is diagnosed. Which one of
the following normal genital flora microorganisms is present in greatly decreased numbers in
bacterial vaginosis?
A. Corynebactenum species
B. Staphylococcus epidermidis
C. Prevotella species
D. Candida albicans
E. * Lactobacillus species
66. Certain microorganisms are never considered to be members of the normal flora. They are always
considered to be pathogens. Which one of the following organisms fits into that category?
A. Streptococcus pneumomae
B. Escherichia coli
C. * Mycobacterium tuberculosis
D. Staphylococcus aureus
E. Neisseria meningitidis
67. A 9-year-old girl develops fever and severe pain on the right side of her throat. On
examination, redness and swelling in the right peritonsillar area are seen а peritonsillar
abscess is diagnosed. The most likely organisms to be cultured from this abscess are
A. Staphylococcus aureus
B. Streptococcus pneumomae
68. A 70-year-old man with a history of diverticulosis of the sigmoid colon experiences a sudden
onset of severe left lower quadrant abdominal pain. Fever develops. The severe pain gradually
subsides and is replaced by a constant aching pain and marked abdominal tenderness. A diagnosis
of probable ruptured diverticulum is made and the patient is taken to the operating room. The
diagnosis of ruptured diverticulum is confirmed and an abscess next to the sigmoid colon is found
The most likely bacteria to be found in the abscess are
69. It is planned to use the territory of an old cattle burial ground (which is not used for more than 50
years) for building houses. But ground analysis revealed presence of the pathogen of the very
dangerous illness. Which of the indicated microorgonisms is likely to remain in the ground for
such a long time?
A. * Bacillus anthracis
B. Francisella tularensis
C. Brucella abortus
D. Yersinia pestis
E. Mycobacterium bovis
70. From the nasopharynx of a 5-year-old child it was excreted amicroorganism which is identical to
Corynebacterium diphtheriae dose according to morphological and biochemical
signs.Microorganism does not produce exotoxin. As a result of what process can this
microorganism become toxigenic?
A. * Phage conversion
71. While studying a microslide obtained from the punctuate of a regional lymph node and stained by
Romanovsky-Giemsa method a physician revealed some light-pink thin microorganisms with 12-
14 regular spiral coils and pointed ends, up to 10-13 micrometer long. This might be the causative
agent of the following disease:
A. * Syphilis
B. Trypanosomiasis
C. Leptospirosis
D. Relapsing fever
E. Leishmaniasis
72. Sanitary bacteriological research on water by the membrane filter method revealed two red
colonies on a membrane filter (Endo agar) through which 500 ml of analyzed water were passed.
Calculate the coli index and coli titer of the analyzed water:
A. * 4 and 250
B. 2 and 500
C. 250 and 4
D. 500 and 2
E. 250 and 2
73. While examining a patient an otolaryngologist noticed hyperaemia and significantly edematous
tonsils with a grayish film upon them. Microscopical examination of this film revealed some
gram-positive bacilli placed at an angle with each other. What disease might be suspected?
A. * Diphtheria
B. Angina
C. Scarlet fever
D. Meningococcal nasopharyngitis
E. Epidemic parotitis
74. Patient with vomiting, dizziness, sensation of dubble vision, difficult swallowing was admitted to
the hospital. Doctor suspects botulism. What diagnostic methods should be used for diagnosis
approving?
C. Bacteriological, mycological
D. Protozoological, microscopical
E. Allergic test
75. A man who was bitten by the unknown dog applied to the surgeon. Wide ragged woundes were
localised on the face. What curative-prophylactic aid should be given to prevent rabies?
76. A patient with complaints of 3-day-long fever, general weakness, loss of appetite came to visit
the infectionist. The doctor suspected enteric fever. Which method of laboratory diagnosis is the
best to confirm the diagnosis?
B. Detachment of myeloculture
77. A consumptive patient has an open pulmonary form of disease. Choose what sputum staining
should be selected for finding out the tubercle (Koch's) bacillus?
A. * Method of Ziel-Neelsen
B. Method of Romanowsky-Giemsa
C. Method of Gram
D. Method of Neisser
E. Method of Burry-Gins
78. During surgical operation a blood transfusion was made. The blood must be checked to find
antigens of some disease. What disease is expected to be found?
A. * Virus of hepatitis B
B. Virus of hepatitis A
C. Adenovirus
D. Enterovirus
E. Virus of hepatitis E
79. A 42-year-old man who has been injured in a car accident is brought into the emergency room.
His blood alcohol level on admission is 250 mg/dL. Hospital records show a prior hospitalization
for alcohol related seizures. His wife confirms that he has been drinking heavily for 3 weeks.
What treatment should be provided to the patient if he goes into withdrawal?
A. * Diazepam
B. Phenobarbital
C. Pentobarbital
D. Phenytoin
E. None
80. A patient who came to the doctor because of his infertility was administered to make tests for
toxoplasmosis and chronic gonorrhoea. Which reaction should be performed to reveal
C. Immunoblot analysis
81. On bacteriological study of rinsing water of the patient with food poisoning, the pure bacterial
culture was inoculated with the following properties: gram-negative motile bacillus in the Endo
environment grows like achromic colony. Representative of what genus has caused the illness?
A. * Salmonella
B. Shigella
C. Yersinia
D. Escherichia
E. Citrobacter
82. The person was selling homemade pork sausages on the market. State sanitary
inspector suspected falcification of the sausages.With help of what serological immune
reaction can food substance be identified?
A. * Precipitation test
D. Immunofluorescence test
83. While registering the child to the school Mantu's test was made to define whether revaccination
was needed test result is negative. What does this result of the test mean?
84. The donor who for a long time didn't donate the blood was investigated with ELISA
method. Anti-HBs antibodies were revealed. What does positive result of IFA in this case
mean?
A. * Previous hepatitis B
B. Acute hepatitis B
C. Acute hepatitis C
D. Chronic hepatitis C
E. Chronic hepatitis B
85. Bacteriological examination of purulent discharges from the urethra revealed gram-
negative bacteria looking like coffee beans. They were localized in the leukocytes and
could decompose glucose to acid. These are the causative agents of the following
disease:
A. * Gonorrhoea
B. Syphilis
C. Veneral lymphogranulomatosis
D. Soft chancre
E. Melioidosis
86. Scraps of the mycelium of a fungus, spores, air bubbles and fat drops were discovered on
microscopy of the patient's hair excluded from the infected areas. For what fungus disease is this
microscopic picture characteristic?
A. * Favus
B. Microspory
C. Trichophytosis
D. Epidermophytosis
E. Sporotrichosis
87. In order to speed up healing of a wound of oral mucosa a patient was prescribed a drug that is a
thermostable protein occuring in tears, saliva, mother's milk as well as in a new-laid hen's egg. It
is known that this protein is a factor of natural resistance of an organism. What is it called?
A. * Lysozyme
B. Complement
C. Interferon
D. Interleukin
E. Imanine
88. Study of bacteriological sputum specimens stained by the Ziel-Neelsen method revealed some
bright-red acid-resistant bacilli that were found in groups or singularly. When inoculated onto the
nutrient media, the signs of their growth show up on the 10-15 day. These bacteria relate to the
following family:
A. * Micobacterium tuberculosis
B. Yersinia pseudotuberculosis
C. Histoplasma dubrosii
D. Klebsiella rhinoscleromatis
E. Coxiella burnettii
89. A man was admitted to the hospital on the 5th day of disease that manifested itself by jaundice,
muscle aching, chill, nose bleedings. In course of laboratory diagnostics a bacteriologist
performed dark-field microscopy of the patient's blood drop. Name a causative agent of this
disease:
A. * Leptospira interrogans
B. Borrelia dutlonii
C. Calymmatobacterium granulomatis
D. Bartonella bacilloformis
E. Rickettsia mooseri
90. Gramnegative bin-shaped diplococcus inside and outside of leucocytes were detected on
bacteriological examination of the purulent exudates from the cervix of the uterus. Name the
causative agent of purulent inflammation of the cervix of the uterus.
A. * Neisseria gonorroeae
B. Chlamidia trachomatis
C. Haemophilus vaginalis
D. Trichomonas vaginalis
E. Calymmatobacterium granulomatis
91. Patient with diarrhoea was admitted to the infection unit. Gramnegative curved rod-like bacteria
were founded on bacterioscopic examination of faecal masses. What is the most likely disease in
this patient?
A. * Cholera
B. Typhoid fever
C. Salmonellosis gastroenteritis
D. Diphtheria
92. In a 2-year-old child with catarrhal presentations and skin rash a pediatrician suspected scarlet
fever. The child was given intracutaneously a small dose of serum antibody to the streptococcal
erythrogenic toxin+ADs- on the site of injection the rash disappeared. What do the reaction
results mean?
93. From the defecation of a 6-year-old ill child, who has artificial feeding, the intestinal bacillus
with antigen structure 0-111 is excreted. What is the diagnosis?
A. * Coli-enteritis
B. Gastroenteritis
C. Cholera-like diseasis
D. Food poisoning
E. Dysentery-like diseasis
94. For serological diagnostics of the whooping cough it was made large-scale reaction with
parapertussis and pertussis diagnosticums. At the bottom of the test-tubes with diagnosticum of
Bordetella parapertussis grain-like sediment formed. What antibodies have this reaction
revealed?
A. * Agglutinins
B. Precipitins
C. Opsonins
D. Bacteriolysins
E. Antitoxins
95. A man died from an acute infectious disease accompanied by fever, jaundice haemorrhagic rash
on the skin and mucous membranes as well as by acute renal insufficiency. Histological
examination of renal tissue (stained by Romanovsky-Giemsa method revealed some convoluted
bacteria looking like C und S letters. What bacteria were revealed?
A. * Leptospira
B. Treponema
C. Spirilla
D. Borrelia
E. Campilobacteria
96. A 16 y.o. boy from a countryside entered an educational establishment. Scheduled Mantoux test
revealed that the boy had negative reaction. What are the most reasonable actions in this case?
97. Examination of a patient with pustular skin lesions allowed to isolate a causative agent that forms
in the blood agar roundish yellow middle-sized colonies surrounded by haemolysis zone. Smears
from the colonies contain irregular-shaped clusters of gram-positive cocci. The culture is oxidase-
and catalase-positive, ferments mannitol and synthesizes plasmocoagulase. What causative agent
was isolated?
A. * Staphylococcus aureus
B. Streptococcus agalactiae
C. Streptococcus pyogenes
D. Staphylococcus epidermidis
E. Staphylococcus saprophyticus
98. Microscopic examination of a Gram-stained scrape from patient's tongue revealed oval, round,
elongated chains of dark-violet gemmating cells. What disease can be caused by this causative
agent?
A. * Candidosis
B. Actinomycosis
C. Streptococcic infection
D. Staphylococcic infection
E. Diphtheria
99. From pharynx of a child with suspected diphtheria a pure culture of microorganisms was isolated.
Their morphological, tinctorial, cultural and biochemical properties appeared to be typical for
diphtheria causative agents. What study should be conducted in order to drow a conclusion that
this is a pathogenic diphtheria bacillus?
100. Examination of a child revealed some whitish spots looking like coagulated milk on the mucous
membrane of his cheeks and tongue. Analysis of smears revealed gram-positive oval yeast-like
cells. What causative agents are they?
A. * Candida
B. Staphylococci
C. Diphtheria bacillus
D. Actinomycetes
E. Fusobacteria
101. A duodenal content smear of a patient with indigestion contains protosoa 10-18 mcm large. They
have piriform bodies, 4 pairs of filaments, two symmetrically located nuclei in the broadened part
of body. What kind of the lowest organisms is it?
A. * Lamblia
B. Dysentery ameba
C. Trichomonas
D. Intestinal ameba
E. Balantidium
102. Blood of a patient with presumable sepsis was inoculated into sugar broth. There appeared
bottom sediment. Repeated inoculation into blood agar caused growth of small transparent round
colonies surrounded by hemolysis zone. Examination of a smear from the sediment revealed
gram-positive cocci in form of long chains. What microorganisms are present in blood of this
patient?
A. * Streptococci
B. Micrococci
C. Staphylococci
D. Tetracocci
E. Sarcina
103. On bacteriological examination of the defecation of a 4-months-old baby with the symptoms of
acute bowel infection there were revealed red colonies spread in the large quantity in the Endo
medium. What microorganism can it be?
A. * Escherichia
B. Salmonella
C. Staphylococcus
D. Streptococcus
E. Shigella
104. Bacteriological examination of a patient with food poisoning required inoculation of a pure
culture of bacteria with the following properties: gram-negative movable bacillus that grows n the
Endo medium in form of colourless colonies. A representative of which species caused this
disease?
A. * Salmonella
B. Shigella
C. Yersinia
D. Esherichia
E. Citrobacter
105. Examination of a young man in the AIDS centre produced a positive result of immune-enzyme
assay with HIV antigens. Patient's complaints about state of his health were absent. What can the
positive result of immune-enzyme assay be evidence of?
A. * HIV infection
E. HBV persistence
106. Microscopy of stained (Ziehl-Neelsen staining) smears taken from the sputum of a patient with
chronic pulmonary disease revealed red bacilli. What property of tuberculous bacillus was shown
up?
A. * Acid resistance
B. Alkali resistance
C. Alcohol resistance
D. Capsule formation
E. Sporification
108. In order to determine toxigenicity of diphtheria bacilli a strip of filter paper impregnated with
antitoxic diphtherial serum was put on the dense nutrient medium. There were also inoculated a
microbal culture under examination and a strain that is known to be toxigenic. If the microbal
culture under examination produces exotoxin, this wil result in formation of:
A. * Precipitin lines
B. Haemolysis zones
E. Precipitin ring
109. A 50-year-old patient with typhoid fever was treated with Levomycetin, the next day his
condition became worse, temperature rised to 39,60C. What caused worthening?
B. Allergic reaction
E. Reinfection
110. In order to estimate toxigenity of diphtheria agents obtained from patients the cultures were
inoculated on Petri dish with nutrient agar on either side of a filter paper strip that was put into
the center and moistened with antidiphtheric antitoxic serum. After incubation of inoculations in
agar the strip-like areas of medium turbidity were found between separate cultures and the strip of
filter paper. What immunological reaction was conducted?
B. Coomb's test
C. Agglutination reaction
E. Opsonization reaction
111. A patient with clinical signs of encephalitis was delivered to the infectious diseases hospital.
Anamnesis registers a tick bite. Hemagglutination-inhibition reaction helped to reveal antibodies
to the causative agent of tick-borne encephalitis in the dilution 1:20 which is not diagnostic. What
actions should the doctor take after he had got such result?
112. The first grade pupils were examined in order to sort out children for tuberculosis revaccination.
What test was applied for this purpose?
A. * Mantoux test
B. Schick test
E. Anthraxine test
113. Clinical diagnosis of a female patient was gonorrhoea. What examination method can be applied
for confirmation of this diagnosis?
D. Hemagglutination reaction
E. Immobilization reaction
114. A patient suffering from periodical attacks caused by inhalation of different flavoring substances
was diagnosed with atopic bronchial asthma. IgE level was increased. This is typical for the
following type of reactions:
A. * Anaphylactic reactions
B. Cytotoxic reactions
C. Immunocomplex reactions
D. delayed-type hypersensitivity
E. Autoimmune reactions
115. Bacteriological laboratory examines canned meat whether it contains botulinum toxin. For this
purpose an extract of test specimen and antitoxic antibotulinic serum of A, B, E types were
introducted to a group of mice under examination+ADs- a control group of mice got the extract
without antibotulinic serum. What serological reaction was applied?
A. * Neutralization
B. Precipitation
C. Complement binding
D. Opsono-phagocytic
116. For the purpose of retrtospective diagnostics of recent bacterial dysentery it was decided to
perform serological examination of blood serum in order to determine antibody titer towards
Shiga bacilli. What of the following reactions should be applied?
A. * Passive hemagglutination
B. Bordet-Gengou test
C. Precipitation
D. Hemolysis
E. Bacteriolysis
117. During the repeated Widal agglutination test it was noticed that the ratio of antibody titers and O-
antigens S. typhi in the patient's serum had increased from 1:100 to 1:400. How would you
interpret these results?
118. A patient recovered from Sonne dysentery and was once more infected with the same causative
agent. What is such infection form called?
A. * Reinfection
B. Recidivation
C. Superinfection
D. Persisting infection
E. Chronic infection
119. A 10-year-old child had the mantoux tuberculin test administered. 48 hours later a papule up to 8
mm in diameter appeared on the site of the injection. What type of hypersensitivity reaction
developed after the tuberculin injection?
B. Arthus phenomenon
C. Seroreaction
D. Atopic reaction
121. As a result of durative antibiotic therapy a 37-year old patient developed intestinal dysbacteriosis.
What type of drugs should be used in order to normalize intestinal microflora?
A. * Eubiotics
B. Sulfanilamides
C. Bacteriophages
D. Autovaccines
E. Vitamins
122. Among junior children of an orphanage an outbreak of intestinal infection with signs of
colienteritis was registered. In order to identify isolated causative agent it is necessary to:
123. Urine examination of a patient with acute cystitis revealed leukocytes and a lot of gram-negative
bacilli. Inoculation resulted in growth of colonies of mucous nature that formed green soluble
pigment. What microorganism is the most probable cause of the disease?
A. * Pseudomonas aeruginosa
B. Escherihia coli
C. Klebsiella pneumoniae
D. Proteus mirabilis
E. Salmonella enteritidis
124. A laboratory received a material from a patient's wound. Preliminary diagnosis is gaseous
gangrene. What microbiological method should be applied to determine species of causative
agent?
A. * Bacteriological
B. Allergic
C. Bacterioscopic
D. Serological
E. RIA
125. A virological laboratory obtained pathological material (mucous discharges from nasal meatuses)
taken from a patient with provisional diagnosis +ACI-influenza+ACI-. What quick test will allow
to reveal specific viral antigen in the material under examination?
D. Radioimmunoassay
E. ELISA
126. In the surgical department of a hospital there was an outbreak of hospital infection that showed
itself in often postoperative wound abscesses. Bacteriological examination of pus revealed
Staphylococcus. What examination shall be conducted to find out the source of this causative
agent among the department personnel?
A. * Phagotyping
B. Microscopical examination
C. Serological identification
E. Biochemical identification
127. A 7 year old child often suffers from streprococcal angina. Doctor suspected development of
rheumatism and administered serological examination. The provisional diagnosis will be most
probably confirmed by presence of antibodies to the following streptococcal antigen:
A. * O-streptolysin
B. C-carbohydrate
C. M-protein
D. Erythrogenic toxin
E. Capsular polysaccharide
128. A culture of monkey cells (Vero) and a group of sucking mouse were infected with an
inoculum taken from a child with provisional diagnosis enterovirus infection. There was
no cytopathic effect on the cell culture but mouse sucking died. What enteric viruses
might have caused disease of this child?
A. * Coxsackie A
B. Coxsackie B
C. ECHO virus
D. Polioviruses
129. A patient has been suffering from elevated temperature and attacks of typical cough for 10 days.
Doctor administered inoculation of mucus from the patient's nasopharynx on the agar. What
microorganism is presumed?
A. * Pertussis bacillus
B. Pfeiffer's bacillus
C. Listeria
D. Klebsiella
E. Staphylococcus
130. A female patient underwent liver transplantation. 1,5 month after it her condition became worse
because of reaction of transplant rejection. What factor of immune system plays the leading part
in this reaction?
A. * T-killers
B. Interleukin-1
C. Natural killers
D. B-lymphocytes
E. T-helpers
A. * Clostridia
B. Streptococci
C. Spirochaete
D. Actinomycete
E. Diplococci
132. A specimen stained by Ozheshko method contains rod-like microorganisms stained blue with
round terminal components stained red. What are these components called?
A. * Spores
B. Cilia
C. Flagella
D. Capsules
E. Mesosomas
A. * Haemolytic streptococci
B. Micrococci
C. Bacilli
D. Yeast fungi
E. Sarcina
134. A bacteriological laboratory received sputum sample of a patient suffering from tuberculosis.
Bacterioscopic examination of smears and detection of tuberculosis bacillus can be realized by
one of enrichment methods that involves processing of sputum only with solution of caustic soda.
What is this method called?
A. * Homogenization
B. Inactivation
C. Flotation
D. Filtration
E. Neutralization
135. A pregnant woman was registered in an antenatal clinic and underwent complex examination for
a number of infections. Blood serum contained IgM to the rubella virus. What is this result
indicative of?
A. * Of primary infection
B. Of a chronic process
A. * Staphylococcus aureus
B. Streptococcus pyogenes
C. Staphylococcus epidermidis
D. Staphylococcus saprophyticus
E. Klebsiella pneumoniae
137. Material taken from a patient with provisional diagnosis +ACI-influenza+ACI- was referred to a
laboratory. For virological examination the hemadsorption reaction was applied. This reaction
can be applied for detection of the following viruses:
D. DNA-genomic viruses
E. Any viruses
138. Inoculum from pharynx of a patient ill with angina was inoculated into blood-tellurite agar. It
resulted in growth of grey, radially striated (in form of rosettes) colonies 4-5 mm in diameter.
Gram-positive bacilli with clublike thickenings on their ends placed in form of spread wide apart
fingers are visible by microscope. What microorganisms are these?
A. * Diphtheria corynebacteria
B. Botulism clostridia
C. Diphtheroids
D. Streptococci
E. Streptobacilli
139. During examination of a patient a dentist revealed a lot of white spots+ - zones of
enamel demineralization. What microorganisms take part in the development of this
process?
A. * Streptococcus mutans
B. Streptococcus salivarius
C. Streptococcus pyogenes
D. Veilonella parvula
E. Staphylococcus epidermidis
140. Planned mass vaccination of all newborn 5-7 day old children against tuberculosis plays an
important role in tuberculosis prevention. In this case the following vaccine is applied:
A. * BCG
E. TABTe
141. Vomiting matters of a patient suspected of having cholera were delivered to the bacteriological
laboratory. The material was used for preparing a +ACI-hanging drop+ACI- specimen. What type
of microscopy will be applied for identification of the causative agent by its mobility?
A. * Phase-contrast microscopy
B. Electron microscopy
D. Fluorescence microscopy
E. Immersion microscopy
142. In the surgical ward, the dressing material was undergoing sterilization in an autoclave. Through
an oversight of a nurse the mode of sterilization was changed and the temperature in the
autoclave reached only 100+ALo-C instead of the due 120+ALo-C. What microorganisms can
stay viable under these conditions?
143. In course of long-term treatment of an infectious patient with penicillin, the pathogen transformed
into the L-form. What changes occur in the pathogen cell in case of L-transformation?
B. Absence of flagella
C. Absence of a capsule
D. Absence of a spore
E. Absence of inclusions
144. There are areas where humans or animals are exposed to the constant risk of contracting certain
types of bacteria. What feature of these bacteria is responsible for their long viability in the soil?
A. * Spore formation
B. Capsule formation
E. Plasmids
145. Analysis of the cerebrospinal fluid of a child with signs of purulent lesion of brain tunics revealed
gram-negative bean-shaped diplococci. What presumptive diagnosis can be made on the basis of
the analysis results?
A. * Meningitis
B. Gonorrhea
C. Cholera
D. Plague
E. Anthrax
146. The causative agent of botulism causes severe food poisoning. Specify the most characteristic
morphological feature of botulism causative agent:
147. Microbiological analysis of medicinal raw materials revealed capsular bacteria. What stain
method was used to detect the capsules?
A. * Burry-Gins
B. Ziehl-Neelsen
C. Neisser
D. Gram
E. Ozheshko
148. Bacterioscopic examination of chancre material revealed some mobile, long, convoluted
microorganisms with 8-12 regular coils. These features are typical for:
A. * Treponema
B. Borrellia
C. Leptospira
D. Vibrios
E. Campylobacter
149. Infectious agents of various ultrastructures can be etiological agents of infectious diseases. Which
of the groups named below HAS NO cellular structure, protein synthesizing, enzyme and energy
systems?
A. * Viruses
B. Fungi
C. Bacteria
D. Protozoa
E. Rickettsia
150. What method of sterilization should be used during the liquid media containing proteins?
A. * Filtering
B. Boiling
C. Gas sterilization
D. Autoclaving
E. Pasteurization
151. Meat peptone broth is prepared for sterilization in bateriological laboratory. What sterilization
method is advisable?
A. * Autoclaving
B. Ignition
C. Boiling
D. Filtering
E. Dry heat
152. A patient has a necrotizing phlegmon of his lower extremity. A doctor suspects a gas gangrene.
Microscopy reveals Gram-positive bacilli. In order to confirm the diagnosis further
bacteriological tests should include inoculation of the material into the following nutrient
medium:
A. * Kitt-Tarozzi medium
B. Endo agar
C. Levine agar
D. Meat-peptone agar
E. Milk-salt agar
153. Bacteria eventually become resistant to antibacterial agents. Resistance of gram-positive bacteria
to penicillin antibiotics is caused by:
A. * Beta-lactamase production
E. Protein synthesis
154. Which one of the listed substances causes formation of acquired artificial passive immunity?
A. * Tetanus antitoxin
B. BCG
C. MMR
D. DPT
E. TABTe
155. It is necessary to carry out preventive vaccination of a student group because of an occurrence of
diphtheria. Which preparation should be used for the creation of the artificial active immunity?
A. * Diphtheria toxoid
B. Specific immunoglobulin
C. DTP vaccine
E. Anti-diphtheria serum
156. For the specific prevention of influenza, the employees of an enterprise were vaccinated
with Influvac. What type of immunity will develop in the body of the vaccinated?
A. * Artificial active
B. Innate congenital
C. Artificial passive
D. Natural active
E. Natural passive
157. Dysbiosis can be treated with drugs that contain living representatives of normal microflora as
well as their metabolic products. Select the microorganisms that are used for the production of
such drugs:
A. * Bifidum bacteria
B. Staphylococcus aureus
C. Proteus
D. Providencia
E. Yersinia
158. A 26-year-old female patient with bronchitis has been administered a broad spectrum antibiotic as
a causal treatment drug. Specify this drug:
A. * Doxycycline
B. Interferon
C. BCG vaccine
D. Ambroxol
E. Dexamethasone
159. What condition may develop 15-30 minutes after re-administration of the antigen as a result of
the increased level of antibodies, mainly IgE, that are adsorbed on the surface of target cells,
namely tissue basophils (mast cells) and blood basophils?
A. * Anaphylaxis
B. Antibody-dependent cytotoxicity
C. Delayed-type hypersensitivity
E. Serum sickness
160. 10 days after having quinsy caused by beta-hemolytic streptococcus a 6-year-old child exhibited
symptoms of glomerulonephritis. What mechanism of glomerular lesion is most likely in this
case?
A. * Immunocomplex
B. Cellular cytotoxicity
C. Anaphylaxis
D. Atopy
161. 6 hours after the initial inoculation of water sample into 1+ACU- peptone water, the growth of a
culture in form of a thin pellicle on the medium surface was registered. Such cultural properties
are typical for the causative agent of the following disease:
A. * Cholera
B. Plague
C. Tuberculosis
D. Dysentery
E. Pseudotuberculosis
162. Analysis of the cerebrospinal fluid of a child with signs of purulent lesion of brain tunics revealed
gram-negative bean-shaped diplococci. What presumptive diagnosis can be made on the basis of
the analysis results?
A. * Meningitis
B. Gonorrhea
C. Cholera
D. Plague
E. Anthrax
163. Microbiological purity of tabulated drugs had been tested at factory. Samples cultivation
in mannitol salt agar resulted in growth of golden-yellow colonies, microscopic
examination of colonies detected Gram-positive globular bacteria positioned in clusters
microorganisms had plasma coagulation properties. What pure bacterial culture was
obtained?
A. * Staphylococcus aureus
B. Escherichia coli
C. Staphylococcus epidermidis
D. Staphylococcus saprophyticus
E. Pseudomonas aeruginosa
164. Microbiological studies of air in the pharmacy room revealed the presence of pathogenic
staphylococci. Select the medium in which you can detect the lecithinase activity of the isolated
microorganism:
A. * Yolk-salt agar
B. Blood agar
D. Sugar agar
E. Meat-extract agar
165. There is a suspicion of active tuberculosis development in patient. The doctor has appointed
Mantoux test to make a diagnosis. What immunobiological agent has to be administered?
A. * Tuberculin
B. BCG vaccine
C. DPT vaccine
D. Tularin
E. DT vaccine
166. A smear from the tonsillar coating of a patient with suspected diphtheria was found to contain
blue bacilli with a thickening at the poles. What method of smear staining was used?
A. * Leffler
B. Burri
C. Hins
D. Gram
E. Neisser
167. A young woman suddenly developed fever up to 39oC accompanied by a strong headache.
Examination revealed marked nuchal rigidity. Spinal puncture was performed. Gram-stained
smear of cerebrospinal fluid contained many neutrophils and Gram-negative diplococci. What
bacteria could be the cause of this disease?
A. * Neisseria meningitidis
B. Streptococcus pneumonia
C. Haemophilus influenzae
D. Staphylococcus aureus
E. Pseudomonas aeruginosa
168. Botulism agent causes severe food toxicoinfection. Point out the most characteristic morphologic
feature of botulism agent.
169. The causative agent of tetanus causes severe disease. Specify the most characteristic
morphological feature of tetanus causative agent:
170. A bacteriological laboratory studied the home-made dried fish which had caused a severe food
poisoning. Microscopy of the culture grown on the Kitt-Tarozzi medium revealed
microorganisms resembling a tennis racket. What is the most likely diagnosis?
A. * Botulism
B. Salmonellosis
C. Cholera
D. Dysentery
E. Typhoid fever
171. Quite often, the soil may contain a number of pathogenic microorganisms. The causative agents
of the following disease may stay viable in the soil for a long time:
A. * Anthrax
B. Diphtheria
C. Viral hepatitis
D. Pertussis
E. Dysentery
172. A patient presents with fever, chill and cough. From his sputum the ovoid Gram-negative bipolar-
stained bacilli with a delicate capsule were isolated. What is the most likely diagnosis?
A. * Plague
B. Tuberculosis
C. Leptospirosis
D. Brucellosis
E. Toxoplasmosis
173. For cultivation of Brucella pure cultures should be incubated in CO2 enriched atmosphere. What
type of breathing is typical for Brucella?
A. * Capnophilic
B. Facultative anaerobic
C. Obligate anaerobic
D. Obligate aerobic
E. Any
174. A patient has been hospitalised with provisional diagnosis of botulism. What serological reaction
should be used to reveal botulinum toxin?
A. * Neutralization reaction
B. Agglutination reaction
C. Bordet-Gengou test
D. Precipitation reaction
E. Immunofluorescence test
175. In a village, a case of anthrax had been registered. Medical services began epidemiologically
indicated specific prophylaxis of population against anthrax. What preparation was used for this
purpose?
A. * Live vaccine
B. Inactivated vaccine
C. Chemical vaccine
E.
Anatoxin
176. A 55-year-old patient with a characteristic rash, fever, dizziness has been admitted to a hospital.
He has been provisionally diagnosed with typhus. No similar cases have been reported. In his
youth (15 years old) the patient suffered typhus in a boarding school. What disease is it?
A. * Brill disease
B. Typhoid fever
C. Measles
D. Rubella
E. Cholera
177. A patient has severe catarrhal symptoms. Material growth on Bordet-Gengou agar showed
mercury-drop like colonies. Examination of the blood smears revealed some small ovoid gram
positive bacilli sized 1-3 microns. What microorganisms were isolated?
A. * Bordetella
B. Corynebacteria
C. Mycobacteria
D. Meningococcus
E. Brucella
178. The laboratory for especially dangerous infections conducts microscopic examination of
pathological material from a patient with suspected plague. The sample was stained by Burri-
Gins technique. What property of the causative agent can be identified by this technique?
A. * Capsule formation
B. Spore formation
C. Acid resistance
D. Alkali resistance
179. A child cut his leg with a piece of glass while playing and was brought to the clinic for the
injection of tetanus toxoid. In order to prevent the development of anaphylactic shock the serum
was administered by Bezredka method. What mechanism underlies this method of desensitization
of the body?
180. A 32-year-old patient undergoing dental examination was found to have some rash-like lesions
resembling secondary syphilis in the oral cavity. The patient was referred for the serological
study with the purpose of diagnosis confirmation. In order to detect antibodies in the serum,
living Treponema were used as diagnosticum. What serological test was performed?
A. * Immobilization
B. Neutralization
C. Complement binding
D. Precipitation
E. Passive hemagglutination
181. Examination of a 27-year-old donor who had not donated blood for a long time revealed HBs
antibodies detected by ELISA method. In this case, the positive ELISA results indicate that the
donor:
A. * Had hepatitis B
182. In the area being the epicenter of the registered rabies cases among wild animals a 43-year-old
man presented to a clinic and claimed to have been bitten by a stray dog. He was given a course
of anti-rabies vaccine. This preparation relates to the following type of vaccines:
A. * Attenuated
B. Inactivated
C. Molecular
D. Toxoids
E. Synthetic
183. RNA-containing viruses that cause tumors in animals, genetic information can be transmitted in
the opposite direction from the RNA to the DNA via a specific enzyme. The enzyme of reverse
transcription is called:
A. * Reverse transcriptase
B. DNA polymerase
C. Ligase
D. Primase
E. Topoisomerase
184. A patient has been hospitalised with provisional diagnosis of virus B hepatitis. Serological
reaction based on complementation of antigen with antibody chemically bound to peroxidise or
alkaline phosphatase has been used for disease diagnostics. What is the name of the applied
serological reaction?
A. * Immune-enzyme analysis
B. Radioimmunoassay technique
C. Immunofluorescence test
D. Bordet-Gengou test
E. Antigen-binding assay
185. A 3-year-old child has continuous fever, lymph nodes are enlarged, the amount of lymphocytes in
blood is significantly increased. Enzyme linked immunosorbent assay (ELISA) revealed antigen
of Epstein-Barr virus. What diagnosis can be made based on the information given above?
A. * Infectious mononucleosis
B. Burkitt+IBk-s lymphoma
C. Herpetic lymphadenopathy
E. Cytomegalovirus infection
186. Hepatitis B is diagnosed through laboratory tests that determine the presence of HBV-DNA in
blood serum of the patient. What reference method is applied for this purpose?
B. Hybridization method
187. An outbreak of an intestinal infection occurred in a kindergarten on the eve of New Year
holidays. Bacteriological examination of patients+IBk- faeces didn+IBk-t reveal any pathogenic
bacteria. Electronmicroscopy revealed roundish structures with clear outer edges and a thick core
resembling a wheel. Specify the most likely causative agent of this infection:
A. * Rotavirus
B. Adenovirus
C. Coxsacki-virus
D. E. coli
E. P. vulgaris
188. A hospitalized patient bitten by a rabid animal has an avulsive wound of shin. What kind of
vaccine must be given to prevent rabies?
A. * Anti-rabies vaccine
B. DTaP
C. Td
D. BCG
E. TABte
189. A patient has been admitted to the infectious diseases department for malaise, fever up to 38oC,
jaundice. A few months ago, the patient underwent blood transfusion. The doctor suspected viral
hepatitis B. What are the principal methods of laboratory diagnosis of hepatitis B?
B. Virus isolation in cell culture and its identification by the cytopathic effects
190. In our country, routine preventive vaccinations against poliomyelitis ivolve using live vaccine
that is administered orally. What immunoglobulins are responsible for the development of local
post-vaccination immunity in this case?
A. * Secretory IgA
B. IgM
C. IgG
D. Serum IgA
E. IgE
191. A laboratory received a sample of water used in drug production for sanitary and virological
analysis. What group of viruses will indicate faecal contamination of water and thus the need for
its additional purification?
A. * Picornaviridae
B. Herpesviridae
C. Orthomyxoviridae
D. Retroviridae
E. Flaviviridae
192. A sample of water used in drug production has been sent to a laboratory for sanitary and
virological analysis. Presence of what virus group will be indicative of faecal contamination of
water and thus the need for its additional purification?
A. * Picornaviridae
B. Herpesviridae
C. Orthomyxoviridae
D. Retroviridae
E. Flaviviridae
193. Virological laboratory has received patient+IBk-s nasopharyngeal lavage. What can be used to
single out influenza virus from the patient+IBk-s lavage?
A. * Chick embryo
D. Endo+IBk-s medium
E. Lowenstein+IBM-Jensen medium
194. A person has been in contact with influenza patient. What drug should be administered for
specific passive influenza prophylaxis?
A. Amizon
C. Leukocytic interferon
D. * Immunoglobulin
E. Anaferon
195. A child entering the school for the first time was given Mantoux test in order to determine if there
was a need for revaccination. The reaction was negative. What is the meaning of this test result?
A. * No cell-mediated immunity to tuberculosis
196. Bacteriological examination of purulent discharges from the urethra revealed some gram-negative
bean-shaped bacteria located in the leukocytes. They can be identified as the causative agent of
the following disease:
A. * Gonorrhea
B. Syphilis
C. Venereal lymphogranulomatosis
D. Chancroid
E. Trichomoniasis
197. A patient with suspected dysentery has been admitted to the infectious diseases hospital. Which
basic method of laboratory diagnosis must be applied in the first place?
A. * Bacteriological
B. Serological
C. Allergic
D. Biological
E. Microscopic
198. A 12-year-old boy has been hospitalized for suspected food poisoning. The fecal samples were
inoculated on the Endo agar, which resulted in growth of a large number of colorless colonies.
What microorganism is most likely to be EXCLUDED from the list of possible causative agents
of the disease?
A. * Escherichia coli
B. Salmonella enteritidis
C. Proteus vulgaris
D. Pseudomonas aeruginosa
E. Yersinia enterocolitica
199. A child with suspected tuberculosis was given Mantoux test. After 24 hours the site of the
allergen injection got swollen, hyperemic and painful. What are the main components that
determine such response of the body?
D. B-lymphocytes, IgM
200. A 4-year-old child presents with general weakness, sore throat and deglutitive problem. After his
examination a doctor suspected diphtheria and sent the material to the bacteriological laboratory.
In order to determine the diphtheria causative agent the material should be inoculated into the
following differential diagnostic medium:
B. Endo's agar
C. Ploskyrev's agar
D. Sabouraud's agar
E. Levenshtein-Yessen agar
201. Planned mass vaccination of all newborn 5-7 day old children against tuberulosis plays an
important role in tuberculosis prevention. In this case the following vaccine is applied:
A. * BCG
E. Tuberculin
202. Inoculum from pharynx of a patient ill with angina was inoculated into blood-tellurite agar. It
resulted in growth of grey, radially striated (in form of rosettes) colonies 4-5 mm in diameter.
Gram-positive bacilli with clublike thickenings on their ends placed in form of spread wide apart
fingers are visible by microscope. What microorganisms are these?
A. * Diphtheria corynebacteria
B. Botulism clostridia
C. Diphtheroids
D. Streptococci
E. Streptobacilli
203. After the sanitary and bacteriological study of tap water the following results were obtained: the
total number of bacteria in 1,0 ml was 80, coli index was 3. How would you interpret the study
results?
D. Water is contaminated
204. During examination of a 3-monthold infant a pediatrician revealed that the baby's oral
mucosa and tongue were covered with a thick white deposit. In the material taken from
the affected site a bacteriologist revealed the presence of yeast fungi giving the reasons
for suspecting a fungal infection which occurs most often in children of this age, namely:
A. * Candidiasis
B. Favus
C. Epidermophytosis
D. Actinomycosis
E. Trichophytia
205. It is known that infectious type B hepatitis is a systemic disease caused by the type B hepatitis
virus and characterized by a predominant liver affection. Choose from the below given list the
drugs for the etiotropic therapy of this infection:
A. * Acyclovir
B. Penicillin
C. Tetracycline
D. Sulfanilamides
E. Fluoroquinolones
206. Analysis of sputum obtained from a patient with suspected pneumonia revealed gram-positive
diplococci. They were slightly elongated, with the pointed opposite ends. What microorganisms
were revealed in the sputum?
A. * Streptococcus pneumoniae
B. Staphylococcus aureus
C. Klebsiella pneumoniae
D. Neisseria meningitidis
E. Streptococcus pyogene
207. Examination of air state in hospital was done by method of sedimentation. It revealed 5 small
roundish colonies with zone of hemolysis around them. Inoculations were made on the following
cultural medium:
A. * Blood agar
B. Endo agar
E. Levin agar
208. A female patient has been treated with antibiotics for a long time. Thereafter examination of
smears form vaginal secretion revealed oval cells with well-defined nucleus, some cells gemmate.
What preparations can help to confirm the diagnosis +ACI-candidosis+ACI-?
A. * Antifungal
B. Antibacterial
C. Antichlamydial
D. Antiviral
E. Antiprotozoal
209. A patient is suspected to have the typhoid fever. What method of laboratory diagnostics would be
the most appropriate for confirmation of this diagnosis in the first week of disease?
A. * Hemoculture identification
C. Myeloculture identification
D. Biliculture identification
E. Coproculture identification
210. Analysis of sputum obtained from a patient with suspected pneumonia revealed gram-positive
diplococci. They were slightly elongated, with the pointed opposite ends. What microorganisms
were revealed in the sputum?
A. * Streptococcus pneumoniae
B. Staphylococcus aureus
C. Klebsiella pneumoniae
D. Neisseria meningitidis
E. Streptococcus pyogenes
211. During bacteriological analysis of solutions prepared in a pharmacy some red colonies with
metallic glitter have grown on Endo agar. What microbes were revealed?
A. * Escherichia
B. Shigella
C. Staphylococci
D. Streptococci
E. Salmonella
212. Medical examination of a dairymaid revealed affection of the locomotive system, vision
impairment, disorder of the nervous and other systems. To confirm the diagnosis the patient was
referred for a serological assay (Wright+IBk-s reaction) and Burnet's skin allergy test. What was
the provisional diagnosis?
A. * Brucellosis
B. Tularemia
C. Anthrax
D. Rheumatism
E. Leptospirosis
213. A sick child with diphtheria 10 days after the injection of antitoxic diphtheria serum, there were
skin rash, accompanied by severe itching, increased body temperature to 38, there were pains in
the joints. What is the cause of these events you expect?
A. * Delayed-type hypersensitivity
B. Contact allergy
C. Anaphylactic reaction
D. Serum sickness
E. Atopy
214. Veterinarny assistant, working on cattle farm went to a doctor complaining of joint pain, fever,
malaise, night sweats. Ill for about a month. Given that the patient is working on a farm and
livestock related complaints, the doctor suspected he had brucellosis. What material is taken from
this patient to be investigated in the usual microbiological laboratory?
A. Sputum
B. * Blood serum
C. Vomit fluid
D. Urine
E. Feces
215. A patient, sick 3 days ago, complaining of the increased temperature of 38 +ALA- C, abdominal
pain, loose stools, presence of blood in the stool, the doctor diagnosed clinically bacterial
dysentery. What method of microbiological diagnosis is advisable to apply in this case and what
material should be taken from the patient to confirm the diagnosis?
A. * Bacteriologycal
B. Serological ,blood
C. Smear ,blood
D. Smear , feces
E. Bacteriology, urine
216. A patients with gastric ulcer with hyperacidity after endoscopy and bacteriological studies were
identified bacteria of the genus Helicobacter. Thanks to some property of these organisms are not
killed in the acidic environment of the stomach?
A. * Urease activity
C. Oxidase activity
D. Catalase activity
E. Resistance to vancomycin
217. A patient with pustular skin lesions isolated pathogen, which is on blood agar forms a rounded,
medium sized, yellow colonies surrounded by a zone of hemolysis. In smears made from the
colonies Gr +- cocci appear in clusters were found. Pure culture is oxidase-and catalase-positive,
ferments mannitol, synthesizes plasma coagulase. Which of the following type matches the
selected agent?
A. * Staphylococcus aureus
B. Staphylococcus saprophyticus
C. Streptococcus agalactiae
D. Staphylococcus epidermidis
E. Streptococcus pyogenes
218. A patient, 68 years old, was subjected to prolonged antibiotic therapy. After such treatment, the
patient appeared whitish coating on the oral mucosa. Microscopic examination of smear revealed
large, round, various sizes, gram positive microorganisms. What should be done to continue the
microbiological diagnosis?
219. A patient visited a dentist with complaints of redness and edema of his mouth mucous membrane
in a month after dental prosthesis. The patient was diagnosed with allergic stomatitis. What type
of allergic reaction by Gell and Cumbs underlies this disease?
A. Cytotoxic
C. Anaphylactic
D. Stimulating
E. Immunocomplex
220. Donor skin transplantation was performed to a patient with extensive burns. On the 8-th day the
graft became swollen and changed colour+ADs- on the 11-th day graft rejection started. What
cells take part in this process?
A. B-lymphocytes
B. Erythrocytes
C. * T-lymphocytes
D. Basophils
E. Eosinophils
221. A patient with suspected diphtheria went through bacterioscopic examination. Examination of
throat swab revealed rod-shaped bacteria with volutin granules. What etiotropic preparation
should be chosen in this case?
A. Interferon
B. Bacteriophage
C. Diphtheria toxoid
222. ?The most common cause of cystitis after Escherichia coli in young healthy sexually active
women is:
A. Proteus mirabilis.
B. Pseudomonas aeruginosa.
C. Klebsiella pneumoniae.
D. * Staphylococcus saprophyticus.
E. Neisseria gonorrheae
223. This bacterium, although usually not considered as normal flora, frequently colonizes the
stomach and is thought to be the cause of gastric and duodenal ulcers.
A. Lactobacillus acidophilus
B. Pseudomonas aeruginosa
C. Escherichia coli
D. Vibrio cholerae
E. * Helicobacter pylori
224. The most common cause of cystitis after Escherichia coli in young healthy sexually active
women is:
A. Proteus mirabilis.
B. Pseudomonas aeruginosa.
C. Klebsiella pneumoniae.
D. * Staphylococcus saprophyticus.
E. Neisseria gonorrheae
225. During surgical operation a blood transfusion was made. The blood must be checked to find
antigens of some disease. What disease is expected to be found?
A. * Virus of hepatitis B
B. Virus of hepatitis A
C. Adenovirus
D. Enterovirus
E. Virus of hepatitis E
226. A 42-year-old man who has been injured in a car accident is brought into the emergency room.
His blood alcohol level on admission is 250 mg/dL. Hospital records show a prior hospitalization
for alcohol related seizures. His wife confirms that he has been drinking heavily for 3 weeks.
What treatment should be provided to the patient if he goes into withdrawal?
A. * Diazepam
B. Phenobarbital
C. Pentobarbital
D. Phenytoin
E. None
227. A patient who came to the doctor because of his infertility was administered to make tests for
toxoplasmosis and chronic gonorrhoea. Which reaction should be performed to reveal
C. Immunoblot analysis
228. The person was selling homemade pork sausages on the market. State sanitary inspector
suspected falcification of the sausages.With help of what serological immune reaction can food
substance be identified?
A. * Precipitation test
C. Agglutination test
D. Immunofluorescence test
229. The donor who for a long time didn't donate the blood was investigated with IFA method. Anti-
HBs antibodies were revealed. What does positive result of IFA in this case mean?
A. * Previous hepatitis B
B. Acute hepatitis B
C. Acute hepatitis C
D. Chronic hepatitis В
E. Chronic hepatitis С
230. A man was admitted to the hospital on the 5th day of disease that manifested itself by jaundice,
muscle aching, chill, nose bleedings. In course of laboratory diagnostics a bacteriologist
performed dark-field microscopy of the patient's blood drop. Name a causative agent of this
disease:
A. * Leptospira interrogans
B. Borrelia dutlonii
C. Calymmatobacterium granulomatis
D. Bartonella bacilloformis
E. Rickettsia mooseri
231. Microscopic examination of a Gram-stained scrape from patient's tongue revealed oval, round,
elongated chains of dark-violet gemmating cells. What disease can be caused by this causative
agent?
A. * Candidosis
B. Actinomycosis
C. Streptococcic infection
D. Staphylococcic infection
E. Diphtheria
232. The first grade pupils were examined in order to sort out children for tuberculosis revaccination.
What test was applied for this purpose?
A. * Mantoux test
B. Schick test
D. Burnet test
E. Anthraxine test
233. This bacterium, although usually not considered as normal flora, frequently colonizes the
stomach and is thought to be the cause of gastric and duodenal ulcers.
A. Lactobacillus acidophilus
B. Pseudomonas aeruginosa
C. Escherichia coli
D. Vibrio cholerae
E. * Helicobacter pylori
234. Clinical diagnosis of a female patient was gonorrhoea. What examination method can be applied
for confirmation of this diagnosis?
D. Hemagglutination reaction
E. Immobilization reaction
235. A patient recovered from Sonne dysentery and was once more infected with the same causative
agent. What is such infection form called?
A. * Reinfection
B. Recidivation
C. Superinfection
D. Persisting infection
E. Chronic infection
236. During examination of a patient a dentist revealed a lot of "white spots" - zones of enamel
demineralization. What microorganisms take part in the development of this process?
A. * Streptococcus mutans
B. Streptococcus salivarius
C. Streptococcus pyogenes
D. Veilonella parvula
E. Staphylococcus epidermidis
237. During surgical operation a blood transfusion was made. The blood must be checked to find
antigens of some disease. What disease is expected to be found?
A. * Virus of hepatitis B
B. Virus of hepatitis A
C. Adenovirus
D. Enterovirus
E. Virus of hepatitis E
238. Planned mass vaccination of all newborn 5-7 day old children against tuberculosis plays an
important role in tuberculosis prevention. In this case the following vaccine is applied:
A. * BCG
E. TABTe
239. A 4-year-old child presents with general weakness, sore throat and deglutitive problem. After his
examination a doctor suspected diphtheria and sent the material to the bacteriological laboratory.
In order to determine the diphtheria causative agent the material should be inoculated into the
following differential diagnostic medium:
B. Endo agar
C. Ploskyrev agar
D. Sabouraud agar
E. Levenshtein-Yessen agar
240. Vomiting matters of a patient suspected of having cholera were delivered to the bacteriological
laboratory. The material was used for preparing a "hanging drop" specimen. What type of
microscopy will be applied for identification of the causative agent by its mobility?
A. * Phase-contrast microscopy
B. Electron microscopy
D. Fluorescence microscopy
E. Immersion microscopy
241. In the surgical ward, the dressing material was undergoing sterilization in an autoclave. Through
an oversight of a nurse the mode of sterilization was changed and the temperature in the
autoclave reached only 100?C instead of the due 120?C. What microorganisms can stay viable
under these conditions?
242. In course of long-term treatment of an infectious patient with penicillin, the pathogen transformed
into the L-form. What changes occur in the pathogen cell in case of L-transformation?
B. Absence of flagella
C. Absence of a capsule
D. Absence of a spore
E. Absence of inclusions
243. There are areas where humans or animals are exposed to the constant risk of contracting certain
types of bacteria. What feature of these bacteria is responsible for their long viability in the soil?
A. * Spore formation
B. Capsule formation
E. Plasmids
244. Analysis of the cerebrospinal fluid of a child with signs of purulent lesion of brain tunics revealed
gram-negative bean-shaped diplococci. What presumptive diagnosis can be made on the basis of
the analysis results?
A. * Meningitis
B. Gonorrhea
C. Cholera
D. Plague
E. Anthrax
245. The causative agent of botulism causes severe food poisoning. Specify the most characteristic
morphological feature of botulism causative agent:
A. * Kitt-Tarozzi medium
B. Endo agar
C. Levine agar
D. Meat-peptone agar
E. Milk-salt agar
247. A 42-year-old man who has been injured in a car accident is brought into the emergency room.
His blood alcohol level on admission is 250 mg/dL. Hospital records show a prior hospitalization
for alcohol related seizures. His wife confirms that he has been drinking heavily for 3 weeks.
What treatment should be provided to the patient if he goes into withdrawal?
A. * Diazepam
B. Phenobarbital
C. Pentobarbital
D. Phenytoin
E. None
248. For the specific prevention of influenza, the employees of an enterprise were vaccinated with
"Influvac". What type of immunity will develop in the body of the vaccinated?
A. * Artificial active
B. Innate congenital
C. Artificial passive
D. Natural active
E. Natural passive
249. Dysbiosis can be treated with drugs that contain living representatives of normal microflora as
well as their metabolic products. Select the microorganisms that are used for the production of
such drugs:
A. * Bifidum bacteria
B. Staphylococcus aureus
C. Proteus
D. Providencia
E. Yersinia
250. A 26-year-old female patient with bronchitis has been administered a broad spectrum antibiotic as
a causal treatment drug. Specify this drug:
A. * Doxycycline
B. Interferon
C. BCG vaccine
D. Ambroxol
E. Dexamethasone
251. 10 days after having quinsy caused by beta-hemolytic streptococcus a 6-year-old child exhibited
symptoms of glomerulonephritis. What mechanism of glomerular lesion is most likely in this
case?
A. * Immunocomplex
B. Cellular cytotoxicity
C. Anaphylaxis
D. Atopy
252. 6 hours after the initial inoculation of water sample into 1% peptone water, the growth of a
culture in form of a thin pellicle on the medium surface was registered. Such cultural properties
are typical for the causative agent of the following disease:
A. * Cholera
B. Plague
C. Tuberculosis
D. Dysentery
E. Pseudotuberculosis
253. Analysis of the cerebrospinal fluid of a child with signs of purulent lesion of brain tunics revealed
gram-negative bean-shaped diplococci. What presumptive diagnosis can be made on the basis of
the analysis results?
A. * Meningitis
B. Gonorrhea
C. Cholera
D. Plague
E. Anthrax
254. Microbiological purity of tabulated drugs had been tested at factory. Samples cultivation in
mannitol salt agar resulted in growth of golden-yellow colonies, microscopic examination of
colonies detected Gram-positive globular bacteria positioned in clusters; microorganisms had
plasma coagulation properties. What pure bacterial culture was obtained?
A. * Staphylococcus aureus
B. Escherichia coli
C. Staphylococcus saprophyticus
D. Pseudomonas aeruginosa
E. Staphylococcus epidermidis
255. Microbiological studies of air in the pharmacy room revealed the presence of pathogenic
staphylococci. Select the medium in which you can detect the lecithinase activity of the isolated
microorganism:
A. * Yolk-salt agar
B. Blood agar
D. Sugar agar
E. Meat-extract agar
256. There is a suspicion of active tuberculosis development in patient. The doctor has appointed
Mantoux test to make a diagnosis. What immunobiological agent has to be administered?
A. * Tuberculin
B. BCG vaccine
C. DPT vaccine
D. Tularin
E. DT vaccine
257. A young woman suddenly developed fever up to 39oC accompanied by a strong headache.
Examination revealed marked nuchal rigidity. Spinal puncture was performed. Gram-stained
smear of cerebrospinal fluid contained many neutrophils and Gram-negative diplococci. What
bacteria could be the cause of this disease?
A. * Neisseria meningitidis
B. Streptococcus pneumonia
C. Haemophilus influenzae
D. Staphylococcus aureus
E. Pseudomonas aeruginosa
258. A patient who came to the doctor because of his infertility was administered to make tests for
toxoplasmosis and chronic gonorrhoea. Which reaction should be performed to reveal
C. Immunoblot analysis
259. A bacteriological laboratory studied the home-made dried fish which had caused a severe food
poisoning. Microscopy of the culture grown on the Kitt-Tarozzi medium revealed
microorganisms resembling a tennis racket. What is the most likely diagnosis?
A. * Botulism
B. Salmonellosis
C. Cholera
D. Dysentery
E. Typhoid fever
260. A patient presents with fever, chill and cough. From his sputum the ovoid Gram-negative bipolar-
stained bacilli with a delicate capsule were isolated. What is the most likely diagnosis?
A. * Plague
B. Tuberculosis
C. Leptospirosis
D. Brucellosis
E. Toxoplasmosis
261. A patient has been hospitalised with provisional diagnosis of botulism. What serological reaction
should be used to reveal botulinum toxin?
A. * Neutralization reaction
B. Agglutination reaction
C. Bordet-Gengou test
D. Precipitation reaction
E. Immunofluorescence test
262. On bacteriological study of rinsing water of the patient with food poisoning, the pure bacterial
culture was inoculated with the following properties: gram-negative motile bacillus in the Endo
environment grows like achromic colony. Representative of what genus has caused the illness?
A. * Salmonella
B. Shigella
C. Yersinia
D. Escherichia
E. Citrobacter
263. In a village, a case of anthrax had been registered. Medical services began epidemiologically
indicated specific prophylaxis of population against anthrax. What preparation was used for this
purpose?
A. * Live vaccine
B. Inactivated vaccine
C. Chemical vaccine
E. Anatoxin
264. A 55-year-old patient with a characteristic rash, fever, dizziness has been admitted to a hospital.
He has been provisionally diagnosed with typhus. No similar cases have been reported. In his
youth (15 years old) the patient suffered typhus in a boarding school. What disease is it?
A. * Brill disease
B. Typhoid fever
C. Measles
D. Rubella
E. Cholera
265. A patient has severe catarrhal symptoms. Material growth on Bordet-Gengou agar showed
mercury-drop like colonies. Examination of the blood smears revealed some small ovoid gram
positive bacilli sized 1-3 microns. What microorganisms were isolated?
A. * Bordetella
B. Corynebacteria
C. Mycobacteria
D. Meningococcus
E. Brucella
266. The laboratory for especially dangerous infections conducts microscopic examination of
pathological material from a patient with suspected plague. The sample was stained by Burri-
Gins technique. What property of the causative agent can be identified by this technique?
A. * Capsule formation
B. Spore formation
C. Acid resistance
D. Alkali resistance
267. A child cut his leg with a piece of glass while playing and was brought to the clinic for the
injection of tetanus toxoid. In order to prevent the development of anaphylactic shock the serum
was administered by Bezredka method. What mechanism underlies this method of desensitization
of the body?
268. A 32-year-old patient undergoing dental examination was found to have some rash-like lesions
resembling secondary syphilis in the oral cavity. The patient was referred for the serological
study with the purpose of diagnosis confirmation. In order to detect antibodies in the serum,
living Treponema were used as diagnosticum. What serological test was performed?
A. * Immobilization
B. Neutralization
C. Complement binding
D. Precipitation
E. Passive hemagglutination
269. The person was selling homemade pork sausages on the market. State sanitary inspector
suspected falcification of the sausages.With help of what serological immune reaction can food
substance be identified?
A. * Precipitation test
C. Agglutination test
D. Immunofluorescence test
270. Examination of a 27-year-old donor who had not donated blood for a long time revealed HBs
antibodies detected by ELISA method. In this case, the positive ELISA results indicate that the
donor:
A. * Had hepatitis B
271. In the area being the epicenter of the registered rabies cases among wild animals a 43-year-old
man presented to a clinic and claimed to have been bitten by a stray dog. He was given a course
of anti-rabies vaccine. This preparation relates to the following type of vaccines:
A. * Attenuated
B. Inactivated
C. Molecular
D. Toxoids
E. Synthetic
272. ?A smear of sputum from the patient with suspected lobar pneumonia was stained with
the use of the following stains and reagents: solution of gentian violet (crystal violet),
Lugol solution, 96o alcohol, water zaphranin. What staining method was applied in this
case?
A. * Gram
B. Ziehl-Nielsen
C. Romanovsky
D. Neisser
E. Leffler
273. A patient has been hospitalised with provisional diagnosis of virus B hepatitis. Serological
reaction based on complementation of antigen with antibody chemically bound to peroxidise or
alkaline phosphatase has been used for disease diagnostics. What is the name of the applied
serological reaction?
A. * Immune-enzyme analysis
B. Radioimmunoassay technique
C. Immunofluorescence test
D. Bordet-Gengou test
E. Antigen-binding assay
274. The donor who for a long time didn't donate the blood was investigated with IFA method. Anti-
HBs antibodies were revealed. What does positive result of IFA in this case mean?
A. * Previous hepatitis B
B. Acute hepatitis B
C. Acute hepatitis C
275. A 3-year-old child has continuous fever, lymph nodes are enlarged, the amount of lymphocytes in
blood is significantly increased. Enzyme linked immunosorbent assay (ELISA) revealed antigen
of Epstein-Barr virus. What diagnosis can be made based on the information given above?
A. * Infectious mononucleosis
B. Burkitt’s lymphoma
C. Herpetic lymphadenopathy
E. Cytomegalovirus infection
276. Hepatitis B is diagnosed through laboratory tests that determine the presence of HBV-DNA in
blood serum of the patient. What reference method is applied for this purpose?
B. Hybridization method
277. An outbreak of an intestinal infection occurred in a kindergarten on the eve of New Year
holidays. Bacteriological examination of patients’ faeces didn’t reveal any pathogenic bacteria.
Electronmicroscopy revealed roundish structures with clear outer edges and a thick core
resembling a wheel. Specify the most likely causative agent of this infection:
A. * Rotavirus
B. Adenovirus
C. Coxsacki-virus
D. E. coli
E. P. vulgaris
278. A patient has been admitted to the infectious diseases department for malaise, fever up to 38oC,
jaundice. A few months ago, the patient underwent blood transfusion. The doctor suspected viral
hepatitis B. What are the principal methods of laboratory diagnosis of hepatitis B?
B. Virus isolation in cell culture and its identification by the cytopathic effects
279. In our country, routine preventive vaccinations against poliomyelitis ivolve using live vaccine
that is administered orally. What immunoglobulins are responsible for the development of local
post-vaccination immunity in this case?
A. * Secretory IgA
B. IgM
C. IgG
D. Serum IgA
E. IgE
280. A laboratory received a sample of water used in drug production for sanitary and virological
analysis. What group of viruses will indicate faecal contamination of water and thus the need for
its additional purification?
A. * Picornaviridae
B. Herpesviridae
C. Orthomyxoviridae
D. Retroviridae
E. Flaviviridae
281. A sample of water used in drug production has been sent to a laboratory for sanitary and
virological analysis. Presence of what virus group will be indicative of faecal contamination of
water and thus the need for its additional purification?
A. * Picornaviridae
B. Herpesviridae
C. Orthomyxoviridae
D. Retroviridae
E. Flaviviridae
282. Virological laboratory has received patient’s nasopharyngeal lavage. What can be used to single
out influenza virus from the patient’s lavage?
A. * Chick embryo
D. Endo’s medium
E. Lowenstein–Jensen medium
283. A person has been in contact with influenza patient. What drug should be administered for
specific passive influenza prophylaxis?
A. Amizon
C. Leukocytic interferon
D. * Immunoglobulin
E. Anaferon
284. A child entering the school for the first time was given Mantoux test in order to determine if there
was a need for revaccination. The reaction was negative. What is the meaning of this test result?
285. A man was admitted to the hospital on the 5th day of disease that manifested itself by jaundice,
muscle aching, chill, nose bleedings. In course of laboratory diagnostics a bacteriologist
performed dark-field microscopy of the patient's blood drop. Name a causative agent of this
disease:
A. * Leptospira interrogans
B. Borrelia dutlonii
C. Calymmatobacterium granulomatis
D. Bartonella bacilloformis
E. Rickettsia mooseri
286. Bacteriological examination of purulent discharges from the urethra revealed some gram-negative
bean-shaped bacteria located in the leukocytes. They can be identified as the causative agent of
the following disease:
A. * Gonorrhea
B. Syphilis
C. Venereal lymphogranulomatosis
D. Chancroid
E. Trichomoniasis
287. A 12-year-old boy has been hospitalized for suspected food poisoning. The fecal samples were
inoculated on the Endo agar, which resulted in growth of a large number of colorless colonies.
What microorganism is most likely to be EXCLUDED from the list of possible causative agents
of the disease?
A. * Escherichia coli
B. Salmonella enteritidis
C. Proteus vulgaris
D. Pseudomonas aeruginosa
E. Yersinia enterocolitica
288. A child with suspected tuberculosis was given Mantoux test. After 24 hours the site of the
allergen injection got swollen, hyperemic and painful. What are the main components that
determine such response of the body?
D. B-lymphocytes, IgM
289. A 4-year-old child presents with general weakness, sore throat and deglutitive problem. After his
examination a doctor suspected diphtheria and sent the material to the bacteriological laboratory.
In order to determine the diphtheria causative agent the material should be inoculated into the
following differential diagnostic medium:
B. Endo's agar
C. Ploskyrev's agar
D. Sabouraud's agar
E. Levenshtein-Yessen agar
290. Planned mass vaccination of all newborn 5-7 day old children against tuberulosis plays an
important role in tuberculosis prevention. In this case the following vaccine is applied:
A. * BCG
E. Tuberculin
291. Microscopic examination of a Gram-stained scrape from patient's tongue revealed oval, round,
elongated chains of dark-violet gemmating cells. What disease can be caused by this causative
agent?
A. * Candidosis
B. Actinomycosis
C. Streptococcic infection
D. Staphylococcic infection
E. Diphtheria
292. Inoculum from pharynx of a patient ill with angina was inoculated into blood-tellurite agar. It
resulted in growth of grey, radially striated (in form of rosettes) colonies 4-5 mm in diameter.
Gram-positive bacilli with clublike thickenings on their ends placed in form of spread wide apart
fingers are visible by microscope. What microorganisms are these?
A. * Diphtheria corynebacteria
B. Botulism clostridia
C. Diphtheroids
D. Streptococci
E. Streptobacilli
293. After the sanitary and bacteriological study of tap water the following results were obtained: the
total number of bacteria in 1,0 ml was 80, coli index was 3. How would you interpret the study
results?
D. Water is contaminated
294. During examination of a 3-monthold infant a pediatrician revealed that the baby’s oral mucosa
and tongue were covered with a thick white deposit. In the material taken from the affected site a
bacteriologist revealed the presence of yeast fungi giving the reasons for suspecting a fungal
infection which occurs most often in children of this age, namely:
A. * Candidiasis
B. Favus
C. Epidermophytosis
D. Actinomycosis
E. Trichophytia
295. It is known that infectious type B hepatitis is a systemic disease caused by the type B hepatitis
virus and characterized by a predominant liver affection. Choose from the below given list the
drugs for the etiotropic therapy of this infection:
A. * Acyclovir
B. Penicillin
C. Tetracycline
D. Sulfanilamides
E. Fluoroquinolones
296. Analysis of sputum obtained from a patient with suspected pneumonia revealed gram-positive
diplococci. They were slightly elongated, with the pointed opposite ends. What microorganisms
were revealed in the sputum?
A. * Streptococcus pneumoniae
B. Staphylococcus aureus
C. Klebsiella pneumoniae
D. Streptococcus pyogene
E. Neisseria meningitidis
297. The first grade pupils were examined in order to sort out children for tuberculosis revaccination.
What test was applied for this purpose?
A. * Mantoux test
B. Schick test
D. Burnet test
E. Anthraxine test
298. Examination of air state in hospital was done by method of sedimentation. It revealed 5 small
roundish colonies with zone of hemolysis around them. Inoculations were made on the following
cultural medium:
A. * Blood agar
B. Endo agar
E. Levin agar
299. A female patient has been treated with antibiotics for a long time. Thereafter examination of
smears form vaginal secretion revealed oval cells with well-defined nucleus, some cells gemmate.
What preparations can help to confirm the diagnosis "candidosis"?
A. * Antifungal
B. Antibacterial
C. Antichlamydial
D. Antiviral
E. Antiprotozoal
300. A patient is suspected to have the typhoid fever. What method of laboratory diagnostics would be
the most appropriate for confirmation of this diagnosis in the first week of disease?
A. * Hemoculture identification
C. Myeloculture identification
D. Biliculture identification
E. Coproculture identification
301. Analysis of sputum obtained from a patient with suspected pneumonia revealed gram-positive
diplococci. They were slightly elongated, with the pointed opposite ends. What microorganisms
were revealed in the sputum?
A. * Streptococcus pneumoniae
B. Staphylococcus aureus
C. Klebsiella pneumoniae
D. Neisseria meningitidis
E. Streptococcus pyogenes
302. During bacteriological analysis of solutions prepared in a pharmacy some red colonies with
metallic glitter have grown on Endo agar. What microbes were revealed?
A. * Escherichia
B. Shigella
C. Staphylococci
D. Streptococci
E. Salmonella
303. Medical examination of a dairymaid revealed affection of the locomotive system, vision
impairment, disorder of the nervous and other systems. To confirm the diagnosis the
patient was referred for a serological assay (Wright’s reaction) and Burnet skin allergy test.
What was the provisional diagnosis?
A. * Brucellosis
B. Tularemia
C. Anthrax
D. Rheumatism
E. Leptospirosis
304. Clinical diagnosis of a female patient was gonorrhoea. What examination method can be applied
for confirmation of this diagnosis?
D. Hemagglutination reaction
E. Immobilization reaction
305. A sick child with diphtheria 10 days after the injection of antitoxic diphtheria serum, there were
skin rash, accompanied by severe itching, increased body temperature to 38, there were pains in
the joints. What is the cause of these events you expect?
A. Delayed-type hypersensitivity
B. Contact allergy
C. Anaphylactic reaction
D. * Serum sickness
E. Atopy
306. Veterinarny assistant, working on cattle farm went to a doctor complaining of joint pain, fever,
malaise, night sweats. Ill for about a month. Given that the patient is working on a farm and
livestock related complaints, the doctor suspected he had brucellosis. What material is taken from
this patient to be investigated in the usual microbiological laboratory?
A. Sputum
B. * Blood serum
C. Vomit fluid
D. Urine
E. Feces
307. A patient, sick 3 days ago, complaining of the increased temperature of 38 ° C, abdominal pain,
loose stools, presence of blood in the stool, the doctor diagnosed clinically bacterial dysentery.
What method of microbiological diagnosis is advisable to apply in this case and what material
should be taken from the patient to confirm the diagnosis?
A. * Bacteriologycal
B. Serological ,blood
C. Smear ,blood
D. Smear , feces
E. Bacteriology, urine
308. A patients with gastric ulcer with hyperacidity after endoscopy and bacteriological studies were
identified bacteria of the genus Helicobacter. Thanks to some property of these organisms are not
killed in the acidic environment of the stomach?
A. * Urease activity
C. Oxidase activity
D. Catalase activity
E. Resistance to vancomycin
309. A patient recovered from Sonne dysentery and was once more infected with the same causative
agent. What is such infection form called?
A. * Reinfection
B. Recidivation
C. Superinfection
D. Persisting infection
E. Chronic infection
310. A patient with pustular skin lesions isolated pathogen, which is on blood agar forms a rounded,
medium sized, yellow colonies surrounded by a zone of hemolysis. In smears made from the
colonies Gr + cocci appear in clusters were found. Pure culture is oxidase-and catalase-positive,
ferments mannitol, synthesizes plasma coagulase. Which of the following type matches the
selected agent?
A. * Staphylococcus aureus
B. Staphylococcus saprophyticus
C. Streptococcus agalactiae
D. Staphylococcus epidermidis
E. Streptococcus pyogenes
311. A child entering the school for the first time was given Mantoux test in order to determine if there
was a need for revaccination. The reaction was negative. What is the meaning of this test result?
312. A patient, 68 years old, was subjected to prolonged antibiotic therapy. After such treatment, the
patient appeared whitish coating on the oral mucosa. Microscopic examination of smear revealed
large, round, various sizes, gram positive microorganisms. What should be done to continue the
microbiological diagnosis?
313. A patient visited a dentist with complaints of redness and edema of his mouth mucous membrane
in a month after dental prosthesis. The patient was diagnosed with allergic stomatitis. What type
of allergic reaction by Gell and Cumbs underlies this disease?
A. * Cytotoxic
C. Anaphylactic
D. Stimulating
E. Immunocomplex
314. Donor skin transplantation was performed to a patient with extensive burns. On the 8-th day the
graft became swollen and changed colour; on the 11-th day graft rejection started. What cells take
part in this process?
A. * B-lymphocytes
B. Erythrocytes
C. T-lymphocytes
D. Basophils
E. Eosinophils
315. A patient with suspected diphtheria went through bacterioscopic examination. Examination of
throat swab revealed rod-shaped bacteria with volutin granules. What etiotropic preparation
should be chosen in this case?
A.
Interferon
B. Bacteriophage
C. Diphtheria antitoxin
E. Eubiotic
316. During examination of a patient a dentist revealed a lot of +ACI-white spots+ACI- - zones of
enamel demineralization. What microorganisms take part in the development of this process?
A. * Streptococcus mutans
B. Streptococcus salivarius
C. Streptococcus pyogenes
D. Veilonella parvula
E. Staphylococcus epidermidis
317. While studying a microslide obtained from the punctuate of a regional lymph node and stained by
Romanovsky-Giemsa method a physician revealed some light-pink thin microorganisms with 12-
14 regular spiral coils and pointed ends, up to 10-13 micrometer long. This might be the causative
agent of the following disease:
A. * Syphilis
B. Trypanosomiasis
C. Leptospirosis
D. Relapsing fever
E. Leishmaniasis
318. A man who was bitten by the unknown dog applied to the surgeon. Wide ragged woundes were
localised on the face. What curative-prophylactic aid should be given to prevent rabies?
319. A consumptive patient has an open pulmonary form of disease. Choose what sputum staining
should be selected for finding out the tubercle (Koch's) bacillus?
A. * Method of Ziel-Neelsen
B. Method of Romanowsky-Giemsa
C. Method of Gram
D. Method of Neisser
E. Method of Burry-Gins
320. A 42-year-old man who has been injured in a car accident is brought into the emergency room.
His blood alcohol level on admission is 250 mg/dL. Hospital records show a prior hospitalization
for alcohol related seizures. His wife confirms that he has been drinking heavily for 3 weeks.
What treatment should be provided to the patient if he goes into withdrawal?
A. * Diazepam
B. Phenobarbital
C. Pentobarbital
D. Phenytoin
E. None
321. In a 2-year-old child with catarrhal presentations and skin rash a pediatrician suspected scarlet
fever. The child was given intracutaneously a small dose of serum antibody to the streptococcal
erythrogenic toxin; on the site of injection the rash disappeared. What do the reaction results
mean?
322. Planned mass vaccination of all newborn 5-7 day old children against tuberculosis plays an
important role in tuberculosis prevention. In this case the following vaccine is applied:
A. * BCG
E. TABTe
323. From the defecation of a 6-year-old ill child, who has artificial feeding, the intestinal bacillus
with antigen structure 0-111 is excreted. What is the diagnosis?
A. * Coli-enteritis
B. Gastroenteritis
C. Cholera-like diseasis
D. Food poisoning
E. Dysentery-like diseasis
324. For serological diagnostics of the whooping cough it was made large-scale reaction with
parapertussis and pertussis diagnosticums. At the bottom of the test-tubes with diagnosticum of
Bordetella parapertussis grain-like sediment formed. What antibodies have this reaction
revealed?
A. * Agglutinins
B. Precipitins
C. Opsonins
D. Bacteriolysins
E. Antitoxins
325. From pharynx of a child with suspected diphtheria a pure culture of microorganisms was isolated.
Their morphological, tinctorial, cultural and biochemical properties appeared to be typical for
diphtheria causative agents. What study should be conducted in order to drow a conclusion that
this is a pathogenic diphtheria bacillus?
326. On bacteriological examination of the defecation of a 4-months-old baby with the symptoms of
acute bowel infection there were revealed red colonies spread in the large quantity in the Endo
medium. What microorganism can it be?
A. * Escherichia
B. Salmonella
C. Staphylococcus
D. Streptococcus
E. Shigella
327. Reaction of passive hemagglutination conducted with erythrocytic typhoid Vi-diagnosticum
helped to reveal some antibodies in the dilution of the patient's serum at a ratio of 1:80 that
exceeds the diagnostic titer. Such result witnesses of:
328. A 4-year-old child presents with general weakness, sore throat and deglutitive problem. After his
examination a doctor suspected diphtheria and sent the material to the bacteriological laboratory.
In order to determine the diphtheria causative agent the material should be inoculated into the
following differential diagnostic medium:
B. Endo agar
C. Ploskyrev agar
D. Sabouraud agar
E. Levenshtein-Yessen agar
329. Bacteriological examination of purulent discharges from the urethra revealed some gram-negative
bean-shaped bacteria located in the leukocytes. They can be identified as the causative agent of
the following disease:
A. * Gonorrhea
B. Syphilis
C. Venereal lymphogranulomatosis
D. Chancroid
E. Trichomoniasis
330. Vomiting matters of a patient suspected of having cholera were delivered to the
bacteriological laboratory. The material was used for preparing hanging drop specimen.
What type of microscopy will be applied for identification of the causative agent by its
mobility?
A. * Phase-contrast microscopy
B. Electron microscopy
C. Immune and electron microscopy
D. Fluorescence microscopy
E. Immersion microscopy
331. In the surgical ward, the dressing material was undergoing sterilization in an autoclave.
Through an oversight of a nurse the mode of sterilization was changed and the
temperature in the autoclave reached only 100 0C instead of the due 120oC. What
microorganisms can stay viable under these conditions?
332. In course of long-term treatment of an infectious patient with penicillin, the pathogen transformed
into the L-form. What changes occur in the pathogen cell in case of L-transformation?
B. Absence of flagella
C. Absence of a capsule
D. Absence of a spore
E. Absence of inclusions
333. A patient with suspected dysentery has been admitted to the infectious diseases hospital. Which
basic method of laboratory diagnosis must be applied in the first place?
A. * Bacteriological
B. Serological
C. Allergic
D. Biological
E. Microscopic
334. There are areas where humans or animals are exposed to the constant risk of contracting certain
types of bacteria. What feature of these bacteria is responsible for their long viability in the soil?
A. * Spore formation
B. Capsule formation
E. Plasmids
335. Analysis of the cerebrospinal fluid of a child with signs of purulent lesion of brain tunics revealed
gram-negative bean-shaped diplococci. What presumptive diagnosis can be made on the basis of
the analysis results?
A. * Meningitis
B. Gonorrhea
C. Cholera
D. Plague
E. Anthrax
336. The causative agent of botulism causes severe food poisoning. Specify the most characteristic
morphological feature of botulism causative agent:
337. A patient has a necrotizing phlegmon of his lower extremity. A doctor suspects a gas gangrene.
Microscopy reveals Gram-positive bacilli. In order to confirm the diagnosis further
bacteriological tests should include inoculation of the material into the following nutrient
medium:
A. * Kitt-Tarozzi medium
B. Endo agar
C. Levine agar
D. Meat-peptone agar
E. Milk-salt agar
338. For the specific prevention of influenza, the employees of an enterprise were vaccinated
with Influvac. What type of immunity will develop in the body of the vaccinated?
A. * Artificial active
B. Innate congenital
C. Artificial passive
D. Natural active
E. Natural passive
339. Dysbiosis can be treated with drugs that contain living representatives of normal microflora as
well as their metabolic products. Select the microorganisms that are used for the production of
such drugs:
A. * Bifidum bacteria
B. Staphylococcus aureus
C. Proteus
D. Providencia
E. Yersinia
340. A 26-year-old female patient with bronchitis has been administered a broad spectrum antibiotic as
a causal treatment drug. Specify this drug:
A. * Doxycycline
B. Interferon
C. BCG vaccine
D. Ambroxol
E. Dexamethasone
341. 10 days after having quinsy caused by beta-hemolytic streptococcus a 6-year-old child exhibited
symptoms of glomerulonephritis. What mechanism of glomerular lesion is most likely in this
case?
A. * Immunocomplex
B. Cellular cytotoxicity
C. Anaphylaxis
D. Atopy
342. A 12-year-old boy has been hospitalized for suspected food poisoning. The fecal samples were
inoculated on the Endo agar, which resulted in growth of a large number of colorless colonies.
What microorganism is most likely to be EXCLUDED from the list of possible causative agents
of the disease?
A. * Escherichia coli
B. Salmonella enteritidis
C. Proteus vulgaris
D. Pseudomonas aeruginosa
E. Yersinia enterocolitica
343. 6 hours after the initial inoculation of water sample into peptone water, the growth of a
culture in form of a thin pellicle on the medium surface was registered. Such cultural
properties are typical for the causative agent of the following disease:
A. * Cholera
B. Plague
C. Tuberculosis
D. Dysentery
E. Pseudotuberculosis
344. Analysis of the cerebrospinal fluid of a child with signs of purulent lesion of brain tunics revealed
gram-negative bean-shaped diplococci. What presumptive diagnosis can be made on the basis of
the analysis results?
A. * Meningitis
B. Gonorrhea
C. Cholera
D. Plague
E. Anthrax
345. Microbiological purity of tabulated drugs had been tested at factory. Samples cultivation in
mannitol salt agar resulted in growth of golden-yellow colonies, microscopic examination of
colonies detected Gram-positive globular bacteria positioned in clusters+ADs- microorganisms
had plasma coagulation properties. What pure bacterial culture was obtained?
A. * Staphylococcus aureus
B. Escherichia coli
C. Staphylococcus saprophyticus
D. Pseudomonas aeruginosa
E. Staphylococcus epidermidis
346. Microbiological studies of air in the pharmacy room revealed the presence of pathogenic
staphylococci. Select the medium in which you can detect the lecithinase activity of the isolated
microorganism:
A. * Yolk-salt agar
B. Blood agar
D. Sugar agar
E. Meat-extract agar
347. There is a suspicion of active tuberculosis development in patient. The doctor has appointed
Mantoux test to make a diagnosis. What immunobiological agent has to be administered?
A. * Tuberculin
B. BCG vaccine
C. DPT vaccine
D. Tularin
E. DT vaccine
348. A young woman suddenly developed fever up to 39oC accompanied by a strong headache.
Examination revealed marked nuchal rigidity. Spinal puncture was performed. Gram-stained
smear of cerebrospinal fluid contained many neutrophils and Gram-negative diplococci. What
bacteria could be the cause of this disease?
A. * Neisseria meningitidis
B. Streptococcus pneumonia
C. Haemophilus influenzae
D. Staphylococcus aureus
E. Pseudomonas aeruginosa
349. A bacteriological laboratory studied the home-made dried fish which had caused a severe food
poisoning. Microscopy of the culture grown on the Kitt-Tarozzi medium revealed
microorganisms resembling a tennis racket. What is the most likely diagnosis?
A. * Botulism
B. Salmonellosis
C. Cholera
D. Dysentery
E. Typhoid fever
350. A patient presents with fever, chill and cough. From his sputum the ovoid Gram-negative bipolar-
stained bacilli with a delicate capsule were isolated. What is the most likely diagnosis?
A. * Plague
B. Tuberculosis
C. Leptospirosis
D. Brucellosis
E. Toxoplasmosis
351. A patient has been hospitalised with provisional diagnosis of botulism. What serological reaction
should be used to reveal botulinum toxin?
A. * Neutralization reaction
B. Agglutination reaction
C. Bordet-Gengou test
D. Precipitation reaction
E. Immunofluorescence test
352. A child with suspected tuberculosis was given Mantoux test. After 24 hours the site of the
allergen injection got swollen, hyperemic and painful. What are the main components that
determine such response of the body?
D. B-lymphocytes, IgM
353. In a village, a case of anthrax had been registered. Medical services began epidemiologically
indicated specific prophylaxis of population against anthrax. What preparation was used for this
purpose?
A. * Live vaccine
B. Inactivated vaccine
C. Chemical vaccine
E. Anatoxin
354. A 55-year-old patient with a characteristic rash, fever, dizziness has been admitted to a hospital.
He has been provisionally diagnosed with typhus. No similar cases have been reported. In his
youth (15 years old) the patient suffered typhus in a boarding school. What disease is it?
A. * Brill disease
B. Typhoid fever
C. Measles
D. Rubella
E. Cholera
355. A patient has severe catarrhal symptoms. Material growth on Bordet-Gengou agar showed
mercury-drop like colonies. Examination of the blood smears revealed some small ovoid gram
positive bacilli sized 1-3 microns. What microorganisms were isolated?
A. * Bordetella
B. Corynebacteria
C. Mycobacteria
D. Meningococcus
E. Brucella
356. A 4-year-old child presents with general weakness, sore throat and deglutitive problem. After his
examination a doctor suspected diphtheria and sent the material to the bacteriological laboratory.
In order to determine the diphtheria causative agent the material should be inoculated into the
following differential diagnostic medium:
B. Endo's agar
C. Ploskyrev's agar
D. Sabouraud's agar
E. Levenshtein-Yessen agar
357. The laboratory for especially dangerous infections conducts microscopic examination of
pathological material from a patient with suspected plague. The sample was stained by Burri-
Gins technique. What property of the causative agent can be identified by this technique?
A. * Capsule formation
B. Spore formation
C. Acid resistance
D. Alkali resistance
359. A 32-year-old patient undergoing dental examination was found to have some rash-like lesions
resembling secondary syphilis in the oral cavity. The patient was referred for the serological
study with the purpose of diagnosis confirmation. In order to detect antibodies in the serum,
living Treponema were used as diagnosticum. What serological test was performed?
A. * Immobilization
B. Neutralization
C. Complement binding
D. Precipitation
E. Passive hemagglutination
360. Examination of a 27-year-old donor who had not donated blood for a long time revealed HBs
antibodies detected by ELISA method. In this case, the positive ELISA results indicate that the
donor:
A. * Had hepatitis B
361. In the area being the epicenter of the registered rabies cases among wild animals a 43-year-old
man presented to a clinic and claimed to have been bitten by a stray dog. He was given a course
of anti-rabies vaccine. This preparation relates to the following type of vaccines:
A. * Attenuated
B. Inactivated
C. Molecular
D. Toxoids
E. Synthetic
362. A patient has been hospitalised with provisional diagnosis of virus B hepatitis. Serological
reaction based on complementation of antigen with antibody chemically bound to peroxidise or
alkaline phosphatase has been used for disease diagnostics. What is the name of the applied
serological reaction?
A. * Immune-enzyme analysis
B. Radioimmunoassay technique
C. Immunofluorescence test
D. Bordet-Gengou test
E. Antigen-binding assay
363. Planned mass vaccination of all newborn 5-7 day old children against tuberulosis plays an
important role in tuberculosis prevention. In this case the following vaccine is applied:
A. * BCG
E. Tuberculin
364. A 3-year-old child has continuous fever, lymph nodes are enlarged, the amount of lymphocytes in
blood is significantly increased. Enzyme linked immunosorbent assay (ELISA) revealed antigen
of Epstein-Barr virus. What diagnosis can be made based on the information given above?
A. * Infectious mononucleosis
B. Burkitt+IBk-s lymphoma
C. Herpetic lymphadenopathy
E. Cytomegalovirus infection
365. Hepatitis B is diagnosed through laboratory tests that determine the presence of HBV-DNA in
blood serum of the patient. What reference method is applied for this purpose?
B. Hybridization method
366. An outbreak of an intestinal infection occurred in a kindergarten on the eve of New Year
holidays. Bacteriological examination of patients- faeces didn-t reveal any pathogenic
bacteria. Electronmicroscopy revealed roundish structures with clear outer edges and a
thick core resembling a wheel. Specify the most likely causative agent of this infection:
A. * Rotavirus
B. Adenovirus
C. Coxsacki-virus
D. E. coli
E. P. vulgaris
367. A hospitalized patient bitten by a rabid animal has an avulsive wound of shin. What kind of
vaccine must be given to prevent rabies?
A. * Anti-rabies vaccine
B. DTaP
C. Td
D. BCG
E. TABte
368. A patient has been admitted to the infectious diseases department for malaise, fever up to 38oC,
jaundice. A few months ago, the patient underwent blood transfusion. The doctor suspected viral
hepatitis B. What are the principal methods of laboratory diagnosis of hepatitis B?
B. Virus isolation in cell culture and its identification by the cytopathic effects
369. In our country, routine preventive vaccinations against poliomyelitis ivolve using live vaccine
that is administered orally. What immunoglobulins are responsible for the development of local
post-vaccination immunity in this case?
A. * Secretory IgA
B. IgM
C. IgG
D. Serum IgA
E. IgE
370. A laboratory received a sample of water used in drug production for sanitary and virological
analysis. What group of viruses will indicate faecal contamination of water and thus the need for
its additional purification?
A. * Picornaviridae
B. Herpesviridae
C. Orthomyxoviridae
D. Retroviridae
E. Flaviviridae
371. A sample of water used in drug production has been sent to a laboratory for sanitary and
virological analysis. Presence of what virus group will be indicative of faecal contamination of
water and thus the need for its additional purification?
A. * Picornaviridae
B. Herpesviridae
C. Orthomyxoviridae
D. Retroviridae
E. Flaviviridae
372. Virological laboratory has received patient+IBk-s nasopharyngeal lavage. What can be used to
single out influenza virus from the patient+IBk-s lavage?
A. * Chick embryo
D. Endo+IBk-s medium
E. Lowenstein+IBM-Jensen medium
373. A person has been in contact with influenza patient. What drug should be administered for
specific passive influenza prophylaxis?
A. * Amizon
B. Vaccine influenza virus inactivated
C. Leukocytic interferon
D. Immunoglobulin
E. Anaferon
374. Inoculum from pharynx of a patient ill with angina was inoculated into blood-tellurite agar. It
resulted in growth of grey, radially striated (in form of rosettes) colonies 4-5 mm in diameter.
Gram-positive bacilli with clublike thickenings on their ends placed in form of spread wide apart
fingers are visible by microscope. What microorganisms are these?
A. * Diphtheria corynebacteria
B. Botulism clostridia
C. Diphtheroids
D. Streptococci
E. Streptobacilli
375. A child entering the school for the first time was given Mantoux test in order to determine if there
was a need for revaccination. The reaction was negative. What is the meaning of this test result?
376. Bacteriological examination of purulent discharges from the urethra revealed some gram-negative
bean-shaped bacteria located in the leukocytes. They can be identified as the causative agent of
the following disease:
A. * Gonorrhea
B. Syphilis
C. Venereal lymphogranulomatosis
D. Chancroid
E. Trichomoniasis
377. A patient with suspected dysentery has been admitted to the infectious diseases hospital. Which
basic method of laboratory diagnosis must be applied in the first place?
A. * Bacteriological
B. Serological
C. Allergic
D. Biological
E. Microscopic
378. Analysis of sputum obtained from a patient with suspected pneumonia revealed gram-positive
diplococci. They were slightly elongated, with the pointed opposite ends. What microorganisms
were revealed in the sputum?
A. * Streptococcus pneumoniae
B. Staphylococcus aureus
C. Klebsiella pneumoniae
D. Neisseria meningitidis
E. Streptococcus pyogene
379. A 12-year-old boy has been hospitalized for suspected food poisoning. The fecal samples were
inoculated on the Endo agar, which resulted in growth of a large number of colorless colonies.
What microorganism is most likely to be EXCLUDED from the list of possible causative agents
of the disease?
A. * Escherichia coli
B. Salmonella enteritidis
C. Proteus vulgaris
D. Pseudomonas aeruginosa
E. Yersinia enterocolitica
380. A child with suspected tuberculosis was given Mantoux test. After 24 hours the site of the
allergen injection got swollen, hyperemic and painful. What are the main components that
determine such response of the body?
D. B-lymphocytes, IgM
381. A 4-year-old child presents with general weakness, sore throat and deglutitive problem. After his
examination a doctor suspected diphtheria and sent the material to the bacteriological laboratory.
In order to determine the diphtheria causative agent the material should be inoculated into the
following differential diagnostic medium:
B. Endo's agar
C. Ploskyrev's agar
D. Sabouraud's agar
E. Levenshtein-Yessen agar
382. Planned mass vaccination of all newborn 5-7 day old children against tuberulosis plays an
important role in tuberculosis prevention. In this case the following vaccine is applied:
A. * BCG
E. Tuberculin
383. Inoculum from pharynx of a patient ill with angina was inoculated into blood-tellurite agar. It
resulted in growth of grey, radially striated (in form of rosettes) colonies 4-5 mm in diameter.
Gram-positive bacilli with clublike thickenings on their ends placed in form of spread wide apart
fingers are visible by microscope. What microorganisms are these?
A. * Diphtheria corynebacteria
B. Botulism clostridia
C. Diphtheroids
D. Streptococci
E. Streptobacilli
384. After the sanitary and bacteriological study of tap water the following results were obtained: the
total number of bacteria in 1,0 ml was 80, coli index was 3. How would you interpret the study
results?
D. Water is contaminated
E. Water is highly contaminated
385. During examination of a 3-monthold infant a pediatrician revealed that the baby -s oral
mucosa and tongue were covered with a thick white deposit. In the material taken from
the affected site a bacteriologist revealed the presence of yeast fungi giving the reasons
for suspecting a fungal infection which occurs most often in children of this age, namely:
A. * Candidiasis
B. Favus
C. Epidermophytosis
D. Actinomycosis
E. Trichophytia
386. It is known that infectious type B hepatitis is a systemic disease caused by the type B hepatitis
virus and characterized by a predominant liver affection. Choose from the below given list the
drugs for the etiotropic therapy of this infection:
A. * Acyclovir
B. Penicillin
C. Tetracycline
D. Sulfanilamides
E. Fluoroquinolones
387. A female patient has been treated with antibiotics for a long time. Thereafter examination
of smears form vaginal secretion revealed oval cells with well-defined nucleus, some
cells gemmate. What preparations can help to confirm the diagnosis candidosis?
A. * Antifungal
B. Antibacterial
C. Antichlamydial
D. Antiviral
E. Antiprotozoal
388. Analysis of sputum obtained from a patient with suspected pneumonia revealed gram-positive
diplococci. They were slightly elongated, with the pointed opposite ends. What microorganisms
were revealed in the sputum?
A. * Streptococcus pneumoniae
B. Staphylococcus aureus
C. Klebsiella pneumoniae
D. Neisseria meningitidis
E. Streptococcus pyogene
389. Examination of air state in hospital was done by method of sedimentation. It revealed 5 small
roundish colonies with zone of hemolysis around them. Inoculations were made on the following
cultural medium:
A. * Blood agar
B. Endo agar
E. Levin agar
390. A female patient has been treated with antibiotics for a long time. Thereafter examination of
smears form vaginal secretion revealed oval cells with well-defined nucleus, some cells gemmate.
What preparations can help to confirm the diagnosis +ACI-candidosis+ACI-?
A. * Antifungal
B. Antibacterial
C. Antichlamydial
D. Antiviral
E. Antiprotozoal
391. A patient is suspected to have the typhoid fever. What method of laboratory diagnostics would be
the most appropriate for confirmation of this diagnosis in the first week of disease?
A. * Hemoculture identification
C. Myeloculture identification
D. Biliculture identification
E. Coproculture identification
392. Analysis of sputum obtained from a patient with suspected pneumonia revealed gram-positive
diplococci. They were slightly elongated, with the pointed opposite ends. What microorganisms
were revealed in the sputum?
A. * Streptococcus pneumoniae
B. Staphylococcus aureus
C.
Klebsiella pneumoniae
D. Neisseria meningitidis
E. Streptococcus pyogenes
393. During bacteriological analysis of solutions prepared in a pharmacy some red colonies with
metallic glitter have grown on Endo agar. What microbes were revealed?
A. * Escherichia
B. Shigella
C. Staphylococci
D. Streptococci
E. Salmonella
394. Medical examination of a dairymaid revealed affection of the locomotive system, vision
impairment, disorder of the nervous and other systems. To confirm the diagnosis the patient was
referred for a serological assay (Wright+IBk-s reaction) and Burnet's skin allergy test. What was
the provisional diagnosis?
A. * Brucellosis
B. Tularemia
C. Anthrax
D. Rheumatism
E. Leptospirosis
395. A sick child with diphtheria 10 days after the injection of antitoxic diphtheria serum, there were
skin rash, accompanied by severe itching, increased body temperature to 38, there were pains in
the joints. What is the cause of these events you expect?
A. Delayed-type hypersensitivity
B. Contact allergy
C. Anaphylactic reaction
D. * Serum sickness
E. Atopy
396. Veterinarny assistant, working on cattle farm went to a doctor complaining of joint pain, fever,
malaise, night sweats. Ill for about a month. Given that the patient is working on a farm and
livestock related complaints, the doctor suspected he had brucellosis. What material is taken from
this patient to be investigated in the usual microbiological laboratory?
A. Sputum
B. * Blood serum
C. Vomit fluid
D. Urine
E. Feces
397. A patient, sick 3 days ago, complaining of the increased temperature of 38 +ALA- C, abdominal
pain, loose stools, presence of blood in the stool, the doctor diagnosed clinically bacterial
dysentery. What method of microbiological diagnosis is advisable to apply in this case and what
material should be taken from the patient to confirm the diagnosis?
A. * Bacteriologycal
B. Serological ,blood
C. Smear ,blood
D. Smear , feces
E. Bacteriology, urine
398. A patient is suspected to have the typhoid fever. What method of laboratory diagnostics would be
the most appropriate for confirmation of this diagnosis in the first week of disease?
A. * Hemoculture identification
C. Myeloculture identification
D. Biliculture identification
E. Coproculture identification
399. A patients with gastric ulcer with hyperacidity after endoscopy and bacteriological studies were
identified bacteria of the genus Helicobacter. Thanks to some property of these organisms are not
killed in the acidic environment of the stomach?
A. * Urease activity
C. Oxidase activity
D. Catalase activity
E. Resistance to vancomycin
400. A patient with pustular skin lesions isolated pathogen, which is on blood agar forms a rounded,
medium sized, yellow colonies surrounded by a zone of hemolysis. In smears made from the
colonies Gr +- cocci appear in clusters were found. Pure culture is oxidase-and catalase-positive,
ferments mannitol, synthesizes plasma coagulase. Which of the following type matches the
selected agent?
A. * Staphylococcus aureus
B. Staphylococcus saprophyticus
C. Streptococcus agalactiae
D. Staphylococcus epidermidis
E. Streptococcus pyogenes
401. A patient, 68 years old, was subjected to prolonged antibiotic therapy. After such treatment, the
patient appeared whitish coating on the oral mucosa. Microscopic examination of smear revealed
large, round, various sizes, gram positive microorganisms. What should be done to continue the
microbiological diagnosis?
402. A patient visited a dentist with complaints of redness and edema of his mouth mucous membrane
in a month after dental prosthesis. The patient was diagnosed with allergic stomatitis. What type
of allergic reaction by Gell and Cumbs underlies this disease?
A. Cytotoxic
C. Anaphylactic
D. Stimulating
E. Immunocomplex
403. Donor skin transplantation was performed to a patient with extensive burns. On the 8-th day the
graft became swollen and changed colour+ADs- on the 11-th day graft rejection started. What
cells take part in this process?
A. * B-lymphocytes
B. Erythrocytes
C. T-lymphocytes
D. Basophils
E. Eosinophils
404. A patient with suspected diphtheria went through bacterioscopic examination. Examination of
throat swab revealed rod-shaped bacteria with volutin granules. What etiotropic preparation
should be chosen in this case?
A. * Interferon
B. Bacteriophage
C. Diphtheria antitoxin
E. Eubiotic
405. While studying a microslide obtained from the punctuate of a regional lymph node and stained by
Romanovsky-Giemsa method a physician revealed some light-pink thin microorganisms with 12-
14 regular spiral coils and pointed ends, up to 10-13 micrometer long. This might be the causative
agent of the following disease:
A. * Syphilis
B. Trypanosomiasis
C. Leptospirosis
D. Relapsing fever
E. Leishmaniasis
406. A man who was bitten by the unknown dog applied to the surgeon. Wide ragged woundes were
localised on the face. What curative-prophylactic aid should be given to prevent rabies?
407. A consumptive patient has an open pulmonary form of disease. Choose what sputum staining
should be selected for finding out the tubercle (Koch's) bacillus?
A. * Method of Ziel-Neelsen
B. Method of Romanowsky-Giemsa
C. Method of Gram
D. Method of Neisser
E. Method of Burry-Gins
408. Vomiting matters of a patient suspected of having cholera were delivered to the bacteriological
laboratory. The material was used for preparing a -hanging drop- specimen. What type of
microscopy will be applied for identification of the causative agent by its mobility?
A. * Phase-contrast microscopy
B. Electron microscopy
D. Fluorescence microscopy
E. Immersion microscopy
409. In a 2-year-old child with catarrhal presentations and skin rash a pediatrician suspected scarlet
fever. The child was given intracutaneously a small dose of serum antibody to the streptococcal
erythrogenic toxin+ADs- on the site of injection the rash disappeared. What do the reaction
results mean?
410. From the defecation of a 6-year-old ill child, who has artificial feeding, the intestinal bacillus
with antigen structure 0-111 is excreted. What is the diagnosis?
A. * Coli-enteritis
B. Gastroenteritis
C. Cholera-like diseasis
D. Food poisoning
E. Dysentery-like diseasis
411. For serological diagnostics of the whooping cough it was made large-scale reaction with
parapertussis and pertussis diagnosticums. At the bottom of the test-tubes with diagnosticum of
Bordetella parapertussis grain-like sediment formed. What antibodies have this reaction
revealed?
A. * Agglutinins
B. Precipitins
C. Opsonins
D. Bacteriolysins
E. Antitoxins
412. From pharynx of a child with suspected diphtheria a pure culture of microorganisms was isolated.
Their morphological, tinctorial, cultural and biochemical properties appeared to be typical for
diphtheria causative agents. What study should be conducted in order to drow a conclusion that
this is a pathogenic diphtheria bacillus?
413. On bacteriological examination of the defecation of a 4-months-old baby with the symptoms of
acute bowel infection there were revealed red colonies spread in the large quantity in the Endo
medium. What microorganism can it be?
A. * Escherichia
B. Salmonella
C. Staphylococcus
D. Streptococcus
E. Shigella
415. Retrospective diagnostics of old bacillary dysentery required serologic examination of blood
serum in order to determine blood titer to the shigella. What reaction should be applied for this
purpose?
C. Precipitation reaction
D. Hemolysis reaction
E. Bacteriolysis reaction
416. A hospitalized patient bitten by a rabid animal has an avulsive wound of shin. What kind of
vaccine must be given to prevent rabies?
A. * Anti-rabies vaccine
B. DTaP
C. Td
D. BCG
E. TABte
417. A smear of sputum from the patient with suspected lobar pneumonia was stained with
the use of the following dyies and reagents: crystal violet, iodin, alcohol, water, safranin.
What staining method was applied in this case?
A. * Gram metod
B. Neisser method
C. Burri method
E. Gray method
418. ?It is planned to use the territory of an old cattle burial ground (which is not used for more than
50 years) for building houses. But ground analysis revealed presence of the pathogen of the very
dangerous illness. Which of the indicated microorgonisms is likely to remain in the ground for
such a long time?
A. * Bacillus anthracis
B. Francisella tularensis
C. Brucella abortus
D. Yersinia pestis
E. Mycobacterium bovis
419. From the nasopharynx of a 5-year-old child it was excreted amicroorganism which is identical to
Corynebacterium diphtheriae dose according to morphological and biochemical
signs.Microorganism does not produce exotoxin. As a result of what process can this
microorganism become toxigenic?
A. *
Phage conversion
E. Chromosome mutation
420. While studying a microslide obtained from the punctuate of a regional lymph node and stained by
Romanovsky-Giemsa method a physician revealed some light-pink thin microorganisms with 12-
14 regular spiral coils and pointed ends, up to 10-13 micrometer long. This might be the causative
agent of the following disease:
A. * Syphilis
B. Trypanosomiasis
C. Leptospirosis
D. Relapsing fever
E. Leishmaniasis
421. Sanitary bacteriological research on water by the membrane filter method revealed two red
colonies on a membrane filter (Endo agar) through which 500 ml of analyzed water were passed.
Calculate the coli index and coli titer of the analyzed water:
A. * 4 and 250
B. 2 and 500
C. 250 and 4
D. 500 and 2
E. 250 and 2
422. While examining a patient an otolaryngologist noticed hyperaemia and significantly edematous
tonsils with a grayish film upon them. Microscopical examination of this film revealed some
gram-positive bacilli placed at an angle with each other. What disease might be suspected?
A. * Diphtheria
B. Angina
C. Scarlet fever
D. Meningococcal nasopharyngitis
E. Epidemic parotitis
423. Patient with vomiting, dizziness, sensation of dubble vision, difficult swallowing was admitted to
the hospital. Doctor suspects botulism. What diagnostic methods should be used for diagnosis
approving?
C. Bacteriological, mycological
D. Protozoological, microscopical
E. Allergic test
424. A man who was bitten by the unknown dog applied to the surgeon. Wide ragged woundes were
localised on the face. What curative-prophylactic aid should be given to prevent rabies?
425. In a patient with clinical signs of immunodeficiency the number and functional activity of T and
B lymphocytes are not changed. Defect with dysfunction of antigen-presentation to the
immunocompetent cells was found during investigation on the molecule level. Defect of what
cells is the most probable?
A. * Macrophages, monocytes
B. Т-lymphocytes, В-lymphocytes
C. NK-cells
E. 0-lymphocytes
426. A patient with complaints of 3-day-long fever, general weakness, loss of appetite came to visit
the infectionist. The doctor suspected enteric fever. Which method of laboratory diagnosis is the
best to confirm the diagnosis?
B. Detachment of myeloculture
427. A consumptive patient has an open pulmonary form of disease. Choose what sputum staining
should be selected for finding out the tubercle (Koch's) bacillus?
A. * Method of Ziel-Neelsen
B. Method of Romanowsky-Giemsa
C. Method of Gram
D. Method of Neisser
E. Method of Burry-Gins
428. During surgical operation a blood transfusion was made. The blood must be checked to find
antigens of some disease. What disease is expected to be found?
A. * Virus of hepatitis B
B. Virus of hepatitis A
C. Adenovirus
D. Enterovirus
E. Virus of hepatitis E
429. A patient who came to the doctor because of his infertility was administered to make tests for
toxoplasmosis and chronic gonorrhoea. Which reaction should be performed to reveal
C. Immunoblot analysis
430. On bacteriological study of rinsing water of the patient with food poisoning, the pure bacterial
culture was inoculated with the following properties: gram-negative motile bacillus in the Endo
environment grows like achromic colony. Representative of what genus has caused the illness?
A. * Salmonella
B. Shigella
C. Yersinia
D. Escherichia
E. Citrobacter
431. The person was selling "homemade pork" sausages on the market. State sanitary inspector
suspected falcification of the sausages.With help of what serological immune reaction can food
substance be identified?
A. * Precipitation test
C. Agglutination test
D. Immunofluorescence test
432. While registering the child to the school Mantu's test was made to define whether revaccination
was needed test result is negative. What does this result of the test mean?
433. The donor who for a long time didn't donate the blood was investigated with IFA method. Anti-
HBs antibodies were revealed. What does positive result of IFA in this case mean?
A. * Previous hepatitis B
B. Acute hepatitis B
C. Acute hepatitis C
D. Chronic hepatitis В
E. Chronic hepatitis С
434. Bacteriological examination of purulent discharges from the urethra revealed gram-negative
bacteria looking like coffee beans. They were localized in the leukocytes and could decompose
glucose to acid. These are the causative agents of the following disease:
A. * Gonorrhoea
B. Syphilis
C. Veneral lymphogranulomatosis
D. Soft chancre
E. Melioidosis
435. Scraps of the mycelium of a fungus, spores, air bubbles and fat drops were discovered on
microscopy of the patient's hair excluded from the infected areas. For what fungus disease is this
microscopic picture characteristic?
A. * Favus
B. Microspory
C. Trichophytosis
D. Epidermophytosis
E. Sporotrichosis
436. In order to speed up healing of a wound of oral mucosa a patient was prescribed a drug that is a
thermostable protein occuring in tears, saliva, mother's milk as well as in a new-laid hen's egg. It
is known that this protein is a factor of natural resistance of an organism. What is it called?
A. * Lysozyme
B. Complement
C. Interferon
D. Interleukin
E. Imanine
437. Study of bacteriological sputum specimens stained by the Ziel-Neelsen method revealed some
bright-red acid-resistant bacilli that were found in groups or singularly. When inoculated onto the
nutrient media, the signs of their growth show up on the 10-15 day. These bacteria relate to the
following family:
A. * Micobacterium tuberculosis
B. Yersinia pseudotuberculosis
C. Histoplasma dubrosii
D. Klebsiella rhinoscleromatis
E. Coxiella burnettii
438. A man was admitted to the hospital on the 5th day of disease that manifested itself by jaundice,
muscle aching, chill, nose bleedings. In course of laboratory diagnostics a bacteriologist
performed dark-field microscopy of the patient's blood drop. Name a causative agent of this
disease:
A. * Leptospira interrogans
B. Borrelia dutlonii
C. Calymmatobacterium granulomatis
D. Bartonella bacilloformis
E. Rickettsia mooseri
439. Gramnegative bin-shaped diplococcus inside and outside of leucocytes were detected on
bacteriological examination of the purulent exudates from the cervix of the uterus. Name the
causative agent of purulent inflammation of the cervix of the uterus.
A. * Neisseria gonorroeae
B. Chlamidia trachomatis
C. Haemophilus vaginalis
D. Trichomonas vaginalis
E. Calymmatobacterium granulomatis
440. Patient with diarrhoea was admitted to the infection unit. Gramnegative curved rod-like bacteria
were founded on bacterioscopic examination of faecal masses. What is the most likely disease in
this patient?
A. * A Cholera
B. B Typhoid fever
C. C Salmonellosis gastroenteritis
D. D Diphtheria
441. In a 2-year-old child with catarrhal presentations and skin rash a pediatrician suspected scarlet
fever. The child was given intracutaneously a small dose of serum antibody to the streptococcal
erythrogenic toxin; on the site of injection the rash disappeared. What do the reaction results
mean?
442. From the defecation of a 6-year-old ill child, who has artificial feeding, the intestinal bacillus
with antigen structure 0-111 is excreted. What is the diagnosis?
A. * Coli-enteritis
B. Gastroenteritis
C. Cholera-like diseasis
D. Food poisoning
E. Disentery-like diseasis
443. For serological diagnostics of the whooping cough it was made large-scale reaction with
parapertussis and pertussis diagnosticums. At the bottom of the test-tubes with diagnosticum of
Bordetella parapertussis grain-like sediment formed. What antibodies have this reaction
revealed?
A. * Agglutinins
B. Precipitins
C. Opsonins
D. Bacteriolysins
E. Antitoxins
444. A man died from an acute infectious disease accompanied by fever, jaundice haemorrhagic rash
on the skin and mucous membranes as well as by acute renal insufficiency. Histological
examination of renal tissue (stained by Romanovsky-Giemsa method revealed some convoluted
bacteria looking like C und S letters. What bacteria were revealed?
A. * Leptospira
B. Treponema
C. Spirilla
D. Borrelia
E. Campilobacteria
445. A 16 y.o. boy from a countryside entered an educational establishment. Scheduled Mantoux test
revealed that the boy had negative reaction. What are the most reasonable actions in this case?
446. Examination of a patient with pustular skin lesions allowed to isolate a causative agent that forms
in the blood agar roundish yellow middle-sized colonies surrounded by haemolysis zone. Smears
from the colonies contain irregular-shaped clusters of gram-positive cocci. The culture is oxidase-
and catalase-positive, ferments mannitol and synthesizes plasmocoagulase. What causative agent
was isolated?
A. * Staphylococcus aureus
B. Streptococcus agalactiae
C. Streptococcus pyogenes
D. Staphylococcus epidermidis
E. Staphylococcus saprophyticus
447. Microscopic examination of a Gram-stained scrape from patient's tongue revealed oval, round,
elongated chains of dark-violet gemmating cells. What disease can be caused by this causative
agent?
A. * Candidosis
B. Actinomycosis
C. Streptococcic infection
D. Staphylococcic infection
E. Diphtheria
448. From pharynx of a child with suspected diphtheria a pure culture of microorganisms was isolated.
Their morphological, tinctorial, cultural and biochemical properties appeared to be typical for
diphtheria causative agents. What study should be conducted in order to drow a conclusion that
this is a pathogenic diphtheria bacillus?
449. Examination of a child revealed some whitish spots looking like coagulated milk on the mucous
membrane of his cheeks and tongue. Analysis of smears revealed gram-positive oval yeast-like
cells. What causative agents are they?
A. * Candida
B. Staphylococci
C. Diphtheria bacillus
D. Actinomycetes
E. Fusobacteria
450. Blood of a patient with presumable sepsis was inoculated into sugar broth. There appeared
bottom sediment. Repeated inoculation into blood agar caused growth of small transparent round
colonies surrounded by hemolysis zone. Examination of a smear from the sediment revealed
gram-positive cocci in form of long chains. What microorganisms are present in blood of this
patient?
A. * Streptococci
B. Micrococci
C. Staphylococci
D. Tetracocci
E. Sarcina
451. On bacteriological examination of the defecation of a 4-months-old baby with the symptoms of
acute bowel infection there were revealed red colonies spread in the large quantity in the Endo
medium. What microorganism can it be?
A. * Escherichia
B. Salmonella
C. Staphylococcus
D. Streptococcus
E. Shigella
452. Bacteriological examination of a patient with food poisoning required inoculation of a pure
culture of bacteria with the following properties: gram-negative movable bacillus that grows n the
Endo medium in form of colourless colonies. A representative of which species caused this
disease?
A. * Salmonella
B. Shigella
C. Yersinia
D. Esherichia
E. Citrobacter
453. Examination of a young man in the AIDS centre produced a positive result of immune-enzyme
assay with HIV antigens. Patient's complaints about state of his health were absent. What can the
positive result of immune-enzyme assay be evidence of?
A. * HIV infection
E. HBV persistence
454. Microscopy of stained (Ziehl-Neelsen staining) smears taken from the sputum of a patient with
chronic pulmonary disease revealed red bacilli. What property of tuberculous bacillus was shown
up?
A. * Acid resistance
B. Alkali resistance
C. Alcohol resistance
D. Capsule formation
E. Sporification
456. In order to determine toxigenicity of diphtheria bacilli a strip of filter paper impregnated with
antitoxic diphtherial serum was put on the dense nutrient medium. There were also inoculated a
microbal culture under examination and a strain that is known to be toxigenic. If the microbal
culture under examination produces exotoxin, this wil result in formation of:
A. * Precipitin lines
B. Haemolysis zones
E. Precipitin ring
457. A 50-year-old patient with typhoid fever was treated with Levomycetin, the next day his
condition became worse, temperature rised to 39,6 о С. What caused worthening?
E. Reinfection
458. In order to estimate toxigenity of diphtheria agents obtained from patients the cultures were
inoculated on Petri dish with nutrient agar on either side of a filter paper strip that was put into
the center and moistened with antidiphtheric antitoxic serum. After incubation of inoculations in
agar the strip-like areas of medium turbidity were found between separate cultures and the strip of
filter paper. What immunological reaction was conducted?
B. Coomb's test
C. Agglutination reaction
E. Opsonization reaction
459. A patient with clinical signs of encephalitis was delivered to the infectious diseases hospital.
Anamnesis registers a tick bite. Hemagglutination-inhibition reaction helped to reveal antibodies
to the causative agent of tick-borne encephalitis in the dilution 1:20 which is not diagnostic. What
actions should the doctor take after he had got such result?
460. The first grade pupils were examined in order to sort out children for tuberculosis revaccination.
What test was applied for this purpose?
A. * Mantoux test
B. Schick test
D. Burnet test
E. Anthraxine test
461. Clinical diagnosis of a female patient was gonorrhoea. What examination method can be applied
for confirmation of this diagnosis?
A. * Microscopy of pathological material
D. Hemagglutination reaction
E. Immobilization reaction
462. A patient suffering from periodical attacks caused by inhalation of different flavoring substances
was diagnosed with atopic bronchial asthma. IgE level was increased. This is typical for the
following type of reactions:
A. * Anaphylactic reactions
B. Cytotoxic reactions
C. Immunocomplex reactions
D. delayed-type hypersensitivity
E. Autoimmune reactions
463. Bacteriological laboratory examines canned meat whether it contains botulinum toxin. For this
purpose an extract of test specimen and antitoxic antibotulinic serum of A, B, E types were
introducted to a group of mice under examination; a control group of mice got the extract without
antibotulinic serum. What serological reaction was applied?
A. * Neutralization
B. Precipitation
C. Complement binding
D. Opsono-phagocytic
464. For the purpose of retrtospective diagnostics of recent bacterial dysentery it was decided to
perform serological examination of blood serum in order to determine antibody titer towards
Shiga bacilli. What of the following reactions should be applied?
A. * Passive hemagglutination
B. Bordet-Gengou test
C. Precipitation
D. Hemolysis
E. Bacteriolysis
465. During the repeated Widal agglutination test it was noticed that the ratio of antibody titers and O-
antigens S. typhi in the patient's serum had increased from 1:100 to 1:400. How would you
interpret these results?
466. A patient recovered from Sonne dysentery and was once more infected with the same causative
agent. What is such infection form called?
A. * Reinfection
B. Recidivation
C. Superinfection
D. Persisting infection
E. Chronic infection
467. A 10-year-old child had the mantoux tuberculin test administered. 48 hours later a papule up to 8
mm in diameter appeared on the site of the injection. What type of hypersensitivity reaction
developed after the tuberculin injection?
B. Arthus phenomenon
C. Seroreaction
D. Atopic reaction
469. As a result of durative antibiotic therapy a 37-year old patient developed intestinal dysbacteriosis.
What type of drugs should be used in order to normalize intestinal microflora?
A. * Eubiotics
B. Sulfanilamides
C. Bacteriophages
D. Autovaccines
E. Vitamins
470. Among junior children of an orphanage an outbreak of intestinal infection with signs of
colienteritis was registered. In order to identify isolated causative agent it is necessary to:
471. Urine examination of a patient with acute cystitis revealed leukocytes and a lot of gram-
negative bacilli. Inoculation resulted in growth of colonies of mucous nature that formed green
soluble pigment. What microorganism is the most probable cause of the disease?
A. * Pseudomonas aeruginosa
B. Escherihia coli
C. Klebsiella pneumoniae
D. Proteus mirabilis
E. Salmonella enteritidis
472. A laboratory received a material from a patient's wound. Preliminary diagnosis is gaseous
gangrene. What microbiological method should be applied to determine species of causative
agent?
A. * Bacteriological
B. Allergic
C. Bacterioscopic
D. Serological
E. RIA
473. A virological laboratory obtained pathological material (mucous discharges from nasal meatuses)
taken from a patient with provisional diagnosis "influenza". What quick test will allow to reveal
specific viral antigen in the material under examination?
B. AT
D. Radioimmunoassay
E. ELISA
474. In the surgical department of a hospital there was an outbreak of hospital infection that showed
itself in often postoperative wound abscesses. Bacteriological examination of pus revealed
Staphylococcus. What examination shall be conducted to find out the source of this causative
agent among the department personnel?
A. * Phagotyping
B. Microscopical examination
C. Serological identification
E. Biochemical identification
475. A 7 year old child often suffers from streprococcal angina. Doctor suspected development of
rheumatism and administered serological examination. The provisional diagnosis will be most
probably confirmed by presence of antibodies to the following streptococcal antigen:
A. * O-streptolysin
B. C-carbohydrate
C. M-protein
D. Erythrogenic toxin
E. Capsular polysaccharide
476. A culture of monkey cells (Vero) and a group of sucking mouse were infected with an inoculum
taken from a child with provisional diagnosis "enterovirus infection". There was no cytopathic
effect on the cell culture but mouse sucking died. What enteric viruses might have caused disease
of this child?
A. * Coxsackie A
B. Coxsackie B
C. ECHO virus
D. Polioviruses
477. A patient has been suffering from elevated temperature and attacks of typical cough for 10 days.
Doctor administered inoculation of mucus from the patient's nasopharynx on the agar. What
microorganism is presumed?
A. * Pertussis bacillus
B. Pfeiffer's bacillus
C. Listeria
D. Klebsiella
E. Staphylococcus
478. A patient of surgical department complains about pain in the small of her back and in the lower
part of her belly; painful and frequent urination. Bacteriological examination of urine revealed
gram-negative oxidase-positive rod-like bacteria forming greenish mucoid colonies with specific
smell. What causative agent can it be?
A. * Pseudomonas aeruginosa
B. Proteus mirabilis
C. E.coli
D. Str.pyogenes
E. Mycoplasma pneumonie
A. * Clostridia
B. Streptococci
C. Spirochaete
D. Actinomycete
E. Diplococci
480. A specimen stained by Ozheshko method contains rod-like microorganisms stained blue with
round terminal components stained red. What are these components called?
A. * Spores
B. Cilia
C. Flagella
D. Capsules
E. Mesosomas
A. * Haemolytic streptococci
B. Micrococci
C. Bacilli
D. Yeast fungi
E. Sarcina
482. A bacteriological laboratory received sputum sample of a patient suffering from tuberculosis.
Bacterioscopic examination of smears and detection of tuberculosis bacillus can be realized by
one of enrichment methods that involves processing of sputum only with solution of caustic soda.
What is this method called?
A. * Homogenization
B. Inactivation
C. Flotation
D. Filtration
E. Neutralization
483. A pregnant woman was registered in an antenatal clinic and underwent complex examination for
a number of infections. Blood serum contained IgM to the rubella virus. What is this result
indicative of?
A. * Of primary infection
B. Of a chronic process
484. A 65-year-old man has purulent abscess on his neck. Analyses revealed a culture of gram-positive
cocci with plasmocoagulase activity. This culture relates most likely to:
A. * Staphylococcus aureus
B. Streptococcus pyogenes
C. Staphylococcus epidermidis
D. Staphylococcus saprophyticus
E. Klebsiella pneumoniae
485. Material taken from a patient with provisional diagnosis "influenza" was referred to a laboratory.
For virological examination the hemadsorption reaction was applied. This reaction can be applied
for detection of the following viruses:
D. DNA-genomic viruses
E. Any viruses
486. Inoculum from pharynx of a patient ill with angina was inoculated into blood-tellurite agar. It
resulted in growth of grey, radially striated (in form of rosettes) colonies 4-5 mm in diameter.
Gram-positive bacilli with clublike thickenings on their ends placed in form of spread wide apart
fingers are visible by microscope. What microorganisms are these?
A. * Diphtheria corynebacteria
B. Botulism clostridia
C. Diphtheroids
D. Streptococci
E. Streptobacilli
487. During examination of a patient a dentist revealed a lot of "white spots" - zones of enamel
demineralization. What microorganisms take part in the development of this process?
A. * Streptococcus mutans
B. Streptococcus salivarius
C. Streptococcus pyogenes
D. Veilonella parvula
E. Staphylococcus epidermidis
488. Planned mass vaccination of all newborn 5-7 day old children against tuberculosis plays an
important role in tuberculosis prevention. In this case the following vaccine is applied:
A. * BCG
E. TABTe
489. A 4-year-old child presents with general weakness, sore throat and deglutitive problem. After his
examination a doctor suspected diphtheria and sent the material to the bacteriological laboratory.
In order to determine the diphtheria causative agent the material should be inoculated into the
following differential diagnostic medium:
B. Endo agar
C. Ploskyrev agar
D. Sabouraud agar
E. Levenshtein-Yessen agar
490. Vomiting matters of a patient suspected of having cholera were delivered to the bacteriological
laboratory. The material was used for preparing a "hanging drop" specimen. What type of
microscopy will be applied for identification of the causative agent by its mobility?
A. * Phase-contrast microscopy
B. Electron microscopy
D. Fluorescence microscopy
E. Immersion microscopy
491. In the surgical ward, the dressing material was undergoing sterilization in an autoclave. Through
an oversight of a nurse the mode of sterilization was changed and the temperature in the
autoclave reached only 100?C instead of the due 120?C. What microorganisms can stay viable
under these conditions?
B. Absence of flagella
C. Absence of a capsule
D. Absence of a spore
E. Absence of inclusions
493. There are areas where humans or animals are exposed to the constant risk of contracting certain
types of bacteria. What feature of these bacteria is responsible for their long viability in the soil?
A. * Spore formation
B. Capsule formation
E. Plasmids
494. Analysis of the cerebrospinal fluid of a child with signs of purulent lesion of brain tunics revealed
gram-negative bean-shaped diplococci. What presumptive diagnosis can be made on the basis of
the analysis results?
A. * Meningitis
B. Gonorrhea
C. Cholera
D. Plague
E. Anthrax
495. The causative agent of botulism causes severe food poisoning. Specify the most characteristic
morphological feature of botulism causative agent:
496. Microbiological analysis of medicinal raw materials revealed capsular bacteria. What stain
method was used to detect the capsules?
A. * Burry-Gins
B. Ziehl-Neelsen
C. Neisser
D. Gram
E. Ozheshko
497. Bacterioscopic examination of chancre material revealed some mobile, long, convoluted
microorganisms with 8-12 regular coils. These features are typical for:
A. * Treponema
B. Borrellia
C. Leptospira
D. Vibrios
E. Campylobacter
498. Infectious agents of various ultrastructures can be etiological agents of infectious diseases. Which
of the groups named below HAS NO cellular structure, protein synthesizing, enzyme and energy
systems?
A. * Viruses
B. Fungi
C. Bacteria
D. Protozoa
E. Rickettsia
499. What method of sterilization should be used during the liquid media containing proteins?
A. * Filtering
B. Boiling
C. Gas sterilization
D. Autoclaving
E. Pasteurization
500. Meat peptone broth is prepared for sterilization in bateriological laboratory. What sterilization
method is advisable?
A. * Autoclaving
B. Ignition
C. Boiling
D. Filtering
E. Dry heat
501. A patient has a necrotizing phlegmon of his lower extremity. A doctor suspects a gas gangrene.
Microscopy reveals Gram-positive bacilli. In order to confirm the diagnosis further
bacteriological tests should include inoculation of the material into the following nutrient
medium:
A. * Kitt-Tarozzi medium
B. Endo agar
C. Levine agar
D. Meat-peptone agar
E. Milk-salt agar
502. Bacteria eventually become resistant to antibacterial agents. Resistance of gram-positive bacteria
to penicillin antibiotics is caused by:
A. * Beta-lactamase production
E. Protein synthesis
503. Which one of the listed substances causes formation of acquired artificial passive immunity?
A. * Tetanus toxoid
B. BCG
C. MMR
D. DPT
E. TABTe
504. It is necessary to carry out preventive vaccination of a student group because of an occurrence of
diphtheria. Which preparation should be used for the creation of the artificial active immunity?
A. * Diphtheria toxoid
B. Specific immunoglobulin
C. DTP vaccine
E. Anti-diphtheria serum
505. For the specific prevention of influenza, the employees of an enterprise were vaccinated with
"Influvac". What type of immunity will develop in the body of the vaccinated?
A. * Artificial active
B. Innate congenital
C. Artificial passive
D. Natural active
E. Natural passive
506. Dysbiosis can be treated with drugs that contain living representatives of normal microflora as
well as their metabolic products. Select the microorganisms that are used for the production of
such drugs:
A. * Bifidum bacteria
B. Staphylococcus aureus
C. Proteus
D. Providencia
E. Yersinia
507. A 26-year-old female patient with bronchitis has been administered a broad spectrum antibiotic as
a causal treatment drug. Specify this drug:
A. * Doxycycline
B. Interferon
C. BCG vaccine
D. Ambroxol
E. Dexamethasone
508. What condition may develop 15-30 minutes after re-administration of the antigen as a result of
the increased level of antibodies, mainly IgE, that are adsorbed on the surface of target cells,
namely tissue basophils (mast cells) and blood basophils?
A. * Anaphylaxis
B. Antibody-dependent cytotoxicity
C. Delayed-type hypersensitivity
E. Serum sickness
509. 10 days after having quinsy caused by beta-hemolytic streptococcus a 6-year-old child exhibited
symptoms of glomerulonephritis. What mechanism of glomerular lesion is most likely in this
case?
A. * Immunocomplex
B. Cellular cytotoxicity
C. Anaphylaxis
D. Atopy
510. 6 hours after the initial inoculation of water sample into 1% peptone water, the growth of a
culture in form of a thin pellicle on the medium surface was registered. Such cultural properties
are typical for the causative agent of the following disease:
A. * Cholera
B. Plague
C. Tuberculosis
D. Dysentery
E. Pseudotuberculosis
511. Analysis of the cerebrospinal fluid of a child with signs of purulent lesion of brain tunics revealed
gram-negative bean-shaped diplococci. What presumptive diagnosis can be made on the basis of
the analysis results?
A. * Meningitis
B. Gonorrhea
C. Cholera
D. Plague
E. Anthrax
512. Microbiological purity of tabulated drugs had been tested at factory. Samples cultivation in
mannitol salt agar resulted in growth of golden-yellow colonies, microscopic examination of
colonies detected Gram-positive globular bacteria positioned in clusters; microorganisms had
plasma coagulation properties. What pure bacterial culture was obtained?
A. * Staphylococcus aureus.
B. Escherichia coli
C. Staphylococcus epidermidis.
D. Staphylococcus saprophyticus.
E. Pseudomonas aeruginosa.
513. Microbiological studies of air in the pharmacy room revealed the presence of pathogenic
staphylococci. Select the medium in which you can detect the lecithinase activity of the isolated
microorganism:
A. * Yolk-salt agar
B. Blood agar
D. Sugar agar
E. Meat-extract agar
514. There is a suspicion of active tuberculosis development in patient. The doctor has appointed
Mantoux test to make a diagnosis. What immunobiological agent has to be administered?
A. * Tuberculin
B. BCG vaccine
C. DPT vaccine
D. Tularin
E. DT vaccine
515. A young woman suddenly developed fever up to 39oC accompanied by a strong headache.
Examination revealed marked nuchal rigidity. Spinal puncture was performed. Gram-stained
smear of cerebrospinal fluid contained many neutrophils and Gram-negative diplococci. What
bacteria could be the cause of this disease?
A. * Neisseria meningitidis
B. Streptococcus pneumonia
C. Haemophilus influenzae
D. Staphylococcus aureus
E. Pseudomonas aeruginosa
516. A bacteriological laboratory studied the home-made dried fish which had caused a severe food
poisoning. Microscopy of the culture grown on the Kitt-Tarozzi medium revealed
microorganisms resembling a tennis racket. What is the most likely diagnosis?
A. * Botulism
B. Salmonellosis
C. Cholera
D. Dysentery
E. Typhoid fever
517. Quite often, the soil may contain a number of pathogenic microorganisms. The causative agents
of the following disease may stay viable in the soil for a long time:
A. * Anthrax
B. Diphtheria
C. Viral hepatitis
D. Pertussis
E. Dysentery
518. A patient presents with fever, chill and cough. From his sputum the ovoid Gram-negative bipolar-
stained bacilli with a delicate capsule were isolated. What is the most likely diagnosis?
A. * Plague
B. Tuberculosis
C. Leptospirosis
D. Brucellosis
E. Toxoplasmosis
519. A patient has been hospitalised with provisional diagnosis of botulism. What serological reaction
should be used to reveal botulinum toxin?
A. * Neutralization reaction
B. Agglutination reaction
C. Bordet-Gengou test
D. Precipitation reaction
E. Immunofluorescence test
520. In a village, a case of anthrax had been registered. Medical services began epidemiologically
indicated specific prophylaxis of population against anthrax. What preparation was used for this
purpose?
A. * Live vaccine
B. Inactivated vaccine
C. Chemical vaccine
E. Anatoxin
521. A 55-year-old patient with a characteristic rash, fever, dizziness has been admitted to a hospital.
He has been provisionally diagnosed with typhus. No similar cases have been reported. In his
youth (15 years old) the patient suffered typhus in a boarding school. What disease is it?
A. * Brill disease
B. Typhoid fever
C. Measles
D. Rubella
E. Cholera
522. A patient has severe catarrhal symptoms. Material growth on Bordet-Gengou agar showed
mercury-drop like colonies. Examination of the blood smears revealed some small ovoid gram
positive bacilli sized 1-3 microns. What microorganisms were isolated?
A. * Bordetella
B. Corynebacteria
C. Mycobacteria
D. Meningococcus
E. Brucella
523. The laboratory for especially dangerous infections conducts microscopic examination of
pathological material from a patient with suspected plague. The sample was stained by Burri-
Gins technique. What property of the causative agent can be identified by this technique?
A. * Capsule formation
B. Spore formation
C. Acid resistance
D. Alkali resistance
E. Presence of volutin granules
524. A child cut his leg with a piece of glass while playing and was brought to the clinic for the
injection of tetanus toxoid. In order to prevent the development of anaphylactic shock the serum
was administered by Bezredka method. What mechanism underlies this method of desensitization
of the body?
525. A 32-year-old patient undergoing dental examination was found to have some rash-like lesions
resembling secondary syphilis in the oral cavity. The patient was referred for the serological
study with the purpose of diagnosis confirmation. In order to detect antibodies in the serum,
living Treponema were used as diagnosticum. What serological test was performed?
A. * Immobilization
B. Neutralization
C. Complement binding
D. Precipitation
E. Passive hemagglutination
526. Examination of a 27-year-old donor who had not donated blood for a long time revealed HBs
antibodies detected by ELISA method. In this case, the positive ELISA results indicate that the
donor:
A. * Had hepatitis B
527. In the area being the epicenter of the registered rabies cases among wild animals a 43-year-old
man presented to a clinic and claimed to have been bitten by a stray dog. He was given a course
of anti-rabies vaccine. This preparation relates to the following type of vaccines:
A. * Attenuated
B. Inactivated
C. Molecular
D. Toxoids
E. Synthetic
528. RNA-containing viruses that cause tumors in animals, genetic information can be transmitted in
the opposite direction from the RNA to the DNA via a specific enzyme. The enzyme of reverse
transcription is called:
A. * Reverse transcriptase
B. DNA polymerase
C. Ligase
D. Primase
E. Topoisomerase
529. A patient has been hospitalised with provisional diagnosis of virus B hepatitis. Serological
reaction based on complementation of antigen with antibody chemically bound to peroxidise or
alkaline phosphatase has been used for disease diagnostics. What is the name of the applied
serological reaction?
A. * Immune-enzyme analysis
B. Radioimmunoassay technique
C. Immunofluorescence test
D. Bordet-Gengou test
E. Antigen-binding assay
530. A 3-year-old child has continuous fever, lymph nodes are enlarged, the amount of lymphocytes in
blood is significantly increased. Enzyme linked immunosorbent assay (ELISA) revealed antigen
of Epstein-Barr virus. What diagnosis can be made based on the information given above?
A. * Infectious mononucleosis
B. Burkitt’s lymphoma
C. Herpetic lymphadenopathy
E. Cytomegalovirus infection
531. Hepatitis B is diagnosed through laboratory tests that determine the presence of HBV-DNA in
blood serum of the patient. What reference method is applied for this purpose?
B. Hybridization method
C. Hybridization signal amplification
532. An outbreak of an intestinal infection occurred in a kindergarten on the eve of New Year
holidays. Bacteriological examination of patients’ faeces didn’t reveal any pathogenic bacteria.
Electronmicroscopy revealed roundish structures with clear outer edges and a thick core
resembling a wheel. Specify the most likely causative agent of this infection:
A. * Rotavirus
B. Adenovirus
C. Coxsacki-virus
D. E. coli
E. P. vulgaris
533. A patient has been admitted to the infectious diseases department for malaise, fever up to 38oC,
jaundice. A few months ago, the patient underwent blood transfusion. The doctor suspected viral
hepatitis B. What are the principal methods of laboratory diagnosis of hepatitis B?
B. Virus isolation in cell culture and its identification by the cytopathic effects
534. In our country, routine preventive vaccinations against poliomyelitis ivolve using live vaccine
that is administered orally. What immunoglobulins are responsible for the development of local
post-vaccination immunity in this case?
A. * Secretory IgA
B. IgM
C. IgG
D. Serum IgA
E. IgE
535. A laboratory received a sample of water used in drug production for sanitary and virological
analysis. What group of viruses will indicate faecal contamination of water and thus the need for
its additional purification?
A. * Picornaviridae
B. Herpesviridae
C. Orthomyxoviridae
D. Retroviridae
E. Flaviviridae
536. A sample of water used in drug production has been sent to a laboratory for sanitary and
virological analysis. Presence of what virus group will be indicative of faecal contamination of
water and thus the need for its additional purification?
A. * Picornaviridae
B. Herpesviridae
C. Orthomyxoviridae
D. Retroviridae
E. Flaviviridae
537. Virological laboratory has received patient’s nasopharyngeal lavage. What can be used to single
out influenza virus from the patient’s lavage?
A. * Chick embryo
D. Endo’s medium
E. Lowenstein–Jensen medium
538. A person has been in contact with influenza patient. What drug should be administered for
specific passive influenza prophylaxis?
A. Amizon
C. Leukocytic interferon
D. * Immunoglobulin
E. Anaferon
539. After the sanitary and bacteriological study of tap water the following results were obtained: the
total number of bacteria in 1,0 ml was 80, coli index was 3. How would you interpret the study
results?
D. Water is contaminated
540. During examination of a 3-monthold infant a pediatrician revealed that the baby’s oral mucosa
and tongue were covered with a thick white deposit. In the material taken from the affected site a
bacteriologist revealed the presence of yeast fungi giving the reasons for suspecting a fungal
infection which occurs most often in children of this age, namely:
A. * Candidiasis
B. Favus
C. Epidermophytosis
D. Actinomycosis
E. Trichophytia
541. A sick child with diphtheria 10 days after the injection of antitoxic diphtheria serum, there were
skin rash, accompanied by severe itching, increased body temperature to 38, there were pains in
the joints. What is the cause of these events you expect?
A. Delayed-type hypersensitivity
B. Contact allergy
C. Anaphylactic reaction
D. * Serum sickness
E. Atopy
542. Veterinarny assistant, working on cattle farm went to a doctor complaining of joint pain, fever,
malaise, night sweats. Ill for about a month. Given that the patient is working on a farm and
livestock related complaints, the doctor suspected he had brucellosis. What material is taken from
this patient to be investigated in the usual microbiological laboratory?
A. Sputam
B. * Blood serum
C. Vomal fluid
D. Urine
E. Feces
543. A patient, sick 3 days ago, complaining of the increased temperature of 38 ° C, abdominal pain,
loose stools, presence of blood in the stool, the doctor diagnosed clinically bacterial dysentery.
What method of microbiological diagnosis is advisable to apply in this case and what material
should be taken from the patient to confirm the diagnosis?
A. * Bacteriologycal
B. Serological ,blood
C. Smear ,blood
D. Smear , feces
E. Bacteriology, urine
544. A patients with gastric ulcer with hyperacidity after endoscopy and bacteriological studies were
identified bacteria of the genus Helicobacter. Thanks to some property of these organisms are not
killed in the acidic environment of the stomach?
A. * Urease activity
C. Oxidase activity
D. Catalase activity
E. Resistance to vancomycin
545. A patient with pustular skin lesions isolated pathogen, which is on blood agar forms a rounded,
medium sized, yellow colonies surrounded by a zone of hemolysis. In smears made from the
colonies Gr + cocci appear in clusters were found. Pure culture is oxidase-and catalase-positive,
ferments mannitol, synthesizes plasma coagulase. Which of the following type matches the
selected agent?
A. * Staphylococcus aureus
B. Staphylococcus saprophyticus
C. Streptococcus agalactiae
D. Staphylococcus epidermidis
E. Streptococcus pyogenes
546. A patient, 68 years old, was subjected to prolonged antibiotic therapy. After such treatment, the
patient appeared whitish coating on the oral mucosa. Microscopic examination of smear revealed
large, round, various sizes, gram positive microorganisms. What should be done to continue the
microbiological diagnosis?
547. In , brain cells of fox’s, trapped in the city had been found the inclusion – Babesh- Negri bodies.
What is the causative agent of fox’s infection?
A. * Rabies
B. Flu
C. Chickenpox
E. Kissing disease
548. A patient visited a dentist with complaints of redness and edema of his mouth mucous membrane
in a month after dental prosthesis. The patient was diagnosed with allergic stomatitis. What type
of allergic reaction by Gell and Cumbs underlies this disease?
A. * Cytotoxic
C. Anaphylactic
D. Stimulating
E. Immunocomplex
549. Donor skin transplantation was performed to a patient with extensive burns. On the 8-th day the
graft became swollen and changed colour; on the 11-th day graft rejection started. What cells take
part in this process?
A. * B-lymphocytes
B. Erythrocytes
C. T-lymphocytes
D. Basophils
E. Eosinophils
550. A patient with suspected diphtheria went through bacterioscopic examination. Examination of
throat swab revealed rod-shaped bacteria with volutin granules. What etiotropic preparation
should be chosen in this case?
A. * Interferon
B. Bacteriophage
C. Diphtheria antitoxin
D. Antidiphtheric antitoxic serum
E. Eubiotic
551. A person was selling "homemade pork" sausages at the market. State sanitary inspector suspected
falcification of the sausages. What serological immune reaction can identifiy food substance?
A. * Precipitation test
C. Complement-fixation test
D. Immunofluorescence test
E. Agglutination test
552. From the nasopharynx of a 5-year-old child it was excreted a microorganism which is identical to
Corynebacterium diphtheriae dose according to morphological and biochemical signs. But this
microorganism does not produce exotoxin. As a result of what process can this microorganism
become toxigenic?
A. * Phage conversion
D. Chromosome mutation
553. A 50-year-old patient with typhoid fever was treated with Levomycetin, next day his condition
became worse, temperature rose to 39,60С. What caused the complication?
C. Reinfection
D. Allergic reaction
554. A large-scale reaction with parapertussis and pertussis diagnosticums was made in order to make
serological diagnostics of the whooping cough. At the bottom of the test-tubes with diagnosticum
of Bordetella parapertussis a granular sediment formed. What antibodies did this reaction reveal?
A. Bacteriolysins
B. Precipitins
C. Antitoxins
D. Opsonins
E. * Agglutinins
555. A 27-year-old woman has dropped penicillin containing eye drops. In a few minutes there
appeared feeling of itching, burning of the skin, lips and eyelids edema, whistling cough,
decrease of BP. What imunoglobulins take part in the development of this allergic reaction?
556. It is planned to use the territory of an old cattle burial ground (which is not used for more than 50
years) for building houses. But ground analysis revealed presence of the pathogen of a very
dangerous illness. Which of the indicated microorganisms is likely to remain in the ground for
such a long time?
A. * Bacillus anthracis
B. Brucella abortus
C. Francisella tularensis
D. Mycobacterium bovis
E. Yersinia pestis
557. Blood analysis of a patient showed signs of HIV infection (human immunodeficiency virus).
Which cells does HIV-virus primarily affect?
E. Mast cells
558. A patient had been suffering from profuse diarrhea and vomiting for 2 days. He died from acute
dehydration. Autopsy revealed that the intestinal wall was edematic and hyperemic, with multiple
haemorrhages in the mucous membrane. Intestine lumen contains whitish fluid resembling of rice
water. What disease caused death?
A. * Cholera
B. Dysentery
C. Salmonellosis
D. Enterocolitis
E. Typhoid fever
559. A 19 year old woman suffers from primary syphilis. Doctor administered her complex therapy
that includes benzylpenicillin sodium salt. What is the mechanism of action of this drug?
560. During examination of a patient a dentist revealed a lot of "white spots" - zones of enamel
demineralization. What microorganisms take part in the development of this process?
A. * Streptococcus agalactia
B. * Streptococcus mutans
C. Staphylococcus epidermidis
D. Veilonella parvula
E. Streptococcus pyogenes
561. 48 hours after tuberculine test (Mantoux test) a child had a papule 10 mm in diameter on the spot
of tuberculine injection. What hypersensitivity mechanism underlies these changes?
A. * Cellular cytotoxicity
B. Granulomatosis
C. Anaphylaxy
D. Immunocomplex cytotoxicity
E. Antibody-dependent cytotoxicity
562. 48 hours after tuberculine test (Mantoux test) a child had a papule 10 mm in diameter on the spot
of tuberculine injection. What hypersensitivity mechanism underlies these changes?
A. * Cellular cytotoxicity
B. Granulomatosis
C. Anaphylaxy
D. Immunocomplex cytotoxicity
E. Antibody-dependent cytotoxicity
563. While studying a microslide obtained from the punctuate of a regional lymph node and stained by
Romanovsky-Giemsa method a physician revealed some light-pink thin microorganisms with 12-
14 regular spiral coils and pointed ends, up to 10-13 micrometer long. This might be the causative
agent of the following disease:
A. * Syphilis
B. Leishmaniasis
C. Leptospirosis
D. Trypanosomiasis
E. Relapsing fever
564. Patient with vomiting, dizziness, sensation of dubble vision, difficult swallowing was admitted to
the hospital. Doctor suspects botulism. What diagnostic methods should be used for diagnosis
approving?
B. Protozoological, microscopical
C. Bacteriological, mycological
E. microscopic
565. A man who was bitten by the unknown dog applied to the surgeon. Wide ragged woundes were
localised on the face. What curative-prophylactic aid should be given to prevent rabies?
566. The immunoblot detected gp120 protein in the blood serum. This protein is typical for the
following disease:
A. *
HIV-infection
B. Tuberculosis
C. Poliomyelitis
D. Syphilis
E. Virus B hepatitis
567. A patient with complaints of 3-day-long fever, general weakness, loss of appetite came to visit
the infectionist. The doctor suspected enteric fever. Which method of laboratory diagnosis is the
best to confirm the diagnosis?
B. Detachment of myeloculture
568. During surgical operation a blood transfusion was made. The blood must be checked to find
antigens of some disease. What disease is expected to be found?
A. * Virus of hepatitis B
B. Virus of hepatitis A
C. Virus of hepatitis E
D. Adenovirus
E. Enterovirus
569. On bacteriological study of rinsing water of the patient with food poisoning, the pure bacterial
culture was inoculated with the following properties: gram-negative motile bacillus in the Endo
environment grows like achromic colony. Representative of what genus has caused the illness?
A. * Salmonella
B. Escherichia
C. Citrobacter
D. Shigella
E. Yersinia
570. From the defecation of a 6-year-old ill child, who has artificial feeding, the intestinal bacillus
with antigen structure 0-111 is excreted. What is the diagnosis?
A. * Coli-enteritis
B. Dysentery-like diseasis
C. Food poisoning
D. Gastroenteritis
E. Cholera-like diseasis
571. Examination of a child revealed some whitish spots looking like coagulated milk on the mucous
membrane of his cheeks and tongue. Analysis of smears revealed gram-positive oval yeast-like
cells. What causative agents are they?
A. * Candida
B. Diphtheria bacillus
C. Fusobacteria
D. Actinomycetes
E. Staphylococci
572. Blood of a patient with presumable sepsis was inoculated into sugar broth. There appeared
bottom sediment. Repeated inoculation into blood agar caused growth of small transparent round
colonies surrounded by hemolysis zone. Examination of a smear from the sediment revealed
gram-positive cocci in form of long chains. What microorganisms are present in blood of this
patient
A. * Streptococci
B. Micrococci
C. Staphylococci
D. Tetracocci
E. Sarcina
573. Bacterioscopy of nasopharyngeal mucus taken from a 2,5 year old child with nasopharyngitis
revealed gram-negative diplococci looking like coffee grains. What organs of the child are most
likely to be affected if these microorganisms penetrate the blood?
A. * Brain tunics
B. Lymph nodes
C. Urogenital tracts
D. Cardiac valves
E. Renal glomeruli
574. A 50-year-old man developed watery diarrhea, little vomiting, nausea, and low-grade fever on the
airplane when he was returning from holidays spent in the Middle East. A stool culture reveals
lactose-positive colonies on MacConkey agar. Which one of the following organisms is the most
likely to be the cause?
A. * Escherichia coli
B. Klebsiella pneumonia
C. Shigella flexneri
D. Salmonella paratyphi A
E. Salmonella enteritidis
575. A child with asphyxia was admitted to the hospital. On examination of the child whitish and spots
difficult to separate were revealed in his larynx. What type of inflammation was in that case?
A. * Serous inflammation
B. Croupous inflammaton
C. Purulent inflammation
D. Diphtheritic inflammation
E. Catarrhal inflammation
576. After consumption some tinned meat a patient had diplopia, acute headache, deglutition disorder,
hard breathing, muscle weakness. The diagnosis was botulism. What factor of pathogenicity are
the clinic presentations of this disease connected with?
A. Hemolysin
B. Endotoxin
C. * Exotoxin
D. Fibrinolysin
E. Plasmocoagulase
577. Large gram-positive rods were isolated from patient’s cutaneous papules. The "string of pearl"
colonies on nutrient agar were demonstrated on penicillin-containing medium. Which one of the
following organisms caused the disease?
A. * Bacillus anthracis
B. Staphylococcus aureus
C. Mycobacteria tuberculosis
D. Corynebacterium diphtheriae
E. Clostridia perfringens
578. A 3-year-old child has a temperature of 38,3C; discrete vesiculoulcerative lesions (Koplik’s
sports) can be seen on the mucous membrane of the mouth. The most probable diagnosis is:
A. * Measles
B. Rubella
C. Herpangina
D. Mumps
E. Scarlet fever
579. A 6-year –old boy was bitten by a dog about the face and neck. What strategy will be used for
post exposure therapy?
D. Use active immunization, than check if adequate antibody titers will be produced
580. In the area being the epicenter of the registered rabies cases among wild animals a 43-year-old
man presented to a clinic and claimed to have been bitten by a stray dog. He was given a course
of anti-rabies vaccine. This preparation relates to the following type of vaccines:
A. * Attenuated
B. Inactivated
C. Molecular
D. Toxoids
E. Synthetic
581. A yond adult male appeared HIV-negative on routine laboratory tests three weeks after the
suspicious sexual contact. A doctor suggested that the patient had “window’’ period. Which one
of the following tests will be most valuable?
A. * PCR
C. Western Blot
D. T4/T8 ratio
E. CD4 counts
582. During the breakout of acute respiratory infection іn order to diagnose influenza the express-
diagnosis, based оn revealing of specific viral antigen іn the examined material
(nasopharyngiallavage), is carried out. Which reaction is used for this?
A. * IFT
B. CFT
C. ELISA
D. IHT
E. HIT
583. A child strung by a honeybee developes respiratory failure and vascular collapse within minutes
and goes into shock. The reaction is caused by which of the following factors:
A. * IgE
B. IgG
C. Macrophages
D. Sensitized T cells
E. IgM
584. Skin tuberculin test was carried out to a patient with chronic lung tuberculosis. Restricted
hyperemia and edema appeared in the site of inracutaneous introduction of tuberculin preparation
during 24-48 hours. What cells are primary effectors in this mechanism reaction?
A. * T-lymphocytes
B. Neutrophils
C. B-lymphocytes
D. Endotheliocytes
585. A patient develops induration in skin-test with purified protein derivative (PPD) to determine
previous exposure to Mycobacterium tuberculosis within 48 hours. Histologically, what kinds of
cells can be found in the inoculation site?
B. B cells
C. Basophils
D. Eosinophils
E. Neutrophils
586. Allergic diagnostic tests are used for the diagnosis of many infectious diseases (tuberculosis,
brucelosis, tularemia etc). Papula and redness in the place of the allergen injection appear, if
diagnose was confirmed. Antigens interaction reaction is conditioned by:
587. ?22-old man presents with a painless 1-cm ulcer on the shaft of his penis. Inguinal
lymphadenopathy is present. The patient admits trading drugs for sex and has several sexual
partners. An RPR test is positive; however, a Gram stain of a swab specimen from the ulcer
shows no bacteria. Treponema patlidum, the causative agent of syphilis,cannot be visualized by
light microscopy because
A. It is transparent
588. An unculturable gram-positive microorganism has been visualized in tissue specimens obtained
from patients with a previously undescribed disease. Which of the following techniques would
be most useful in identifying this organism?
A. Serology
589. A patient with a central nervous system disorder is maintained on the drug methyldopa;
Hemolytic anemia develops, which resolves shortly after the drug is withdrawn. This is MOST
probably an example of:
A. atopic hypersensitivity
B. * cytotoxic hypersensitivity
C. immune-complex hypersensitivity
D. cell-mediated hypersensitivity
E. delayed-type hypersensitivity
590. Hemolytic disease of the newborn caused by Rh blood group incompatibility requires maternal
antibody to enter the fetal bloodstream. Therefore, the mediator of this disease is:
A. IgA antibody
B. SIgA antibody
C. IgE antibody
D. IgM antibody
E. * IgG antibody
591. Your patient is a child who has no detectable T and B cells. This immunodeficiency is most
probably the result of a defect in:
B. the thymus
E. phagocytosis
592. A respiratory disease spread by the droplet aerosol route during winter, causing severe headache,
cough, fever, malaise and congestion is most compatible with which of the following as an
etiological agent?
A. Mumps
B. Influenza C
C. Chicken pox
D. Hepatitis A Virus
E. * Influenza A virus
593. An acute respiratory disease spread by the droplet aerosol route during winter is most compatible
with which of the following as an etiological agent?
A. Chicken pox
B. Mumps
C. Influenza C
D. * Influenza A
E. Hepatitis B Virus
594. Vaccines protect us against dangerous viral diseases by training the body to recognize
and destroy specific invading viruses.Vaccinez contain:
B. antibiotics
E. DNA of virus
595. If you had a phage suspension in a test tube, which of the following techniques would you use to
determine the number of viruses in the tube?
A. spectrophotometric analysis
E. * plaque assay
596. Staphylococcus aureus is easily transmitted within the hospital environment. Which feature of
this bacterium is most largely responsible for its ease of spread?
E. Ability to form spores which are resistant to disinfectants and easily dispersed.
597. Bacteroides fragilis is isolated from an abdominal abscess. Which phrase is the most appropriate?
598. An old woman has a urinary tract infection caused by E. coli. By what route are such infections
most commonly acquired?
A. * Fecal-oral route, followed by colonization of the intestine and then the perineum.
D. Intestinal infection, followed by spread of bacteria to the urethra via the bloodstream.
599. A child at-risk for Tetanus is treated with Tetanus immune globulin (human). What is a major
advantage of this method for immunization over the use of Tetanus toxoid?
600. A 20-year-old student has a deep laceration. He received the standard four-doses of Diphtheria-
Pertussis-Tetanus immunizations in infancy, and a booster at age 5, but has had no anti-Tetanus
immunization since. Which of the following would it be most appropriate to administer?
B. Tetanus toxoid.
601. Five days ago a 65-year-old woman with a lower urinary tract infection began taking ampicillin.
She now has a fever and severe diarrhea. Of the organisms listed, which one is MOST likely to be
the cause of the diarrhea?
A. Bacteroides fragilis
B. * Clostridium difficile
C. Proteus mirabilis
D. Bordetella pertussis
E. Vibrio cholerae
602. A stool sample is plated on standard Endo-agar plates. After overnight incubation some of the
colonies are dark pink in color, others are pale, nearly colorless. What can you conclude about the
bacteria that produced dark-pink colonies?
A. Gram-positive
B. * Ferment lactose.
D. Non-motile.
E. Produce capsules.
603. A 35-year-old woman presents with a history of increasing respiratory distress and cough
productive of sputum. Chest X-ray shows evidence of bilateral pneumonia. Cultures of blood and
sputum on Chocolate agar grow out slender Gram-negative rods; cultures on standard sheep
blood agar are negative. Which bacterium is most likely to be the cause of her infection?
A. Mycoplasma pneumoniae.
B. Chlamydia pneumoniae
C. * Haemophilus influenzae.
D. Escherichia coli.
E. Streptococcus pneumoniae.
604. You suspect Chlamydia infection in a sexually-active young woman. Which test would be most
likely to provide results which would confirm this diagnosis?
605. A 23-year-old medical student had headache and fever one evening. By the next morning she had
become very ill, with high fever, severe headache, and stiff neck. She was admitted to the
hospital. Later that day she developed rash, at first petechial and then purpuric. Gram stain of
CSF showed many white cells and Gram-negative cocci, many in pairs. Which organism is most
likely to be the cause of her infection?
A. Escherichia coli.
B. Haemophilus influenzae.
C. *
Neisseria meningitidis.
D. Streptococcus agalactiae.
E. Streptococcus pneumoniae.
606. A patient with septicemia goes into septic shock. Blood cultures later showed the pathogen to be
Escherichia coli. Which component of the bacteria was most likely to have produced shock?
A. Capsular polysaccharide.
B. Teichoic acid.
C. A superantigen toxin.
D. * Lipopolysaccharide [LPS].
607. A patient has cellulitis of the left arm, persistent fever, and a heart murmur. He is a
habitual user of illegal drugs, which he self-administers intravenously. An
echocardiogram shows a bacterial vegetation on the aortic valve. Three blood cultures
all grow an alpha-hemolytic Gram-positive coccus, catalasenegative. Which organism is
most likely to have caused his infection?
D. Staphylococcus aureus.
E. * A Viridans-group Streptococcus.
608. A sexually-active woman develops a urinary tract infection which ascends to the kidneys. Blood
culture yields a ?-hemolytic Gram-negative rod. In this situation, which pathogen is most
common?
A. * Escherichia coli.
B. Klebsiella pneumoniae.
C. Pseudomonas aeruginosa.
D. Salmonella enteritidis.
E. Serratia marcescens.
609. A child has got a “cold” which progresses to severe pharyngitis. Pharyngeal exudate forms a
tough ‘membrane’, difficult to remove without causing bleeding. Culture of a throat swab on
selective tellurite agar produces black colonies.Which microorganism does have caus this
infection?
A. Corynebacterium xerosis
B. Bordetella pertussis.
C. Candida albicans.
D. * Corynebacterium diphtheriae.
E. Mycobacterium avium/intracellulare.
610. A patient has rapidly spreading soft-tissue infection with gas in infected tissue. Fluid from a
blister contains many large Gram-positive rods which grow on sheep blood agar anaerobically but
not aerobically. Which pathogen is most consistent with these findings?
A. Bacillus anthracis.
B. Bacillus cereus.
C. Clostridium difficile.
D. * Clostridium perfringens.
E. Listeria monocytogenes.
611. Untreated pharyngitis produced by Group A streptococci can result in later development of Acute
rheumatic fever, created by cross-reaction of anti-streptococcal antibodies with heart tissue.
Which streptococcal antigen stimulates formation of these cross-reactive antibodies?
A. Group A carbohydrate.
B. Flagella.
C. * M protein
D. Peptidoglycan.
E. Teichoic acid.
612. A child is diagnosed with Pertussis. How was this infection most likely to have been acquired?
613. A mother is worried that the child may have contracted Lyme disease. Which finding would most
strongly suggest that she is correct?
C. A lesion at the site of the tick bite that develops a thick, black, crusted, scab.
614. A 25-year-old female presented with a three-day history of fever, headache and a non-productive
cough. X-rays show bilateral diffuse lung infiltrates. Culture of a throat swab (on sheep blood and
chocolate agars) revealed only “normal oral flora”. Penicillin G was prescribed and the patient
sent home. Her illness did not respond to this antibiotic. She was then successfully treated with
erythromycin. Which organism is most likely to have caused her illness?
A. * Mycoplasma pneumoniae.
B. Haemophilus influenzae.
C. Mycobacterium tuberculosis.
D. Streptococcus pyogenes.
E. Streptococcus pneumoniae.
615. A patient has a positive Rapid Plasma Reagin (RPR) test for ‘non-treponemal antibodies’. A test
for specific anti-treponemal antibodies is positive but the patient has no rash, genital lesions,
inflammation, or discharge. Which conclusion best follows from these findings
A. The patient does not require antibiotic treatment; absence of symptoms indicates that the
positive RPR is a ‘biological false positive”.
B. The patient has Syphilis, but does not require antibiotic treatment, because infections with
C. * The patient has Syphilis, but the diagnosis should be confirmed by culture before antibiotic
616. A vacationer has “Traveler’s diarrhea” produced by enterotoxigenic E. coli. His profuse watery
diarrhea is produced by a protein exotoxin secreted by this bacterium. Which statement below
best describes the effect of this exotoxin?
E. Causes rearrangement of the enterocyte cytoskeleton and disappearance of the brush border.
617. The severe diarrhea of cholera is produced by a protein exotoxin that acts on enterocytes. Which
statement below best describes the effect of Cholera toxin?
B. Inactivates ribosomes.
618. A medical student with a three-week history of respiratory illness is treated with erythromycin [a
macrolide]. Which phrase best describes the process inhibited by this drug?
C. Cross-linking of peptidoglycan.
D. Polymerization of peptidoglycan.
619. An elderly resident of a nursing home develops pneumococcal pneumonia. This illness might
have been prevented had the patient been immunized against this organism. Which antigens of S.
pneumoniae are included in the adult vaccine?
A. M-proteins.
C. * Capsular polysaccharides.
620. Before the advent of immunization, outbreaks of meningococcal disease were frequent in military
camps. A factor in development of such outbreaks is the ability of Neisseria meningitidis to
establish an asymptomatic ‘carrier state’. What is the predominant site of carriage of Neisseria?
A. Urethral epithelium.
B. Gall bladder.
C. Large intestine.
D. * Nasopharynx.
A. Antibiotic-associated colitis.
B. Diphtheria.
C. Anthrax.
D. * Botulism.
E. Tetanus.
622. In August, a young man presents with a history of fever for 1 week and a red rash which extends
to the palms and soles. IgM titer to Rickettsia rickettsii is 1:640. which vector of this infection
was From?
A. * Dog tick.
C. Blood-sucking sandfly.
E. Flea.
623. In a patient with systemic febrile illness, which item in the history would indicate the highest risk
for Brucella infection?
624. A patient becomes infected with Salmonella enterica serotype typhi. Which condition would most
strongly favor development of a “chronic carrier state” with this organism?
C. * Gallstones.
D. Kidney stones.
625. A patient has an abscess from which Bacterioides fragilis is isolated. Which condition is most
likely to have facilitated infection by this organism?
A. Treatment with trimethoprim/sulfa.
B. * Tissue ischemia.
C. Presence of neutrophils.
626. A patient develops pneumonia caused by Streptococcus pneumoniae. The predominant virulence
factor of this organism is a thick capsule . Which phrase best describes the role of this capsule in
virulence?
627. A patient has a peptic ulcer. Which organism is most likely to have created this lesion?
A. Campylobacter jejuni.
B. Enterococcus faecalis.
C. Escherichia coli.
D. * Helicobacter pylori
E. Salmonella enteritidis.
628. A 50-year-old woman was treated for cystitis [caused by E. coli] with an oral broad-spectrum
antibiotic. Three weeks later she developed abdominal pain and diarrhea. A stained smear of stool
contained many neutrophils. Cultures for bacterial pathogens were negative. A serological test of
stool for Clostridium difficile toxin was positive. Which of the following is most likely to have
been the immediately predisposing cause of this patient’s C. difficile infection?
A. Borrelia burgdorferi
B. Chlamydia pneumoniae
C. Franciscella tularensis
D. Yersinia pestis.
E. * Rickettsia rickettsii
630. A 13-year old boy develops pain and marked (10 cm.) swelling in his right inguinal lymph node
accompanied by fever of 104oF. He and his family have just returned from a vacation trip to
Arizona and New Mexico. Aspirate from the node contains Gram-negative rods. A direct
immunofluorescent assay on the aspirate provides a definitive diagnosis. The child is
(successfully) treated with streptomycin, and the aspirate is sent for culture to the CDC. Culture
yields pale colonies on MacConkey agar. What is the most probable pathogen?
A. * Yersinia pestis.
B. Helicobacteri pylori.
C. Franciscella tularensis
D. Coxiella burnetii
E. Klebsiella pneumoniae
631. A 15-year-old female presented with a three-day history of fever, headache and a non-productive
cough. Penicillin was prescribed and the patient was sent home. Her illness did not respond to
this antibiotic. Gram stain and culture of the patient's sputum and a throat swab revealed only
'normal oral flora'. She was then successfully treated with erythromycin. Which organism below
is most likely to have caused her illness?
A. * Mycoplasma pneumoniae.
B. Haemophilus influenzae.
C. Mycobacterium tuberculosis
D. Corynebacterium diphtheriae
E. Streptococcus pneumoniae.
632. Which organism below, if present in only one of three blood cultures from a single patient, is
most likely to represent contamination caused by poor antiseptic technique, rather than genuine
infection?
A. Escherichia coli.
B. * Staphylococcus epidermidis.
C. Streptococcus pneumoniae.
D. Pseudomonas aeruginosa
E. Haemophilus influenzae
633. An 81-year-old man with chronic emphysemea and hypertension develops fever, chills, and
respiratory distress. X-rays show lobar pneumonia. Sputum smear contains numerous Gram-
positive cocci, many of which are in pairs. Sputum culture on Sheep blood agar produces many
colonies of an alpha-hemolytic Gram-positive coccus. It is catalase-negative, Bile-esculin
negative, and optochin-sensitive. Which of the organisms below is most likely?
C. Streptococcus mitis
E. * Streptococcus pneumoniae
634. A post-surgical patient develops redness around the incision and a day later pus begins to ooze
from the wound. Culture of a swab of pus produces beta-hemolytic colonies on Sheep blood agar.
Gram stain shows that the bacteria are Gram-positive cocci. Catalase and coagulase tests are
positive. Which of the following organisms is most likely?
A. Streptococcus pyogenes
B. Streptococcus agalactiae
C. Streptococcus mitis
D. * Staphylococcus aureus
E. Staphylococcus epidermidis
635. Vacationer develops watery diarrhea but not chills or fever. In this setting which of the following
pathogens is the most common?
A. Salmonella enterica
B. * Escherichia coli
C. Campylobacter jejuni
D. Helicobacter pylori
E. Staphylococcus aureus
636. A young woman develops urgency, frequency, and dysuria. Two days later she develops fever,
chills, and costovertebral angle tenderness. Tests of urine for leukocyte esterase and nitrate
reductase are positive. Urine culture yields a beta-hemolytic Gram-negative rod which forms
dark-pink colonies on MacConkey agar. Which of the following organisms is most likely?
A. Pseudomonas aeruginosa
B. Klebsiellla pneumoniae
C. * Escherichia coli
D. Staphylococcus saprophyticus
E. Enterobacter, spp.
637. A child develops a "cold" which progresses to severe pharyngitis. Pharyngeal exudate forms a
tough 'membrane', difficult to remove without causing bleeding. Culture of a throat swab on
selective tellurite agar produces abundant black colonies. Other cultures produce only normal
flora. Which organism below is most likely to have caused this infection?
A. Streptococcus pneumoniae.
B. Haemophilus influenzae.
C. * Corynebacterium diphtheriae.
D. Bordetella pertussis.
E. Mycobacterium avium/intracellulare.
638. A burn patient develops infection of the burn wound. Culture of exudate on Brain-Heart Infusion
agar produces numerous green-pigmented colonies of a Gram-negative rod. The organism grows
well aerobically on standard media but shows no evidence of acid production on differential
media such as MacConkey, Hektoen Enteric, or Triple-Sugar-Iron agars.
A. Bacteroides fragilis
B. Campylobacter fetus
C. Eschericahia coli
D. * Pseudomonas aeruginosa
E. Citrobacter spp.
639. A young man develops peritonitis following abdominal trauma. During surgery foul-smelling
purulent material is aspirated from the infected area. Gram stain of this material contains a
mixture of Gram-positive cocci, Gram-positive rods, and many Gram-negative rods. Aerobic
culture on sheep blood agar yields many colonies of enterococci and a few colonies of E. coli;
culture on MacConkey agar produces only E. coli. Anaerobic culture on sheep blood agar
produces a few colonies of beta-hemolytic Gram-positive rods and many colonies of slender
Gram-negative rods.
A. Salmonella typhi
B. Clostridium tetani
C. Clostridium perfringens
D. * Pseudomonas aeruginosa
E. Bacteroides fragilis
640. An outbreak of sepsis caused by Staphylococcus aureus has occurred in the newborn nursery.
You are called upon to investigate. According to your knowledge of the normal flora, what is the
MOST likely source of the organism?
A. Colon
B. * Nose
C. Throat
D. Vagina
641. A 75-year-old man reports malaise, headache, and fever. On examination his neck is stiff. Gram
stain of CSF reveals neutrophils and numerous Gram-negative cocci, many in pairs.
A. Pseudomonas aeruginosa
B. Klebsiella pneumoniae
C. Haemophilus influenzae
D. * Neisseria gonorrhoeae
E. Neisseria meningitidis
642. Your patient is a 30-year-old woman with nonbloody diarrhea for the past 14 hours. Which one of
the following organisms is LEAST likely to cause this illness?
A. Clostridium difficile
B. * Streptococcus pyogenes
C. Shigella dysenteriae
D. Salmonella enteritidis
643. A 50-year-old man has a fever and is coughing up. The patient has contacted with pigeons.
Which one of the following microorganism is MOST likely to be the cause?
A. Listeria monocytogenes
B. Clostridium perfringens
C. Peptostreptococcus intermedins
D. * Chlamydia psittaci
E. Fusobacterium nucleatum
644. A group of medical students developed nausea, vomiting, and abdominal cramping a few hours
after eating sandwiches and potato salad at the school cafeteria. They had no diarrhea. By the
next morning they were fatigued but recovered.
A. Salmonella typhi
B. Borrelia burgdorferi
C. Clostridium botulinum
D. * Clostridium perfringens
E. Staphylococcus aureus
645. A 19-year-old man comes to clinic purulent urethral discharge . On Gram-stain of the discharge
huge numbers of neutrophils are seen, a few of which contain intracellular Gram-negative cocci.
Many of the cocci are in pairs. What is the most likely organism?
A. Neisseria meningitidis
B. * Neisseria gonorrhoeae.
C. Haemophilus ducreyi.
D. Mycoplasma hominis
E. Treponema pallidum.
646. A culture of skin lesions from a patient with pyoderma (impetigo) shows numerous colonies sur
rounded by a zone of beta hemolysis on a blood agar plate. A Gram-stained smear shows gram-
positive cocci in chain. If you found the catalase test to be negative, which one of the following
organisms would you MOST probably have isolated?
A. * Streptococcus pyogenes
B. Staphylococcus aureus
C. Staphylococcus epidermidis
D. Streptococcus pneumoniae
647. Your patient is a 70-year-old man who underwent bowel surgery for colon cancer 3 days ago. He
now has a fever and abdominal pain. You are concerned that he may have peritonitis. Which one
of the following pairs of organisms is MOST likely to be the cause?
648. A 65-year-old man develops dysuria and hematuria. A Gram stain of a urine sample shows gram-
negative rods. Culture of the urine on EMB agar reveals lactose-negative colonies with out
evidence of swarming motility. Which one of the following organisms is MOST likely to be the
cause of his urinary tract infection?
A. Enterococcusfaecalis
B. * Proteus vulgaris
C. Pseudomonas aeruginosa
D. Escherichia coli
E. Staphylococcus epidermidis
649. A 25-year-old man complains of a urethral discharge. You perform a Gram stain on a specimen
of the discharge and see neutrophils but no bacteria. Of the organisms listed, the one MOST
likely to cause the discharge is
A. Treponema pallidum
B. * Chlamydia trachomatis
C. Candida albicans
D. Coxiella burnetii
E. Neisseria gonorrheae
650. Two hours after a delicious Thanksgiving dinner of barley soup, roast turkey, stuffing, sweet
potato, green beans, cranberry sauce, and pump kin pie topped with whipped cream, the Smith
family of four experience vomiting and diarrhea. Which one of the following organisms is MOST
likely to cause these symptoms?
A. Shigella flexneri
B. Helicobacter pylori
C. * Campylobacter jejuni
D. Salmonella enteritidis
E. Vibrio cholerae
651. A patient had abdominal pain. Doctor suspects gastritis caused by Helicobacter pylori. What
below used to confirm diagnosis?
A. Sputum culture.
652. A man suffers many month from gastritis caused by Helicobacter pylori. Of the tests below,
which would it be best to order, to confirm your diagnosis?
C. Sputum culture.
653. Epidemiologists (using the Kauffman-White classification) report that an outbreak has been
caused by Salmonella enterica, serotype Newport. Structure of which pair of antigens determined
the serotype as Newport?
654. Bacteriologists (using the Kauffman-White classification) made conclusion that outbreak has
been caused by Salmonella enterica, serotype Anatum. Structure of which pair of antigens
determined the serotype as Anatum?
655. A patient with a bacterial infection is treated with a bacteriostatic inhibitor of 70S
ribosomes.Which drug below best fits this description?
A. Rifampin.
B. A fluoroquinolone.
C. * Tetracycline.
D. An aminoglycoside.
E. Erythromycin.
656. The young man was injuring in car accident. The affected area of the leg is painful, red, and
swollen. During the next 12 hours the swelling increases markedly, the skin of the leg becomes
discolored, and fluid-filled blisters appear on the skin. Gram stain of fluid aspirated from a blister
reveals large Gram-positive rods. Which organism below is most likely to have caused this
infection?
A. Clostridium botulinum
B. * Clostridium perfringens.
C. Clostridium tetani.
D. Listeria monocytogenes
E. Clostridium difficile.
657. A patient has sat pelvic inflammatory disease (PID).Doctor suspected that it is caused by
Chlamydia trachomatis. what test be used to which useto confirm C. trachomatis?
658. An HIV-positive patient develops mycobacterial infection. Two likely possibilities are
Mycobacterium avium-intracellulare and Mycobacterium tuberculosis. How would these bacteria
be best distinguished in the laboratory?
A. * Nucleic acid from one hybridizes to a specific DNA probe, nucleic acid from the other does not.
A. Mycobacterium tuberculosis
B. * Corynebacterium diphtheriae
C. Legionella pneumophilia.
D. Pseudomonas aeruginosa
E. Nocardia asteroids
660. A listless child with extremely poor muscle tone is brought to the Hospital. Which condition is
most likely, given this presentation?
A. Lyme Disease
B. Tetanus
C. * Botulism
E. Rheumatic fever
661. A patient has a large carbuncles you suspect is caused by Bacillus anthracis. In this case, which of
the following would you expect to see in a Gram-stained smear of pus from the carbuncles?
A. Slender spirochetes.
E. Gram-negative coccobacilli.
662. A young man has urethritis. Titer of Complement-fixing (CF) antibody to Chlamydia
trachomatis, in a serum sample taken on the day on which treatment was initiated, is 1:64. A
second CF titer, taken two weeks later, is 1:640. Which conclusion below best follows from the
antibody titers?
B. Although the patient’s condition may appear to be improving, in fact his infection is getting
worse.
663. You suspect that a patient’s respiratory illness may be due to infection by Mycoplasma
pneumoniae. If this is true, which antibiotic below should you expect to be most effective?
A. Penicillin G.
B. * Erythromycin.
C. Vancomycin.
D. A third-generation cephalosporin.
E. An ‘extended-spectrum’ penicillin.
664. A young man complaines on pain on urination and profuse purulent discharge from his urethral
opening. Gram stain of the discharge revealed large numbers Gram-negative cocci, mostly in
pairs within neutrophils. Which bacterium is most likely to have caused his infection?
A. Mycoplasma hominis
B. Chlamydia trachomatis
C. * Neisseria gonorrhoeae
D. Ureaplasma urealyticum
E. Treponema pallidum
665. Members of a family suffer acute attacks of nausea and vomiting a few hours after returning from
a daylong picnic, at which they ate hamburgers, potato salad, and custard pie. By morning all are
feeling better. Which bacterial toxin is most likely to have caused their symptoms?
666. A mother brings her 12-year-old son to your office because he has a skin lesion on his back. It is
20 cm in diameter, with a red inflamed border and a red center, with a paler ring between the red
areas. The mother says it has expanded rapidly over the last few days. Which of the following
would be most likely to cause such a lesion?
A. Rickettsia rickettsii
B. Treponema pallidum
C. * Borrelia burgdorferi
D. Chlamydia psittaci
E. Streptococcus pyogenes
667. A young man has a large pus-filled abscess on his upper arm but does not go to a doctor. Several
days later he is brought to the Emergency Room with fever, hypotension, and multiple organ
failure. Which of the following is most likely to be responsible?
668. A nine years-old child develops high fever and stiff neck. A spinal tap is performed. A Gram-
stained smear of cerebrospinal fluid reveals many neutrophils and Gram-negative cocci that
resemble paired kidney beans within leucocytes. Which organism below is most likely to have
caused this infection?
A. Escherichia coli
B. Haemophilus influenzae
C. Listeria monocytogenes
D. * Neisseria meningitidis
E. Streptococcus agalactiae
669. A child develops ‘strep throat’ (pharyngitis caused by Streptococcus pyogenes). The pediatrician
explains to the child’s mother that the pharyngitis normally resolves spontaneously but
nevertheless it is important the patient take a full course of Penicillin G. Why is this antibiotic
treatment recommended in this situation?
670. A patient has endocarditis produced by an a-hemolytic Streptococcus. This usually reflects spread
of normal flora to the bloodstream. From which site is this isolate most likely to have spread?
A. Anterior nares.
B. Facial skin.
C. Lower respiratory tract.
D. Distal urethra.
E. * Oropharynx.
671. An elderly man develops pneumonia and septicemia. Gram stain of sputum contains many
neutrophils and Gram-positive cocci, in pairs. Blood and sputum cultures grow out a Gram-
positive coccus, alpha-hemolytic on sheep blood agar, and catalase-negative. Which organism
below is most likely?
A. Staphylococcus epidermidis
B. * Streptococcus pneumoniae
C. Staphylococcus aureus
D. Streptococcus agalactiae
E. Streptococcus pyogenes
672. A 17-year-old sexually-active male presents with a four-day history of pain on urination and
profuse purulent discharge from his urethral opening. Gram stain of the discharge revealed large
numbers of neutrophils. A small fraction of neutrophils contained Gram-negative cocci, mostly in
pairs. Which bacterium is most likely to have caused his infection?
A. Mycoplasma hominis
B. Chlamydia trachomatis
C. Treponema pallidum
D. Ureaplasma urealyticum
E. * Neisseria gonorrhoeae
673. In October a young man comes to your office with two ulcers on his right hand. His history
includes the fact that he is an avid hunter, and has been rabbit hunting in southern New Jersey
every weekend for the last several months. Which organism should his hobby cause you to add to
the list of possible causes of his illness?
A. Yersinia pestis.
B. Coxiella burnetii.
C. Listeria monocytogenes.
E. * Francisella tularensis.
674. Bacteroides fragilis is isolated from an operative wound. Which sentence below best describes
such infection?
A. Can be successfully treated with aminoglycosides alone
E. Sexually-transmitted.
675. Residents of an assisted-living facility are immunized against Streptococcus pneumoniae. What is
the antigenic component of the vaccine?
A. A protein toxoid.
C. Purified toxin.
676. In the upper respiratory tract of patient doctor observed thick grey ‘pseudomembrane’ formation.
Which organism can cause this symptoms?
A. Mycobacterium tuberculosis
B. Legionella pneumophilia.
C. Streptococcus mitis
D. * Corynebacterium diphtheriae
E. Pseudomonas aeruginosa
677. A post-surgical patient develops fever and there is redness and swelling around the surgical
incision. A day later pus begins to flow from the incision; Gram stain of pus shows large numbers
of neutrophils and many Gram-positive cocci, some in clusters. The organism is beta-hemolytic
on blood agar, and is coagulase- and catalase-positive.Which organism is most likely?
A. * Staphylococcus aureus
B. Staphylococcus epidermidis
E. Streptococcus pneumoniae
678. A five-day-old child develops high fever and stiff neck. A spinal tap is performed. A Gram-
stained smear of cerebrospinal fluid reveals many neutrophils and short, thick, Gram-negative
rods of uniform length. This microbes can ferment lactose.Which organism below is most likely
to have caused this infection?
A. * Escherichia coli
B. Streptococcus pneumoniae
C. Streptococcus agalactiae
D. Haemophilus influenzae
E. Neisseria meningitidis
679. After Gram stain of spinal fluid sample from a patient with meningitis Medical personal during
microscopy examination detected Gram-negative cocci, many in pairs. Which organism below is
most probable?
A. * Neisseria meningitidis
B. Haemophilus influenzae
C. Streptococcus pneumoniae
D. Pseudomonas aeruginosa
E. Escherichia coli
680. A patient presented with upper back pain. MRI imaging demonstrated inflammation of the T8-T9
vertebral bodies consistent with osteomyelitis. A bone marrow biopsy, when cultured, produced
colonies of Gram-positive cocci.Colonies on Sheep blood agar were 3-4 mm in diameter, off-
white and beta-hemolytic. Colonies on Brain-heart infusion (BHI) agar were 2-3 mm in diameter
and golden-yellow. Catalase and coagulase tests were positive.
A. * Staphylococcus aureus
B. Staphylococcus epidermidis
C. Streptococcus pyogenes
D. Streptococcus pneumoniae
E. Streptococcus mitis
681. A 20-year-old women presents at a local clinic with fever and abdominal tenderness. She reports
having had unprotected sexual intercourse a few weeks earlier. Gram-negative diplococci are
seen in a Gram stain of vaginal secretions; many of the bacteria are inside polymorphonuclear
leukocytes. Which virulence factor is most important in aiding this bacterium to initiate
infection?
B. * Adhesive pili.
C. Anti-phagocytic capsule.
D. Endotoxin.
E. Flagella.
682. Encapsulated strains of Neisseria meningitidis are more virulent than non-encapsulated isolates.
How does a capsule enhance virulence?
683. Before the advent of immunization, outbreaks of meningococcal disease were frequent in military
camps. A factor in development of such outbreaks is the ability of Neisseria meningitidis to
establish an asymptomatic 'carrier state'. What is the predominant site of carriage of
meningococci?
A. Urethral epithelium.
B. Gall bladder.
C. * Nasopharynx.
D. Large intestine.
684. A 23-year-old medical student had headache and fever one evening. By the next morning she had
become very ill, with high fever, severe headache, and stiff neck. She was admitted to the
hospital. Later that day she developed rash, first petechial and then pupuric. Her white count was
26,000/ull, with 80% neutrophils and 10% 'bands'. Gram stain ofCSF showed many white cells
and numerous bacteria. If the pathogen is Neisseria meningitidis, how would you
685. A patient with septicemia goes into septic shock. Blood cultures later show the pathogen to be
Neisseria meningitidis. Which component of the bacteria was most likely to be responsible for
producing shock?
A. * Lipopolysaccharide [LPS].
B. Teichoic acid.
C. A superantigen toxin.
D. Capsular polysaccharide.
686. A stool sample is plated on standard MacConkey-agar plates. After overnight incubation some of
the colonies are dark pink in color, others are pale, nearly colorless. What can you conclude about
the bacteria that produced dark- pink colonies?
A. Gram-positive.
B. * Ferment lactose.
D. Non-motile.
E. Produce capsules.
687. An engineer recently returned from visiting his family in India develops severe diarrhea. He in
fact has cholera. His diarrhea is produced by a protein exotoxin secreted by Vibrio cholerae. How
does this toxin act?
688. A vacationing couple both develop severe watery diarrhea which lasts three days.
A. Shigella sonnei.
B. Pseudomonas aeruginosa.
C. Salmonella typhi.
D. * Campylobacter jejuni.
E. Escherichia coli.
689. During a vacation at a Caribbean resort, a 26-year-old executive develops profuse watery
diarrhea. If his illness is produced by E. coli, which virulence factor of this organism is most
likely to be responsible for the diarrhea?
B. * A plasmid-borne gene which encodes a heat-stable peptide toxin that stimulates guanyl cyclase.
C. A pathogenicity island which enables E. coli to invade mammalian cells.
690. Several medical students go to Cancun over spring break. One evening, at a local bar, they drink
several pitchers of Margaritas made with crushed ice. They spend the next two days in their hotel
room with severe watery diarrhea, but no chills or fever. By the third day they recover
completely. Which organism is most likely to have been the cause of their illness?
A. Non-typhoidal Salmonella.
B. Helicobacter pylori.
C. Campylobacter jejuni.
D. * Enterotoxigenic E. coli.
E. Non-cholera Vibrio.
691. A sexually-active woman develops a urinary tract infection which ascends to the kidneys
(pyelonephritis). Blood culture yields a beta-hemolytic Gram-negative rod. In this situation,
which pathogen below is most common?
A. Salmonella enteritidis.
B. Pseudomonas aeruginosa.
C. Serratia marcescens.
D. Klebsiella pneumoniae.
E. * Escherichia coli.
692. 23-year-old woman comes to your office because, for 3 days, she has experienced burning with
urination, increased frequency of urination, and a continual feeling that she needs to urinate. She
does not have vaginal discharge, fever, or flank pain. Rapid 'dipstick' urine tests are consistent
with uncomplicated cystitis. Culture of urine on standard media produces a lactose-fermenting
Gram-negative rod.In situations such as this, what is the most likely pathogen?
A. Enterobacter faecium
B. * Escherichia coli
C. Proteus vulgaris
D. Klebsiella pneumoniae
E. Pseudomonas aeruginosa
693. A young child develops bloody diarrhea, produced by Shigella infection. How had he
gotten sicrness&
A. Cow, horse, or sheep.
B. Rare hamburger.
C. Dog or cat.
694. A patient becomes infected with Salmonella enterica serotype typhi. Which condition would most
strongly favor development of a "chronic carrier state" with this organism?
C. * Presence of gallstones.
695. A patient has chronic gastritis. Which of the following organisms is most likely?
A. A non-cholera Vibrio.
B. * Helicobacter pylori.
C. Campylobacter fetus.
D. Campylobacter jejuni.
E. Escherichia coli.
696. Gastroscopy reveals that a patient with stomach pain has a large ulceration of his gastric mucosa.
During the procedure a biopsy of the stomach epithelium is obtained.
A. * Curved or spiral bacteria are seen in the biopsy after silver staining.
E. Bacteria are found in stomach fluid, but not in contact with the gastric epithelium
697. A patient with systemic febrile illness is found to have an infection with Brucella abortus. Which
item in his history represents a known risk factor for such infections?
698. In Europe human Tetanus is now a rare disease. Which measure below has been most important
in preventing it?
699. A 61-year-old diabetic man is admitted to the hospital for severe pain in his right leg; the
diagnosis is obstruction of the femoral artery. The affected area of the leg is painful, red, and
swollen. During the next 12 hours the swelling increases markedly, the skin of the leg becomes
discolored, and fluid-filled blisters appear on the skin. Gram stain of fluid aspirated from a blister
reveals large Gram-positive rods. Which organism below is most likely to have caused this
infection?
A. Clostridium botulinum.
B. Clostridium difficile.
C. * Clostridium perfringens.
D. Clostridium tetani.
E. Listeria monocytogenes
700. 16-year-old boy has a skin lesion on his back. It is 20 cm in diameter, with a red inflamed border
and a red center, with a paler ring between the red areas. He says it has expanded rapidly over the
last few days after tick bite. Which of the following would be most likely to cause such a lesion?
A. Rickettsia rickettsii
B. * Borrelia burgdorferi
C. Treponema pallidum
D. Chlamydia psittaci
E. Streptococcus pyogenes
701. Five young men suffer acute attacks of nausea and vomiting a few hours after returning from a
student’s party, at which they ate hamburgers, potato salad, and custard pie. By morning all are
feeling better. Which bacterial toxin is most likely to have caused their symptoms?
A. Helicobacteri pylori Cytotoxin.
702. A patient with Cystic fibrosis develops pneumonia. A Gram-negative rod is cultured from her
respiratory tract. On nutrient-poor media it grows well and secretes large amounts of bright green
pigment. It produces acid from none of the sugars on which it was tested (including lactose and
sucrose). Which organism below is most appropriate?
A. . Shigella sonnei
B. Salmonella typhi
C. * Pseudomonas aeruginosa
D. Escherichia coli
E. Klebsiella pneumoniae
703. A 25-year-old woman presents with a swollen, warm, painful knee. Aspirated joint fluid is cloudy
and when cultured on chocolate agar gives rise to oxidase-positive colonies of Gram-negative
diplococci. Which bacterium is most likely to be the cause of this infection?
A. Haemophilus influenzae
B. Staphylococcus aureus
C. Streptococcus pneumoniae
D. * Neisseria gonorrhoeae
E. Nocardia asteroides
A. Haemophilus influenzae
B. * Neisseria gonorrhoeae
C. Streptococcus pneumoniae
D. Staphylococcus aureus
E. Nocardia asteroides
705. You suspect that a patient might have latent Syphilis. Which test below would most reliably
detect it?
706. An 18-year-old high school student has a large pus-filled lesion on the left forearm, tender, red,
and swollen. It is incised, drained, washed, and bandaged. A smear of the pus is Gram-stained.
Many neutrophils are seen, also many Gram-positive cocci, most in large clusters. The organism
is b-hemolytic on sheep blood agar. Colonies are pale-yellow, opaque, and shiny, with neat round
outlines. Of the bacteria listed below, which is most likely to have produced the abscess?
A. * Staphylococcus aureus
B. Streptococcus bovis
C. Streptococcus pneumoniae
D. Streptococcus mitis
E. Enterococcus faecalis
707. A workman has a large pus-filled red, and swollen lesion on the left forearm. A smear of the pus
is Gram-stained. Many neutrophils are seen, also many Gram-positive cocci, most in large
clusters. The organism is b-hemolytic on sheep blood agar. Colonies are pale-yellow, opaque, and
shiny, with neat round outlines. Of the bacteria listed below, which is most likely to have
produced the abscess?
A. Enterococcus faecalis
B. * Staphilococcus aureus
C. Streptococcus bovis
D. Streptococcus pneumoniae
E. Streptococcus mitis
708. A cyanotic, listless, infant with extremely poor muscle tone is brought to the Emergency Room.
Which condition is most likely, given this presentation?
A. * Botulism
B. Tetanus
C. Lyme Disease
709. A patient has a blackskin ulcer Doctor suspects that it is caused by Bacillus anthracis. If this
hypothesis is true, which of the following mscroorganisms are in exudate from the wound?
A. Slender spirochetes.
E. Gram-negative coccobacilli.
710. As a result of durative antibiotic therapy a 37-year old patient developed intestinal dysbacteriosis.
What type of drugs should be used in order to normalize intestinal microflora?
A. * Eubiotics
B. Sulfanilamides
C. Bacteriophages
D. Autovaccines
E. Vitamins
711. Among junior children of an orphanage an outbreak of intestinal infection with signs of
colienteritis was registered. In order to identify isolated causative agent it is necessary to:
712. Urine examination of a patient with acute cystitis revealed leukocytes and a lot of gram-
negative bacilli. Inoculation resulted in growth of colonies of mucous nature that formed green
soluble pigment. What microorganism is the most probable cause of the disease?
A. * Pseudomonas aeruginosa
B. Escherihia coli
C. Klebsiella pneumoniae
D. Proteus mirabilis
E. Salmonella enteritidis
713. A laboratory received a material from a patient's wound. Preliminary diagnosis is gaseous
gangrene. What microbiological method should be applied to determine species of causative
agent?
A. * Bacteriological
B. Allergic
C. Bacterioscopic
D. Serological
E. RIA
714. A virological laboratory obtained pathological material (mucous discharges from nasal meatuses)
taken from a patient with provisional diagnosis "influenza". What quick test will allow to reveal
specific viral antigen in the material under examination?
D. Radioimmunoassay
E. ELISA
715. In the surgical department of a hospital there was an outbreak of hospital infection that showed
itself in often postoperative wound abscesses. Bacteriological examination of pus revealed
Staphylococcus. What examination shall be conducted to find out the source of this causative
agent among the department personnel?
A. * Phagotyping
B. Microscopical examination
C. Serological identification
E. Biochemical identification
716. A 7 year old child often suffers from streprococcal angina. Doctor suspected development of
rheumatism and administered serological examination. The provisional diagnosis will be most
probably confirmed by presence of antibodies to the following streptococcal antigen:
A. * O-streptolysin
B. C-carbohydrate
C. M-protein
D. Erythrogenic toxin
E. Capsular polysaccharide
717. A culture of monkey cells (Vero) and a group of sucking mouse were infected with an inoculum
taken from a child with provisional diagnosis "enterovirus infection". There was no cytopathic
effect on the cell culture but mouse sucking died. What enteric viruses might have caused disease
of this child?
A. * Coxsackie A
B. Coxsackie B
C. ECHO virus
D. Polioviruses
718. A patient has been suffering from elevated temperature and attacks of typical cough for 10 days.
Doctor administered inoculation of mucus from the patient's nasopharynx on the agar. What
microorganism is presumed?
A. * Pertussis bacillus
B. Pfeiffer's bacillus
C. Listeria
D. Klebsiella
E. Staphylococcus
719. A patient of surgical department complains about pain in the small of her back and in the lower
part of her belly; painful and frequent urination. Bacteriological examination of urine revealed
gram-negative oxidase-positive rod-like bacteria forming greenish mucoid colonies with specific
smell. What causative agent can it be?
A. * Pseudomonas aeruginosa
B. Proteus mirabilis
C. E.coli
D. Str.pyogenes
E. Mycoplasma pneumonie
720. A female patient underwent liver transplantation. 1,5 month after it her condition became worse
because of reaction of transplant rejection. What factor of immune system plays the leading part
in this reaction?
A. * T-killers
B. Interleukin-1
C. Natural killers
D. B-lymphocytes
E. T-helpers
721. A bacteriological laboratory received sputum sample of a patient suffering from tuberculosis.
Bacterioscopic examination of smears and detection of tuberculosis bacillus can be realized by
one of enrichment methods that involves processing of sputum only with solution of caustic soda.
What is this method called?
A. * Homogenization
B. Inactivation
C. Flotation
D. Filtration
E. Neutralization
722. A pregnant woman was registered in an antenatal clinic and underwent complex examination for
a number of infections. Blood serum contained IgM to the rubella virus. What is this result
indicative of?
A. * Of primary infection
B. Of a chronic process
723. A 65-year-old man has purulent abscess on his neck. Analyses revealed a culture of gram-positive
cocci with plasmocoagulase activity. This culture relates most likely to:
A. * Staphylococcus aureus
B. Streptococcus pyogenes
C. Staphylococcus epidermidis
D. Staphylococcus saprophyticus
E. Klebsiella pneumoniae
724. Material taken from a patient with provisional diagnosis "influenza" was referred to a laboratory.
For virological examination the hemadsorption reaction was applied. This reaction can be applied
for detection of the following viruses:
A. * Viruses containing hemagglutinins
D. DNA-genomic viruses
E. Any viruses
725. Inoculum from pharynx of a patient ill with angina was inoculated into blood-tellurite agar. It
resulted in growth of grey, radially striated (in form of rosettes) colonies 4-5 mm in diameter.
Gram-positive bacilli with clublike thickenings on their ends placed in form of spread wide apart
fingers are visible by microscope. What microorganisms are these?
A. * Diphtheria corynebacteria
B. Botulism clostridia
C. Diphtheroids
D. Streptococci
E. Streptobacilli
726. During examination of a patient a dentist revealed a lot of "white spots" - zones of enamel
demineralization. What microorganisms take part in the development of this process?
A. * Streptococcus mutans
B. Streptococcus salivarius
C. Streptococcus pyogenes
D. Veilonella parvula
E. Staphylococcus epidermidis
727. Planned mass vaccination of all newborn 5-7 day old children against tuberculosis plays an
important role in tuberculosis prevention. In this case the following vaccine is applied:
A. * BCG
E. TABTe
728. A 4-year-old child presents with general weakness, sore throat and deglutitive problem. After his
examination a doctor suspected diphtheria and sent the material to the bacteriological laboratory.
In order to determine the diphtheria causative agent the material should be inoculated into the
following differential diagnostic medium:
B. Endo agar
C. Ploskyrev agar
D. Sabouraud agar
E. Levenshtein-Yessen agar
729. Vomiting matters of a patient suspected of having cholera were delivered to the bacteriological
laboratory. The material was used for preparing a "hanging drop" specimen. What type of
microscopy will be applied for identification of the causative agent by its mobility?
A. * Phase-contrast microscopy
B. Electron microscopy
D. Fluorescence microscopy
E. Immersion microscopy
730. Analysis of the cerebrospinal fluid of a child with signs of purulent lesion of brain tunics revealed
gram-negative bean-shaped diplococci. What presumptive diagnosis can be made on the basis of
the analysis results?
A. * Meningitis
B. Gonorrhea
C. Cholera
D. Plague
E. Anthrax
731. A patient has a necrotizing phlegmon of his lower extremity. A doctor suspects a gas gangrene.
Microscopy reveals Gram-positive bacilli. In order to confirm the diagnosis further
bacteriological tests should include inoculation of the material into the following nutrient
medium:
A. * Kitt-Tarozzi medium
B. Endo agar
C. Levine agar
D. Meat-peptone agar
E. Milk-salt agar
732. A 26-year-old female patient with bronchitis has been administered a broad spectrum antibiotic as
a causal treatment drug. Specify this drug:
A. * Doxycycline
B. Interferon
C. BCG vaccine
D. Ambroxol
E. Dexamethasone
733. 10 days after having quinsy caused by beta-hemolytic streptococcus a 6-year-old child exhibited
symptoms of glomerulonephritis. What mechanism of glomerular lesion is most likely in this
case?
A. * Immunocomplex
B. Cellular cytotoxicity
C. Anaphylaxis
D. Atopy
734. 6 hours after the initial inoculation of water sample into 1% peptone water, the growth of a
culture in form of a thin pellicle on the medium surface was registered. Such cultural properties
are typical for the causative agent of the following disease:
A. * Cholera
B. Plague
C. Tuberculosis
D. Dysentery
E. Pseudotuberculosis
735. Analysis of the cerebrospinal fluid of a child with signs of purulent lesion of brain tunics revealed
gram-negative bean-shaped diplococci. What presumptive diagnosis can be made on the basis of
the analysis results?
A. * Meningitis
B. Gonorrhea
C. Cholera
D. Plague
E. Anthrax
736. Microbiological studies of air in the pharmacy room revealed the presence of pathogenic
staphylococci. Select the medium in which you can detect the lecithinase activity of the isolated
microorganism:
A. * Yolk-salt agar
B. Blood agar
D. Sugar agar
E. Meat-extract agar
737. There is a suspicion of active tuberculosis development in patient. The doctor has appointed
Mantoux test to make a diagnosis. What immunobiological agent has to be administered?
A. * Tuberculin
B. BCG vaccine
C. DPT vaccine
D. Tularin
E. DT vaccine
738. A young woman suddenly developed fever up to 39oC accompanied by a strong headache.
Examination revealed marked nuchal rigidity. Spinal puncture was performed. Gram-stained
smear of cerebrospinal fluid contained many neutrophils and Gram-negative diplococci. What
bacteria could be the cause of this disease?
A. * Neisseria meningitidis
B. Streptococcus pneumonia
C. Haemophilus influenzae
D. Staphylococcus aureus
E. Pseudomonas aeruginosa
739. In a village, a case of anthrax had been registered. Medical services began epidemiologically
indicated specific prophylaxis of population against anthrax. What preparation was used for this
purpose?
A. * Live vaccine
B. Inactivated vaccine
C. Chemical vaccine
740. A 55-year-old patient with a characteristic rash, fever, dizziness has been admitted to a hospital.
He has been provisionally diagnosed with typhus. No similar cases have been reported. In his
youth (15 years old) the patient suffered typhus in a boarding school. What disease is it?
A. * Brill disease
B. Typhoid fever
C. Measles
D. Rubella
E. Cholera
741. A patient has severe catarrhal symptoms. Material growth on Bordet-Gengou agar showed
mercury-drop like colonies. Examination of the blood smears revealed some small ovoid gram
positive bacilli sized 1-3 microns. What microorganisms were isolated?
A. * Bordetella
B. Corynebacteria
C. Mycobacteria
D. Meningococcus
E. Brucella
742. The laboratory for especially dangerous infections conducts microscopic examination of
pathological material from a patient with suspected plague. The sample was stained by Burri-
Gins technique. What property of the causative agent can be identified by this technique?
A. * Capsule formation
B. Spore formation
C. Acid resistance
D. Alkali resistance
743. A child cut his leg with a piece of glass while playing and was brought to the clinic for the
injection of tetanus toxoid. In order to prevent the development of anaphylactic shock the serum
was administered by Bezredka method. What mechanism underlies this method of desensitization
of the body?
744. A 32-year-old patient undergoing dental examination was found to have some rash-like lesions
resembling secondary syphilis in the oral cavity. The patient was referred for the serological
study with the purpose of diagnosis confirmation. In order to detect antibodies in the serum,
living Treponema were used as diagnosticum. What serological test was performed?
A. * Immobilization
B. Neutralization
C. Complement binding
D. Precipitation
E. Passive hemagglutination
745. Examination of a 27-year-old donor who had not donated blood for a long time revealed HBs
antibodies detected by ELISA method. In this case, the positive ELISA results indicate that the
donor:
A. * Had hepatitis B
746. In the area being the epicenter of the registered rabies cases among wild animals a 43-year-old
man presented to a clinic and claimed to have been bitten by a stray dog. He was given a course
of anti-rabies vaccine. This preparation relates to the following type of vaccines:
A. * Attenuated
B. Inactivated
C. Molecular
D. Toxoids
E. Synthetic
747. A patient has been hospitalised with provisional diagnosis of virus B hepatitis. Serological
reaction based on complementation of antigen with antibody chemically bound to peroxidise or
alkaline phosphatase has been used for disease diagnostics. What is the name of the applied
serological reaction?
A. * Immune-enzyme analysis
B. Radioimmunoassay technique
C. Immunofluorescence test
D. Bordet-Gengou test
E. Antigen-binding assay
748. A 3-year-old child has continuous fever, lymph nodes are enlarged, the amount of lymphocytes in
blood is significantly increased. Enzyme linked immunosorbent assay (ELISA) revealed antigen
of Epstein-Barr virus. What diagnosis can be made based on the information given above?
A. * Infectious mononucleosis
B. Burkitt’s lymphoma
C. Herpetic lymphadenopathy
E. Cytomegalovirus infection
749. An outbreak of an intestinal infection occurred in a kindergarten on the eve of New Year
holidays. Bacteriological examination of patients’ faeces didn’t reveal any pathogenic bacteria.
Electronmicroscopy revealed roundish structures with clear outer edges and a thick core
resembling a wheel. Specify the most likely causative agent of this infection:
A. * Rotavirus
B. Adenovirus
C. Coxsacki-virus
D. E. coli
E. P. vulgaris
750. A hospitalized patient bitten by a rabid animal has an avulsive wound of shin. What kind of
vaccine must be given to prevent rabies?
A. * Anti-rabies vaccine
B. DTaP
C. Td
D. BCG
E. TABte
751. A patient has been admitted to the infectious diseases department for malaise, fever up to 38oC,
jaundice. A few months ago, the patient underwent blood transfusion. The doctor suspected viral
hepatitis B. What are the principal methods of laboratory diagnosis of hepatitis B?
B. Virus isolation in cell culture and its identification by the cytopathic effects
752. A laboratory received a sample of water used in drug production for sanitary and virological
analysis. What group of viruses will indicate faecal contamination of water and thus the need for
its additional purification?
A. * Picornaviridae
B. Herpesviridae
C. Orthomyxoviridae
D. Retroviridae
E. Flaviviridae
753. A person has been in contact with influenza patient. What drug should be administered for
specific passive influenza prophylaxis?
A. Amizon
C. Leukocytic interferon
D. * Immunoglobulin
E. Anaferon
754. A child entering the school for the first time was given Mantoux test in order to determine if there
was a need for revaccination. The reaction was negative. What is the meaning of this test result?
755.
A patient with suspected dysentery has been admitted to the infectious diseases hospital. Which
basic method of laboratory diagnosis must be applied in the first place?
A. * Bacteriological
B. Serological
C. Allergic
D. Biological
E. Microscopic
756. A 12-year-old boy has been hospitalized for suspected food poisoning. The fecal samples were
inoculated on the Endo agar, which resulted in growth of a large number of colorless colonies.
What microorganism is most likely to be EXCLUDED from the list of possible causative agents
of the disease?
A. * Escherichia coli
B. Salmonella enteritidis
C. Proteus vulgaris
D. Pseudomonas aeruginosa
E. Yersinia enterocolitica
757. A child with suspected tuberculosis was given Mantoux test. After 24 hours the site of the
allergen injection got swollen, hyperemic and painful. What are the main components that
determine such response of the body?
D. B-lymphocytes, IgM
758. A 4-year-old child presents with general weakness, sore throat and deglutitive problem. After his
examination a doctor suspected diphtheria and sent the material to the bacteriological laboratory.
In order to determine the diphtheria causative agent the material should be inoculated into the
following differential diagnostic medium:
B. Endo's agar
C. Ploskyrev's agar
D. Sabouraud's agar
E. Levenshtein-Yessen agar
759. Planned mass vaccination of all newborn 5-7 day old children against tuberulosis plays an
important role in tuberculosis prevention. In this case the following vaccine is applied:
A. * BCG
E. Tuberculin
760. Inoculum from pharynx of a patient ill with angina was inoculated into blood-tellurite agar. It
resulted in growth of grey, radially striated (in form of rosettes) colonies 4-5 mm in diameter.
Gram-positive bacilli with clublike thickenings on their ends placed in form of spread wide apart
fingers are visible by microscope. What microorganisms are these?
A. * Diphtheria corynebacteria
B. Botulism clostridia
C. Diphtheroids
D. Streptococci
E. Streptobacilli
761. During examination of a 3-month old infant a pediatrician revealed that the baby’s oral mucosa
and tongue were covered with a thick white deposit. In the material taken from the affected site a
bacteriologist revealed the presence of yeast fungi giving the reasons for suspecting a fungal
infection which occurs most often in children of this age, namely:
A. * Candidiasis
B. Favus
C. Epidermophytosis
D. Actinomycosis
E. Trichophytia
762. Diphtheria exotoxin had been treated with 0,3-0,4% formalin and kept in a thermostat for 30 days
at a temperature of 40oC. What preparation was obtained as a result of these manipulations?
A. * Anatoxin (toxoid)
B. Antitoxin
C. Diagnosticum
D. Therapeutic serum
E. Diagnostic serum
763. A female patient has been treated with antibiotics for a long time. Thereafter examination of
smears form vaginal secretion revealed oval cells with well-defined nucleus, some cells gemmate.
What preparations can help to confirm the diagnosis "candidosis"?
A. * Antifungal
B. Antibacterial
C. Antichlamydial
D. Antiviral
E. Antiprotozoal
764. One of mass production drugs is produced by inactivation of bacterial exotoxin by formalin.
What is this drug for?
E. For immunocorrection
765. A patient is suspected to have the typhoid fever. What method of laboratory diagnostics would be
the most appropriate for confirmation of this diagnosis in the first week of disease?
A. * Hemoculture identification
C. Myeloculture identification
D. Biliculture identification
E. Coproculture identification
766. Inoculation of chicken embryos is the main method of detection of influenza virus. In order to
neutralize associated bacterial flora in the material under examination (nasopharyngeal lavage) it
is necessary to add beforehand:
A. * Antibiotics
B. Eubiotics
C. Fluorescent serum
D. Leukocytic interferon
767. Analysis of sputum obtained from a patient with suspected pneumonia revealed gram-positive
diplococci. They were slightly elongated, with the pointed opposite ends. What microorganisms
were revealed in the sputum?
A. * Streptococcus pneumoniae
B. Staphylococcus aureus
C. Klebsiella pneumoniae
D. Neisseria meningitidis
E. Streptococcus pyogenes
768. A sick child with diphtheria 10 days after the injection of antitoxic diphtheria serum, there were
skin rash, accompanied by severe itching, increased body temperature to 38, there were pains in
the joints. What is the cause of these events you expect?
A. * Delayed-type hypersensitivity
B. Contact allergy
C. Anaphylactic reaction
D. Serum sickness
E. Atopy
769. Veterinarny assistant, working on cattle farm went to a doctor complaining of joint pain, fever,
malaise, night sweats. Ill for about a month. Given that the patient is working on a farm and
livestock related complaints, the doctor suspected he had brucellosis. What material is taken from
this patient to be investigated in the usual microbiological laboratory?
A. Sputum
B. * Blood serum
C. Vomit fluid
D. Urine
E. Feces
770. A patient, sick 3 days ago, complaining of the increased temperature of 38 ° C, abdominal pain,
loose stools, presence of blood in the stool, the doctor diagnosed clinically bacterial dysentery.
What method of microbiological diagnosis is advisable to apply in this case and what material
should be taken from the patient to confirm the diagnosis?
A. * Bacteriologycal
B. Serological ,blood
C. Smear ,blood
D. Smear , feces
E. Bacteriology, urine
771. A patients with gastric ulcer with hyperacidity after endoscopy and bacteriological studies were
identified bacteria of the genus Helicobacter. Thanks to some property of these organisms are not
killed in the acidic environment of the stomach?
A. * Urease activity
C. Oxidase activity
D. Catalase activity
E. Resistance to vancomycin
772. A patient with pustular skin lesions isolated pathogen, which is on blood agar forms a rounded,
medium sized, yellow colonies surrounded by a zone of hemolysis. In smears made from the
colonies Gr + cocci appear in clusters were found. Pure culture is oxidase-and catalase-positive,
ferments mannitol, synthesizes plasma coagulase. Which of the following type matches the
selected agent?
A. * Staphylococcus aureus
B. Staphylococcus saprophyticus
C. Streptococcus agalactiae
D. Staphylococcus epidermidis
E. Streptococcus pyogenes
773. A patient, 68 years old, was subjected to prolonged antibiotic therapy. After such treatment, the
patient appeared whitish coating on the oral mucosa. Microscopic examination of smear revealed
large, round, various sizes, gram positive microorganisms. What should be done to continue the
microbiological diagnosis?
774. In brain cells of fox’s, trapped in the city had been found the inclusion – Babesh- Negri bodies.
What is the causative agent of fox’s infection?
A. * Rabies
B. Flu
C. Chickenpox
E. Kissing disease
775. A patient visited a dentist with complaints of redness and edema of his mouth mucous membrane
in a month after dental prosthesis. The patient was diagnosed with allergic stomatitis. What type
of allergic reaction by Gell and Cumbs underlies this disease?
A. * Cytotoxic
C. Anaphylactic
D. Stimulating
E. Immunocomplex
776. Donor skin transplantation was performed to a patient with extensive burns. On the 8-th day the
graft became swollen and changed colour; on the 11-th day graft rejection started. What cells take
part in this process?
A. * B-lymphocytes
B. Erythrocytes
C. T-lymphocytes
D. Basophils
E. Eosinophils
777. A patient with suspected diphtheria went through bacterioscopic examination. Examination of
throat swab revealed rod-shaped bacteria with volutin granules. What etiotropic preparation
should be chosen in this case?
A. Interferon
B. Bacteriophage
C. Diphtheria toxoid
E. Eubiotic
778. A person was selling "homemade pork" sausages at the market. State sanitary inspector suspected
falcification of the sausages. What serological immune reaction can identifiy food substance?
A. * Precipitation test
C. Complement-fixation test
D. Immunofluorescence test
E. Agglutination test
779. From the nasopharynx of a 5-year-old child it was excreted a microorganism which is identical to
Corynebacterium diphtheriae dose according to morphological and biochemical signs. But this
microorganism does not produce exotoxin. As a result of what process can this microorganism
become toxigenic?
A. * Phage conversion
D. Chromosome mutation
780. A 50-year-old patient with typhoid fever was treated with Levomycetin, next day his condition
became worse, temperature rose to 39,60С. What caused the complication?
C. Reinfection
D. Allergic reaction
781. A patient who came to the doctor because of his infertility was administered to make tests for
toxoplasmosis and chronic gonorrhoea. Which reaction should be performed to reveal latent
toxoplasmosis and chronic gonorrhoea of the patient?
B. Immunoblot analysis
783. It is planned to use the territory of an old cattle burial ground (which is not used for more than 50
years) for building houses. But ground analysis revealed presence of the pathogen of a very
dangerous illness. Which of the indicated microorganisms is likely to remain in the ground for
such a long time?
A. * Bacillus anthracis
B. Brucella abortus
C. Francisella tularensis
D. Mycobacterium bovis
E. Yersinia pestis
784. A patient had been suffering from profuse diarrhea and vomiting for 2 days. He died from acute
dehydration. Autopsy revealed that the intestinal wall was edematic and hyperemic, with multiple
haemorrhages in the mucous membrane. Intestine lumen contains whitish fluid resembling of rice
water. What disease caused death?
A. * Cholera
B. Dysentery
C. Salmonellosis
D. Enterocolitis
E. Typhoid fever
785. A 19 year old woman suffers from primary syphilis. Doctor administered her complex therapy
that includes benzylpenicillin sodium salt. What is the mechanism of action of this drug?
786. During examination of a patient a dentist revealed a lot of "white spots" - zones of enamel
demineralization. What microorganisms take part in the development of this process?
A. Streptococcus pyogenes
B. * Streptococcus mutans
C. Staphylococcus epidermidis
D. Veilonella parvula
E. Streptococcus pyogenes
787. 48 hours after tuberculine test (Mantoux test) a child had a papule 10 mm in diameter on the spot
of tuberculine injection. What hypersensitivity mechanism underlies these changes?
A. * Cellular cytotoxicity
B. Granulomatosis
C. Anaphylaxy
D. Immunocomplex cytotoxicity
E. Antibody-dependent cytotoxicity
788. 48 hours after tuberculine test (Mantoux test) a child had a papule 10 mm in diameter on the spot
of tuberculine injection. What hypersensitivity mechanism underlies these changes?
A. * Cellular cytotoxicity
B. Granulomatosis
C. Anaphylaxy
D. Immunocomplex cytotoxicity
E. Antibody-dependent cytotoxicity
789. While studying a microslide obtained from the punctuate of a regional lymph node and stained by
Romanovsky-Giemsa method a physician revealed some light-pink thin microorganisms with 12-
14 regular spiral coils and pointed ends, up to 10-13 micrometer long. This might be the causative
agent of the following disease:
A. * Syphilis
B. Leishmaniasis
C. Leptospirosis
D. Trypanosomiasis
E. Relapsing fever
790. Patient with vomiting, dizziness, sensation of dubble vision, difficult swallowing was admitted to
the hospital. Doctor suspects botulism. What diagnostic methods should be used for diagnosis
approving?
B. Protozoological, microscopical
C. Bacteriological
D. mycological
791. A man who was bitten by the unknown dog applied to the surgeon. Wide ragged woundes were
localised on the face. What curative-prophylactic aid should be given to prevent rabies?
792. A patient with complaints of 3-day-long fever, general weakness, loss of appetite came to visit
the infectionist. The doctor suspected enteric fever. Which method of laboratory diagnosis is the
best to confirm the diagnosis?
B. Detachment of myeloculture
793. During surgical operation a blood transfusion was made. The blood must be checked to find
antigens of some disease. What disease is expected to be found?
A. * Virus of hepatitis B
B. Virus of hepatitis A
C. Virus of hepatitis E
D. Adenovirus
E. Enterovirus
794. A man was admitted to the hospital on the 5th day of disease that manifested itself by jaundice,
muscle aching, chill, nose bleedings. In course of laboratory diagnostics a bacteriologist
performed dark-field microscopy of the patients blood drop. Name a causative agent of this
disease
A. * Leptospira interrogans
B. Calymmatobacterium granulomatis
C. Rickettsia mooseri
D. Borrelia dutlonii
E. Bartonella bacilloformis
795. From the defecation of a 6-year-old ill child, who has artificial feeding, the intestinal bacillus
with antigen structure 0-111 is excreted. What is the diagnosis?
A. * Coli-enteritis
B. Dysentery-like diseasis
C. Food poisoning
D. Gastroenteritis
E. Cholera-like diseasis
796. Blood of a patient with presumable sepsis was inoculated into sugar broth. There appeared
bottom sediment. Repeated inoculation into blood agar caused growth of small transparent round
colonies surrounded by hemolysis zone. Examination of a smear from the sediment revealed
gram-positive cocci in form of long chains. What microorganisms are present in blood of this
patient
A. * Streptococci
B. Micrococci
C. Staphylococci
D. Tetracocci
E. Sarcina
797. Bacterioscopy of nasopharyngeal mucus taken from a 2,5 year old child with nasopharyngitis
revealed gram-negative diplococci looking like coffee grains. What organs of the child are most
likely to be affected if these microorganisms penetrate the blood?
A. * Brain tunics
B. Lymph nodes
C. Urogenital tracts
D. Cardiac valves
E. Renal glomeruli
798. A gram-negative heavily encapsulated diplococcus was isolated from the cerebrospinal fluid of
12-year old boy with acute purulent meningitis. It was oxidase positive and ferment glucose and
maltose, but not lactose. The disease most probably caused by which one of the following
organism:
A. * Neisseria meningitides
B. Staphylococcus epidermidis
C. Haemophilus influenza
D. Escherichia coli
E. Neisseria gonorrhoeae
A. * Escherichia
B. Staphylococcus
C. Salmonella
D. Shigella
E. Streptococcus
800. A 50-year-old man developed watery diarrhea, little vomiting, nausea, and low-grade fever on the
airplane when he was returning from holidays spent in the Middle East. A stool culture reveals
lactose-positive colonies on MacConkey agar. Which one of the following organisms is the most
likely to be the cause?
A. * Escherichia coli
B. Klebsiella pneumonia
C. Shigella flexneri
D. Salmonella paratyphi A
E. Salmonella enteritidis
801. Vomiting matters of a patient suspected of having cholera were delivered to the bacteriological
laboratory. The material was used for preparing a "hanging drop" specimen. What type of
microscopy will be applied for identification of the causative agent by its mobility?
A. * Phase-contrast microscopy
B. Electron microscopy
D. Immersion microscopy
E. Fluorescence microscopy
802. A child with asphyxia was admitted to the hospital. On examination of the child whitish and spots
difficult to separate were revealed in his larynx. What type of inflammation was in that case?
A. Serous inflammation
B. Croupous inflammaton
C. Purulent inflammation
D. * Diphtheritic inflammation
E. Catarrhal inflammation
803. After consumption some tinned meat a patient had diplopia, acute headache, deglutition disorder,
hard breathing, muscle weakness. The diagnosis was botulism. What factor of pathogenicity are
the clinic presentations of this disease connected with?
A. Hemolysin
B. Endotoxin
C. * Exotoxin
D. Fibrinolysin
E. Plasmocoagulase
804. Large gram-positive rods were isolated from patient’s cutaneous papules. The "string of pearl"
colonies on nutrient agar were demonstrated on penicillin-containing medium. Which one of the
following organisms caused the disease?
A. * Bacillus anthracis
B. Staphylococcus aureus
C. Mycobacteria tuberculosis
D. Corynebacterium diphtheriae
E. Clostridia perfringens
805. There was a record of some anthrax cases among animals in a countryside. The spread of disease
can be prevented by means of immunization. What kind of vaccine should be used?
A. * STI live vaccine
B. BCG vaccine
C. Salk vaccine
D. Sabin's vaccine
806. A virological laboratory obtained pathological material (mucous discharges from nasal meatuses)
taken from a patient with provisional diagnosis "influenza". What quick test (rapid/ express-test)
will allow to reveal specific viral antigen in the material under examination?
B. CFT
C. immunoassay
E. Radioimmunoassay
807. A 3-year-old child has a temperature of 38,3C; discrete vesiculoulcerative lesions (Koplik’s
sports) can be seen on the mucous membrane of the mouth. The most probable diagnosis is:
A. * Measles
B. Rubella
C. Herpangina
D. Mumps
E. Scarlet fever
808. A 6-year –old boy was bitten by a dog about the face and neck. What strategy will be used for
post exposure therapy?
D. Use active immunization, than check if adequate antibody titers will be produced
809. A hospitalized patient bitten by a rabid animal has an avulsive wound of shin. What kind of
vaccine must be given to prevent rabies?
A. * Anti-rabies vaccine
B. DTaP
C. Td
D. BCG
E. TABte
810. In the area being the epicenter of the registered rabies cases among wild animals a 43-year-old
man presented to a clinic and claimed to have been bitten by a stray dog. He was given a course
of anti-rabies vaccine. This preparation relates to the following type of vaccines:
A. * Attenuated
B. Inactivated
C. Molecular
D. Toxoids
E. Synthetic
811. ?A child had been administered antidiphtheric serum. What resistance was formed in the child?
A. Active
B. Primary
C. * Passive
D. Pathologic
E. Physiological
812. A factory producing typhoid fever vaccine cultivates bacteria of virulent strain in optimal nutrient
medium. Then the cells are separated from culture fluid by means of centrifugation and processed
with formalin. What type of vaccine is it?
A. * Inactivated
B. Autovaccine
C. Attenuated
D. Chemical
E. Anatoxin
813. During blood transfusion a patient has developed intravascular erythrocyte hemolysis. What kind
of hypersensitivity does the patient have?
A. I type (anaphylactic)
B. * II type (antibody-dependent)
E. IV type (granulomatosis)
814. Lymphocytes and other cells of our body synthesize universal antiviral agents as a response to
viral invasion. Name these protein factors:
A. Cytokines
C. * Interferon
D. Interleukin - 2
E. Interleukin - 4
815. 10 days after having quinsy caused by beta-hemolytic streptococcus a 6-year-old child exhibited
symptoms of glomerulonephritis. What mechanism of glomerular lesion is most likely in this
case?
B. Cellular cytotoxicity
C. Anaphylaxis
D. * Immunocomplex
E. Atopy
816. A patient has been diagnosed with acute glomerulonephritis that developed after he had had
streptococcal infection. It is most likely that the affection of basal glomerular membrane is caused
by an allergic reaction of the following type:
A. Anaphylactic
B. Cytotoxic
C. * Immune complex
D. Delayed
E. Stimulating
817. A 30 year old woman has applied a lipstick with a fluorescent substance for a long time. Then she
got a limited erythema and slight peeling on her lip border, later there appeared transversal striae
and cracks. Special methods of microscopic examination of the affected area helped to reveal
sensibilized lymphocytes and macrophages in the connective tissue; cytolysis. What type of
immunological hypersensitivity was developed?
B. I type (reaginic)
E. Granulomatosis
818. In order to speed up healing of a wound of oral mucosa a patient was prescribed a drug that is a
thermostable protein occuring in tears, saliva, mother’s milk as well as in a new-laid hen’s egg. It
is known that this protein is a factor of natural resistance of an organism. What is it called?
A. Interleukin
B. * Lysozyme
C. Complement
D. Interferon
E. Imanine
819. A patient visited a dentist with complaints of redness and edema of his mouth mucous membrane
in a month after dental prosthesis. The patient was diagnosed with allergic stomatitis. What type
of allergic reaction by Gell and Cumbs underlies this disease?
A. Stimulating
B. Cytotoxic
C. Immunocomplex
D. Anaphylactic
820. Purulent endometritis developed in a woman after delivery. Treating with antibiotics inhibitors of
murein synthesis was ineffective. Wide spectrum bactericidal antibiotic was administered to her.
In 6 hours temperature rapidly increased up to 400C with shiver. Muscle pains have appeared. BP
dropped down to 70/40 mmHg. Oligurea has developed. What is the main reason for the
development of this condition?
B. * Endotoxic shock
C. Internal bleeding
D.
Anaphylactic shock
E. Bacteremia
821. During the breakout of acute respiratory infection in order to diagnose influenza the express-
diagnosis, based on revealing of specific viral antigen in the examined material (nasopharyngial
lavage), is carried out. Which reaction is used for this?
A. Opsonization
B. Complement binding
C. * Immunofluorescence
D. Agglutination
E. Precipitation
822. During blood transfusion a patient has developed intravascular erythrocyte hemolysis. What kind
of hypersensitivity does the patient have?
A. I type (anaphylactic)
D. * II type (antibody-dependent)
E. IV type (granulomatosis)
823. 10-year-old child had the Mantoux tuberculin test administered. 48 hours later a papule up to 8
mm in diameter appeared on the site of the injection. What type of hypersensitivity reaction
developed after the tuberculin injection?
A. Seroreaction
B. Atopic reaction
D. Arthus phenomenon
824. Bacteritic preparations are subdivided into groups according to their purpose and production
principles. What group do the preparations for initiation of active immunity relate to?
A. Bacteriophages
B. Immune sera
C. * Vaccines
D. Immunoglobulins
E. Monoclonal antibodies
825. A 27- year-old woman has used penicillin containing eye drops. In a few minutes itching, skin
burning, lips and eyelids edema, whistling cough, decreasing BP appeared. What antibodies can
lead to this allergic reaction?
826. Live vaccine is injected into the human body. Increasing activity of what cells of connective
tissue can be expected?
827. A patient was diagnosed with autoimmune hemolitic cytotoxic anemia. What substances are
antigens in II type allergic reactions?
A. Inflammation modulators
B. * Antibiotics
D. Hormones
E. Serum proteins
828. A 30 year old woman has applied a lipstick with a fluorescent substance for a long time. Then she
got a limited erythema and slight peeling on her lip border, later there appeared transversal striae
and cracks. Special methods of microscopic examination of the affected area helped to reveal
sensibilized lymphocytes and macrophages in the connective tissue; cytolysis. What type of
immunological hypersensitivity was developed?
A. I type (reaginic)
E. Granulomatosis
A. Tuberculine reaction
B. Cytotoxic reaction
D. Delayed-type hypersensitivity
E. * Anaphylactic reaction
830. A patient recovered from Sonne dysentery and was once more infected with the same causative
agent. What is such infection form called?
A. Chronic infection
B. Recidivation
C. Superinfection
D. Persisting infection
E. * Reinfection